Location via proxy:   [ UP ]  
[Report a bug]   [Manage cookies]                

After The Completion of English Solutions.: Mtyynja0K1Nhasblaxjhbitpbgjzbmfhnebnbwfpbc5Jb20Ruvvfu1Rjt04Gma

Download as pdf or txt
Download as pdf or txt
You are on page 1of 141

2019 - Test 4-

Exam Title :
History & Cu...
Email : ilbsnaa4@gmail.com
Contact :

Note: If the exam is multi-lingual i.e. English and Hindi. Hindi solutions will be
after the completion of English solutions.
QUESTION 1. MTYyNjA0K1NhaSBLaXJhbitpbGJzbmFhNEBnbWFpbC5jb20rUVVFU1RJT04gMA==
Consider the following statements regarding Mughal empire.

1. Aurangzeb’s narrow minded policies alone are stated to be the main reason for the fall of
Mughal Empire.

2. Bahadur shah adopted more tolerant attitude towards Hindu chiefs and rajas

3. There was more destruction of temples during the reign of Bahadur shah

Choose the correct code

a) 1 and 2 only
b) 2 and 3 only
c) 2 only
d) 3 only
Correct Answer: C
Your Answer: Unanswered
Explanation

Solution (c)

The Mughal Empire declined and disintegrated during the first half of the eighteenth century. A
study of the processes of the decline of this great Empire reveals the fact that there were some
defects and weaknesses of India’s medieval social, economic and political structure which were
responsible for the eventual subjugation of the country by English east India Company.

The unity and stability of the empire has been shaken up during long and strong reign of
Aurangzeb; yet in spite of his many harmful policies, the Mughal administration was quite
efficient and Mughal army quite strong at the time of his death in 1707. Moreover the Mughal
dynasty still commanded the respect in the country.

On Aurangzeb’s death is 65 year old son Bahadur shah came to power. He followed a policy of
compromise and conciliation, and there was evidence of reversal of some of the narrow minded
policies and measures adopted by Aurangzeb. He adopted more tolerant attitude towards Hindu
chiefs and rajas. There was no destruction of temples during the reign of Bahadur shah.

QUESTION 2. MTYyNjA0K1NhaSBLaXJhbitpbGJzbmFhNEBnbWFpbC5jb20rUVVFU1RJT04gMQ==
Which of the following reasons can be considered to be valid for exploration of sea route to
India by Europeans

1. Europeans wanted to find an alternative route to India as the route through Red sea was
controlled by Arabs

2. They were influenced by the spirit of renaissance which compelled them to explore new
routes.

3. Advances in ship building and navigation encouraged them to undertake adventurous sea
voyages to reach unknown places in the east.

IASbaba
Web: http://ilp.iasbaba.com/ Score:
Email: ilp@iasbaba.com 0.00 / 200
Page 1
2019 - Test 4-
Exam Title :
History & Cu...
Email : ilbsnaa4@gmail.com
Contact :

Choose the correct code

a) 1 and 2 only
b) 2 and 3 only
c) 1 and 3 only
d) All of the above
Correct Answer: D
Your Answer: Unanswered
Explanation

Solution (d)

After the decline of the Roman Empire in the seventh century, the Arabs had established their
domination in Egypt and Persia. Direct contact between the Europeans and India declined and,
with that, the easy accessibility to the Indian commodities like spices, calicoes, silk, and various
precious stones that were greatly in demand was affected.

In 1453, Constantinople fell to the Ottoman Turks, who were on the ascendant. Merchandise
from India went to the European markets through Arab Muslim intermediaries. The Red Sea
trade route was a state monopoly from which Islamic rulers earned tremendous revenues. The
land routes to India were also controlled by the Arabs. In the circumstances, the Europeans
were keen to find a direct sea route to India. Fifteenth-century Europe was gripped by the spirit
of the Renaissance with its call for exploration. At the same time, Europe made great advances
in the art of ship-building and navigation. Hence, there was eagerness all over Europe for
adventurous sea voyages to reach the unknown corners of the East.

QUESTION 3. MTYyNjA0K1NhaSBLaXJhbitpbGJzbmFhNEBnbWFpbC5jb20rUVVFU1RJT04gMg==
Consider the following statements about Portuguese in India.

1. Vasco da Gama was the first European to reach the Indian coast in ship.

2. Factory at Calicut was built by Vasco da Gama

3. Calicut, Cochi and Cannanore were the important trading centres during Portuguese stay in
India

Choose the correct code

a) 1 and 2 only
b) 1 and 3 only
c) 2 and 3 only
d) All of the above
Correct Answer: B
Your Answer: Unanswered
Explanation

Solution (b)

Portuguese navigator, Bartholomew Dias in 1487 rounded the Cape of Good Hope in Africa and
sailed up the eastern coast; he was well convinced that the long sought after sea route to India

IASbaba
Web: http://ilp.iasbaba.com/ Score:
Email: ilp@iasbaba.com 0.00 / 200
Page 2
2019 - Test 4-
Exam Title :
History & Cu...
Email : ilbsnaa4@gmail.com
Contact :
had been found. But it was only ten years later that an expedition of Portuguese ships headed
out to India (in 1497) and arrived in India in slightly less than eleven months’ time, in May
1498.

Vasco da Gama arrived in India in 1498. The Hindu ruler of Calicut, the Zamorin (Samuthiri)
accorded a friendly reception to Vasco Da Gama. Vasco da Gama stayed in India for three
months. When he returned to Portugal, he carried back with him a rich cargo and sold the
merchandise in the European market at a huge profit. Later a voyage was undertaken by Pedro
Alvarez Cabral in September 1500 to trade for spices. He established a factory at Calicut.

Do you know?

Vasco da Gama once again came to India in 1501. The Zamorin declined to exclude the Arab
merchants in favour of the Portuguese when Vasco Da Gama combined commercial greed with
ferocious hostility and wreaked vengeance on Arab shipping wherever he could. His rupture
with the Zamorin thus became total and complete. Vasco da Gama set up a trading factory at
Cannanore. Gradually, Calicut, Cannanore and Cochin became the important trade centres of
the Portuguese.

QUESTION 4. MTYyNjA0K1NhaSBLaXJhbitpbGJzbmFhNEBnbWFpbC5jb20rUVVFU1RJT04gMw==
Who among the following adopted the Blue water policy?

a) Vasco da Gama
b) Alfanso de Albuquerque
c) Francisco de Almeida
d) Nino da Cunha
Correct Answer: C
Your Answer: Unanswered
Explanation

Solution (c)

In 1505, the King of Portugal appointed a governor in India for a three-year term and equipped
the incumbent with sufficient force to protect the Portuguese interests.

Francisco De Almeida, the newly appointed governor, was asked to consolidate the position of
the Portuguese in India and to destroy Muslim trade by seizing Aden, Ormuz and Malacca.

He was also advised to build fortresses at Anjadiva, Cochin, Cannanore and Kilwa. What
Almeida, however, encountered along with the opposition of the Zamorin, was a threat from the
Mameluke Sultan of Egypt. Encouraged by the merchants of Venice whose lucrative commerce
was now at risk due to the Portuguese interference, the Egyptians raised a fleet in the Red Sea
to stop the advance of the Portuguese.

In 1507, the Portuguese squadron was defeated in a naval battle off Diu by the combined
Egyptian and Gujarat navies, and Almeida’s son was killed. Next year, Almeida avenged his
defeat by totally crushing the two navies. Almeida’s vision was to make the Portuguese the
master of the Indian Ocean. His policy was known as the Blue Water Policy (cartaze system).

IASbaba
Web: http://ilp.iasbaba.com/ Score:
Email: ilp@iasbaba.com 0.00 / 200
Page 3
2019 - Test 4-
Exam Title :
History & Cu...
Email : ilbsnaa4@gmail.com
Contact :
QUESTION 5. MTYyNjA0K1NhaSBLaXJhbitpbGJzbmFhNEBnbWFpbC5jb20rUVVFU1RJT04gNA==
Who is considered to be the real founder of Portuguese in the east?

a) Francisco de Almeida
b) Alfanso de Albuquerque
c) Nina da Cunha
d) Vasco da Gama
Correct Answer: B
Your Answer: Unanswered
Explanation

Solution (b)

Albuquerque, who succeeded Almeida as the Portuguese governor in India, was the real
founder of the Portuguese power in the East, a task he completed before his death. He secured
for Portugal the strategic control of the Indian Ocean by establishing bases overlooking all the
entrances to the sea.

There were Portuguese strongholds in East Africa, off the Red Sea, at Ormuz; in Malabar; and
at Malacca. The Portuguese, under Albuquerque bolstered their stranglehold by introducing a
permit system for other ships and exercising control over the major ship-building centres in the
region. The non-availability of timber in the Gulf and Red Sea regions for ship-building also
helped the Portuguese in their objectives.

Albuquerque acquired Goa from the Sultan of Bijapur in 1510 with ease; the principal port of
the Sultan of Bijapur became “the first bit of Indian territory to be under the Europeans since
the time of Alexander the Great”. An interesting feature of his rule was the abolition of sati.

QUESTION 6. MTYyNjA0K1NhaSBLaXJhbitpbGJzbmFhNEBnbWFpbC5jb20rUVVFU1RJT04gNQ==
Who among the following introduced tobacco and cashew nut crops in India?

a) The Portuguese
b) The Dutch
c) The English
d) The French
Correct Answer: A
Your Answer: Unanswered
Explanation

Solution (a)

The Portuguese men who had come on the voyages and stayed back in India were, from
Albuquerque’s day, encouraged to take local wives.

In Goa and the Province of the North they established themselves as village landlords, often
building new roads and irrigation works, introducing new crops like tobacco and cashew nut, or
better plantation varieties of coconut besides planting large groves of coconut to meet the need
for coir rigging and cordage.

IASbaba
Web: http://ilp.iasbaba.com/ Score:
Email: ilp@iasbaba.com 0.00 / 200
Page 4
2019 - Test 4-
Exam Title :
History & Cu...
Email : ilbsnaa4@gmail.com
Contact :

In the cities, such as Goa and Cochin, they settled as artisans and master-craftsmen, besides
being traders.

QUESTION 7. MTYyNjA0K1NhaSBLaXJhbitpbGJzbmFhNEBnbWFpbC5jb20rUVVFU1RJT04gNg==
Consider the following statements regarding the British in India.

1. British were granted permission by Jahangir, to establish their first factory at Surat.

2.The golden Farman was given to them by Jahangir to trade free of cost at the Indian ports.

Choose the correct code

a) 1 only
b) 2 only
c) Both 1 and 2
d) None of the above
Correct Answer: A
Your Answer: Unanswered
Explanation

Solution (a)

On December 31, 1600, Queen Elizabeth I issued a charter with rights of exclusive trading to
the company named the ‘Governor and Company of Merchants of London trading into the East
Indies’. Initially, a monopoly of fifteen years was granted, which in May 1609 was extended
indefinitely by a fresh charter. Captain Hawkins arrived in the court of Jahangir in April 1609
itself. But the mission to establish a factory at Surat didn’t succeed due to opposition from the
Portuguese, and Hawkins left Agra in November 1611.

In 1611, the English had started trading at Masulipatnam on the south-eastern coast of India
and later established a factory there in 1616.

It was in 1612 that Captain Thomas Best defeated the Portuguese in the sea off Surat; an
impressed Jahangir granted permission to the English in early 1613 to establish a factory at
Surat under Thomas Aldworth. The English Company’s position was improved by the ‘Golden
Farman’ issued to them by the Sultan of Golconda in 1632. On a payment of 500 pagodas a year,
they earned the privilege of trading freely in the ports of Golconda.

Do you know?

A member of the Masulipatnam council, the British merchant Francis Day, in 1639 received
from the ruler of Chandragiri permission to build a fortified factory at Madras which later
became the Fort St. George and replaced Masulipatnam as the headquarters of the English
settlements in south India.

QUESTION 8. MTYyNjA0K1NhaSBLaXJhbitpbGJzbmFhNEBnbWFpbC5jb20rUVVFU1RJT04gNw==
Which of the following are the main terms of the Magna Carta of English company?

1. The Company was permitted to issue dastaks (passes) for the transportation of goods.

IASbaba
Web: http://ilp.iasbaba.com/ Score:
Email: ilp@iasbaba.com 0.00 / 200
Page 5
2019 - Test 4-
Exam Title :
History & Cu...
Email : ilbsnaa4@gmail.com
Contact :

2. The Company was permitted to rent more lands around Calcutta.

3. The coins of the Company minted at Bombay were to have currency throughout the Mughal
Empire.

Choose the correct code

a) 1 and 2 only
b) 2 and 3 only
c) 1 and 3 only
d) All of the above
Correct Answer: D
Your Answer: Unanswered
Explanation

Solution (d)

In 1715, an English mission led by John Surman to the court of the Mughal emperor
Farrukhsiyar secured three famous Farmans, giving the Company many valuable privileges in
Bengal, Gujarat and Hyderabad. The Farmans thus obtained were regarded the Magna Carta of
the Company. Their important terms were—

1) In Bengal, the Company’s imports and exports were exempted from additional customs
duties excepting the annual payment of 3,000 rupees as settled earlier.

2) The Company was permitted to issue dastaks (passes) for the transportation of such
goods.

3) The Company was permitted to rent more lands around Calcutta.

4) In Hyderabad, the Company retained its existing privilege of freedom from duties in
trade and had to pay the prevailing rent only for Madras.

5) In Surat, for an annual payment of 10,000 rupees, the East India Company was
exempted from the levy of all duties.

6) It was decreed that the coins of the Company minted at Bombay were to have currency
throughout the Mughal empire.

QUESTION 9. MTYyNjA0K1NhaSBLaXJhbitpbGJzbmFhNEBnbWFpbC5jb20rUVVFU1RJT04gOA==
Consider the following statements regarding the Carnatic wars

1. All Carnatic war were the extension of Anglo-French rivalry in Europe caused by the Austrian
war of succession

2. Treaty of Aix -La Chapelle is related to second Carnatic war.

3. Battle of Wandiwash is related to third Carnatic war.

Choose the correct code

a) 1 and 2 only

IASbaba
Web: http://ilp.iasbaba.com/ Score:
Email: ilp@iasbaba.com 0.00 / 200
Page 6
2019 - Test 4-
Exam Title :
History & Cu...
Email : ilbsnaa4@gmail.com
Contact :

b) 2 and 3 only
c) 3 only
d) 1 and 3 only
Correct Answer: C
Your Answer: Unanswered
Explanation

Solution (c)

Carnatic was the name given by the Europeans to the Coromandel coast and its hinterland. The
First Carnatic War(1740-1748) was an extension of the Anglo-French War in Europe which was
caused by the Austrian War of Succession.

The First Carnatic War ended in 1748 when the Treaty of Aix-La Chapelle was signed bringing
the Austrian War of Succession to a conclusion. Under the terms of this treaty, Madras was
handed back to the English, and the French, in turn, got their territories in North America.

Second Carnatic War (1749-1754)

The background for the Second Carnatic War was provided by rivalry in India. Dupleix, the
French governor who had successfully led the French forces in the First Carnatic War, sought to
increase his power and French political influence in southern India by interfering in local
dynastic disputes to defeat the English.

The French authorities, annoyed at the heavy financial losses that Dupleix’s policy involved,
decided to recall him in 1754. Godeheu succeeded Dupleix as the French Governor-General in
India. Godeheu adopted a policy of negotiations with the English and concluded a treaty with
them. The English and the French agreed not to interfere in the quarrels of native princes. Also,
each party was left in possession of the territories actually occupied by them at the time of the
treaty.

Third Carnatic war (1758-1763)

In Europe, when Austria wanted to recover Silesia in 1756, the Seven Years War (1756-63)
started. Britain and France were once again on opposite sides. In 1758, the French army under
Count de Lally captured the English forts of St. David and Vizianagaram in 1758. Now, the
English became offensive and inflicted heavy losses on the French fleet under Admiral D’Ache
at Masulipatnam.

Battle of Wandiwash The decisive battle of the Third Carnatic War was won by the English on
January 22, 1760 at Wandiwash (or Vandavasi) in Tamil Nadu. General Eyre Coote of the
English totally routed the French army under Count Thomas Arthur de Lally and took Bussy as
prisoner.

QUESTION 10. MTYyNjA0K1NhaSBLaXJhbitpbGJzbmFhNEBnbWFpbC5jb20rUVVFU1RJT04gOQ==


Which of the following can be cited as the reasons for English dominance over French in India?

1. The English company being a private enterprise could take instant decisions than the French
company which was controlled and regulated by the French government

2. The English navy was superior to French navy

IASbaba
Web: http://ilp.iasbaba.com/ Score:
Email: ilp@iasbaba.com 0.00 / 200
Page 7
2019 - Test 4-
Exam Title :
History & Cu...
Email : ilbsnaa4@gmail.com
Contact :

3. The English subordinated their commercial interest to territorial ambition, which made the
English company short of funds.

Choose the correct code

a) 1 and 2 only
b) 2 and 3 only
c) 1 and 3 only
d) All of the above
Correct Answer: A
Your Answer: Unanswered
Explanation

Solution (a)

The English company was a private enterprise—this created a sense of enthusiasm and self-
confidence among the people. With less governmental control over it, this company could take
instant decisions when needed without waiting for the approval of the government. The French
company, on the other hand, was a State concern. It was controlled and regulated by the French
government and was hemmed in by government policies and delays in decision-making.

The English navy was superior to the French navy; it helped to cut off the vital sea link between
the French possessions in India and France.

The English held three important places, namely, Calcutta, Bombay and Madras whereas the
French had only Pondicherry.

The French subordinated their commercial interest to territorial ambition, which made the
French company short of funds.

In spite of their imperialistic motives, the British never neglected their commercial interests. So
they always had the funds and the consequent sound financial condition to help them
significantly in the wars against their rivals.

A major factor in the success of the English in India was the superiority of the commanders in
the British camp. In comparison to the long list of leaders on the English side —Sir Eyre Coote,
Major Stringer Lawrence, Robert Clive and many others—there was only Dupleix on the French
side.

QUESTION 11. MTYyNjA0K1NhaSBLaXJhbitpbGJzbmFhNEBnbWFpbC5jb20rUVVFU1RJT04gMTA=


Arrange the following their correct chronological order

1. Vasco Da Gama discovered sea route to India.

2. Formation of English East India Company.

3. Magna Carta of the East India Company.

4. Treaty of Paris

Choose the correct code

IASbaba
Web: http://ilp.iasbaba.com/ Score:
Email: ilp@iasbaba.com 0.00 / 200
Page 8
2019 - Test 4-
Exam Title :
History & Cu...
Email : ilbsnaa4@gmail.com
Contact :

a) 1-2-3-4
b) 1-2-4-3
c) 2-1-3-4
d) 2-1-4-3
Correct Answer: A
Your Answer: Unanswered
Explanation

Solution (a)

Vasco Da Gama discovered sea route to India in 1498.

Vasco’s second visit in 1502 led to the establishment of trading stations at Calicut, Cochin and
Cannanore.

English East India Company was formed on December 31, 1600 by the charter issued by Queen
Elizabeth I, which gave the company monopoly to trade in the East Indies for 15 years.

In 1717, the Mughal Emperor Farrukhsiyar’s Farmans, called Magna Carta of the East India
Company, gave significant privileges to the Company in Bengal, Gujarat and Hyderabad.

By the Treaty of Paris (1763), the French were allowed to use Indian settlements for
commercial purposes only and fortifications of settlements were banned.

QUESTION 12. MTYyNjA0K1NhaSBLaXJhbitpbGJzbmFhNEBnbWFpbC5jb20rUVVFU1RJT04gMTE=


Match the following

Events Description

1. Battle of Bidara A)Victory of British forces over the French forces

2. Battle of Wandiwash B)English defeated Dutch

3. Fort St. George C)Madras was handed back to British

4. Treaty of Aix-La Chapelle D) Replaced Masulipatnam as the

English headquarters on the east coast

Choose the correct code

a) A-1 B-2 C-3 D-4


b) A-2 B-1 C-4 D-3
c) A-3 B-2 C-4 D-1
d) A-4 B-2 C-3 D-1
Correct Answer: B
Your Answer: Unanswered
Explanation

Solution (b)

IASbaba
Web: http://ilp.iasbaba.com/ Score:
Email: ilp@iasbaba.com 0.00 / 200
Page 9
2019 - Test 4-
Exam Title :
History & Cu...
Email : ilbsnaa4@gmail.com
Contact :

Self-explanatory

QUESTION 13. MTYyNjA0K1NhaSBLaXJhbitpbGJzbmFhNEBnbWFpbC5jb20rUVVFU1RJT04gMTI=


What were the appropriate reasons for the increasingly large number of wars fought at
Panipat?

1. Panipat’s surrounding region has a flat ground which was suitable for cavalry movement
which was main mode of warfare at that time.

2. The duration of monsoon rainfall in the region is short in comparison to other areas making it
easier to fight.

3. Its proximity to Delhi made it easier for the Indian rulers to transport weapons, military and
food supplies.

Choose the correct code

a) 1 and 2 only
b) 2 and 3 only
c) 1 and 3 only
d) All of the above
Correct Answer: D
Your Answer: Unanswered
Explanation

Solution (d)

Why Panipat was a favourite battle field?

Panipat had a strategic location. One of the parties of the war generally came from the north/
northwest through the Khyber Pass to get hold over Delhi, the political capital of northern
India.

To move a military through rough terrains—deserts of Rajasthan or the other northern areas
infested with dense forests—was very risky and difficult. On the other hand, the rulers at Delhi
considered Panipat as a confrontable strategic ground and hence they preferred to take the
fight there.

1) Its proximity to Delhi made it easier for the Indian rulers to transport weapons,
military and food supplies etc., to the battleground, and still keep the capital insulated from the
conflict at hand.

2) Panipat’s surrounding region has a flat ground which was suitable for cavalry
movement—the main mode of warfare at the time.

3) After the construction of the Grand Trunk Road by Sher Shah Suri (1540-45), Panipat
was on this route. It became easier for conquerors to find their way there.

4) The duration of monsoon rainfall in the region is short in comparison to other areas
making it easier to fight.

IASbaba
Web: http://ilp.iasbaba.com/ Score:
Email: ilp@iasbaba.com 0.00 / 200
Page 10
2019 - Test 4-
Exam Title :
History & Cu...
Email : ilbsnaa4@gmail.com
Contact :

5) The artisans/smiths of these regions were experts in making warfare-related materials


and hence it became easier for forces of both parties to replenish their war materials.

QUESTION 14. MTYyNjA0K1NhaSBLaXJhbitpbGJzbmFhNEBnbWFpbC5jb20rUVVFU1RJT04gMTM=


Consider the following statements about the taxation system during the Marathas.

1. Chauth was annual tax nominally levied at 25% on revenue or produce on the lands which
were under Mughal rule.

2. Sardeshmukhi was an additional 10% levy on top of the Chauth which was tribute paid to the
king.

Choose the correct code

a) 1 only
b) 2 only
c) Both 1 and 2
d) None of the above
Correct Answer: C
Your Answer: Unanswered
Explanation

Solution (c)

Chauth was a regular tax or tribute imposed, from early 18th century, by the Maratha Empire in
India. It was an annual tax nominally levied at 25% on revenue or produce, hence the name. It
was levied on the lands which were under Mughal rule. The Sardeshmukhi was an additional
10% levy on top of the Chauth. It is a tribute paid to the king.

Opinions on the function of the Chauth vary. According to M G Ranade, the chauth was charged
to provide armed security for a state by the Marathas and is thus comparable to the system of
subsidiary alliances that was used by Lord Wellesley to bring Indian states under British
control.

Do you know?

Shivaji first demanded chauth in 1665 and the Deccan sultanates of Bijapur and Golconda
began to pay him a combined sum of 8, 00,000 Rs after he was made a raja by Aurangzeb in
1668.

In 1719, the Mughal emperor granted Shahu the chauth and sardeshmukhi rights over the six
Deccan provinces in exchange for his maintaining a contingent of 15,000 troops for the
emperor. The revenues from chauth were in turn divided into four parts that went to various
functionaries of the Maratha Empire.

QUESTION 15. MTYyNjA0K1NhaSBLaXJhbitpbGJzbmFhNEBnbWFpbC5jb20rUVVFU1RJT04gMTQ=


Consider the following statements about the states that emerged as a result of the decline of
the Mughal empire.

IASbaba
Web: http://ilp.iasbaba.com/ Score:
Email: ilp@iasbaba.com 0.00 / 200
Page 11
2019 - Test 4-
Exam Title :
History & Cu...
Email : ilbsnaa4@gmail.com
Contact :

1. Successor states were those Mughal provinces which turned into states after breaking away
from the empire.

2. New states are those which came into existence primarily due to the destabilisation of the
Mughal control over the provinces.

3. Independent states were the states which were set up by the rebels against the Mughal
Empire.

Choose the correct code

a) 1 and 2 only
b) 2 and 3 only
c) 1 and 3 only
d) 1 only
Correct Answer: D
Your Answer: Unanswered
Explanation

Solution (d)

The states that emerged as a result of the decline of the Mughal Empire can be classified into
the following three broad categories:

Successor States: These were the Mughal provinces that turned into states after breaking away
from the empire. Though they did not challenge the sovereignty of the Mughal ruler, the
establishment of virtually independent and hereditary authority by their governors showed the
emergence of autonomous polity in these territories. Some examples are Awadh, Bengal and
Hyderabad.

Independent Kingdoms: These states came into existence primarily due to the destabilisation of
the Mughal control over the provinces, examples being Mysore, Kerala and the Rajput states.

The New States: These were the states set up by the rebels against the Mughal empire,
examples being the Maratha, the Sikh and the Jat states.

QUESTION 16. MTYyNjA0K1NhaSBLaXJhbitpbGJzbmFhNEBnbWFpbC5jb20rUVVFU1RJT04gMTU=


Match the following states with their founders

State Founder

1. Hyderabad A)Martanda Varma

2. Kerala B)Murshid Kuli Khan

3. Bengal C)Burhan-Ul-Mulk

4. Awadh D)Nizam-Ul-Mulk

Choose the correct code

a) A-2 B-3 C-4 D-1

IASbaba
Web: http://ilp.iasbaba.com/ Score:
Email: ilp@iasbaba.com 0.00 / 200
Page 12
2019 - Test 4-
Exam Title :
History & Cu...
Email : ilbsnaa4@gmail.com
Contact :

b) A-3 B-1 C-4 D-2


c) A-2 B-1 C-4 D-3
d) A-2 B-1 C-3 D-4
Correct Answer: A
Your Answer: Unanswered
Explanation

Solution (a)

The founder of the Asaf-Jah house of Hyderabad was Kilich Khan, popularly known as Nizam-Ul-
Mulk.

The founder of the independent principality of Awadh was Saadat Khan, popularly known as
Burhan-ul-Mulk. Saadat Khan was a Shia. He had joined in a conspiracy against the Sayyid
brothers, which resulted in his being given an increased mansab. Later, driven out of the court,
he was prompted to found a new independent state.

Murshid Kuli Khan was the founder of the independent state of Bengal. He was a capable ruler
and made Bengal a prosperous state. He was succeeded in 1727 by his son Shujaud-din. His
successor, Sarfaraz Khan, was killed in 1740 by Alivardi Khan, the deputy governor of Bihar at
Gheria, who assumed power and made himself independent of the Mughal emperor by giving
yearly tribute.

Martanda Varma established an independent state of Kerala with Travancore as his capital. He
extended the boundaries of his state from Kanyakumari to Cochin. He made efforts to organise
his army along the Western model and adopted various measures to develop his state.

QUESTION 17. MTYyNjA0K1NhaSBLaXJhbitpbGJzbmFhNEBnbWFpbC5jb20rUVVFU1RJT04gMTY=


Which of the following were found to be the appropriate reasons for the downfall of Mughal
Empire?

1. Aurangzeb’s Religious and Deccan Policies antagonised various rulers.

2. Endless wars, stagnation in agriculture, and decline in trade and industry emptied the royal
treasury.

3. The vast empire became a difficult task for weak rulers to administer efficiently.

Choose the correct code

a) 1 and 2 only
b) 2 and 3 only
c) 1 and 3only
d) All of the above
Correct Answer: D
Your Answer: Unanswered
Explanation

Solution (d)

Reasons for the decline of Mughal Empire

IASbaba
Web: http://ilp.iasbaba.com/ Score:
Email: ilp@iasbaba.com 0.00 / 200
Page 13
2019 - Test 4-
Exam Title :
History & Cu...
Email : ilbsnaa4@gmail.com
Contact :

Weak Successors- The Mughal Empire was a personal despotism and its success depended upon
a strong and capable monarch.

Absence of Definite Law of Succession- Continuous wars of succession (absence of law of


primogeniture) fostered partisanship at the cost of patriotism.

Aurangzeb’s Religious and Deccan Policies- The religious policy antagonised the Rajputs, Sikhs,
Jats and Marathas; Deccan policy kept the emperor away from the capital for a long duration.

Degeneration of Rulers and Nobles

Deterioration of Army

Too Vast an Empire- The vast empire became a difficult task for weak rulers to administer
efficiently.

External Invasions- Invasions of Irani and Durrani kingdoms (Nadir Shah, Ahmad Shah Abdali)
gave a death-blow.

Economic Decline- Endless wars, stagnation in agriculture, and decline in trade and industry
emptied the royal treasury.

Advent of Europeans- European companies interfered in native politics, hastening the


disintegration of empire.

Shifting Allegiance of Zamindars.

Jagirdari Crisis.

Rise of Regional Aspirations- Rise and establishment of Awadh, Bengal, Hyderabad, Mysore,
Kerala, Rajput states and Jat states accelerated the process of disintegration.

QUESTION 18. MTYyNjA0K1NhaSBLaXJhbitpbGJzbmFhNEBnbWFpbC5jb20rUVVFU1RJT04gMTc=


Who among the following is related to the famous black hole tragedy in the history of modern
India?

a) Siraj-ud-daula
b) Shuja-ud-daula
c) Mir jafar
d) Robert Clive
Correct Answer: A
Your Answer: Unanswered
Explanation

Solution (a)

‘Black Hole Tragedy’- Siraj-ud-daula is believed to have imprisoned 146 English persons who
were lodged in a very tiny room due to which 123 of them died of suffocation.

However, historians either do not believe this story, or say that the number of victims must have
been much smaller.

IASbaba
Web: http://ilp.iasbaba.com/ Score:
Email: ilp@iasbaba.com 0.00 / 200
Page 14
2019 - Test 4-
Exam Title :
History & Cu...
Email : ilbsnaa4@gmail.com
Contact :

QUESTION 19. MTYyNjA0K1NhaSBLaXJhbitpbGJzbmFhNEBnbWFpbC5jb20rUVVFU1RJT04gMTg=


Which of the following statements are correct regarding the battle of Plassey?

1.The battle established the supremacy of the English in whole of India.

2. Mir Jafar became the Nawab of Bengal after the battle and he was completely free from the
British control.

3. The battle changed the form of government followed till then.

Choose the correct code

a) 1 and 2 only
b) 1 only
c) 1 and 3 only
d) None of the above
Correct Answer: D
Your Answer: Unanswered
Explanation

Solution (d)

The arrival of a strong force under the command of Robert Clive at Calcutta from Madras
strengthened the English position in Bengal. Clive forged a secret alliance with the traitors of
the nawab—Mir Jafar, Rai Durlabh, Jagat Seth (an influential banker of Bengal) and Omichand.

Under the deal, Mir Jafar was to be made the nawab who in turn would reward the Company for
its services. The Battle of Plassey placed at the disposal of the English vast resources of Bengal.

After Plassey, the English virtually monopolised the trade and commerce of Bengal. As a result
of this victory, Mir Jafar became the Nawab of Bengal. He gave large sums of money plus the
zamindari of 24 parganas to the English.

The Battle of Plassey had political significance for it laid the foundation of the British empire in
India; it has been rightly regarded as the starting point of British rule in India.

The battle established the military supremacy of the English in Bengal. Their main rivals, the
French, were ousted. They obtained a grant of territories for the maintenance of a properly
equipped military force, and their prestige increased manifold. But there was no apparent
change in the form of government, though the supreme control of affairs passed to Clive, on
whose support the new nawab, Mir Jafar, was entirely dependent for maintaining his newly
acquired position.

The sovereignty of the English over Calcutta was recognised, and the English posted a Resident
at the nawab’s court.

QUESTION 20. MTYyNjA0K1NhaSBLaXJhbitpbGJzbmFhNEBnbWFpbC5jb20rUVVFU1RJT04gMTk=


Consider the following statements about dual system of government.

1. It was introduced by Robert Clive immediately after the battle of Plassey.

IASbaba
Web: http://ilp.iasbaba.com/ Score:
Email: ilp@iasbaba.com 0.00 / 200
Page 15
2019 - Test 4-
Exam Title :
History & Cu...
Email : ilbsnaa4@gmail.com
Contact :

2. The company exercise diwani rights as the diwan whereas the nizamat function was
exercised by the nawab.

Choose the correct code

a) 1 only
b) 2 only
c) Both 1 and 2
d) None of the above
Correct Answer: D
Your Answer: Unanswered
Explanation

Solution (d)

After the battle of Buxar, the East India Company became the real masters of Bengal. Robert
Clive introduced the dual system of government, i.e., the rule of the two—the Company and the
Nawab—in Bengal in which both the diwani, i.e., collecting revenues, and nizamat, i.e., police
and judicial functions, came under the control of the Company.

The Company exercised diwani rights as the diwan and the nizamat rights through its right to
nominate the deputy subahdar. The Company acquired the diwani functions from the emperor
and nizamat functions from the subahdar of Bengal.

Do you know?

The system held a great advantage for the Company. It left the appearance of authority to the
puppet Indian ruler while keeping the sovereign power in the hands of the Company. The nawab
was responsible for maintaining peace and order, but he depended both for funds and forces
upon the Company because the latter controlled the army and revenues.

QUESTION 21. MTYyNjA0K1NhaSBLaXJhbitpbGJzbmFhNEBnbWFpbC5jb20rUVVFU1RJT04gMjA=


Which of the following are correct about the great leap forward campaign?

a) It is a Chinese campaign initiated in 1958 aimed at industrialising the country on a massive


scale.
b) It is an Indian campaign initiated in 1992 aimed at industrialising the country on a massive
scale.
c) It is a Chinese campaign initiated in 1958 aimed at import substitution of the consumer
goods.
d) It is a Pakistani campaign initiated in 1958 aimed at increasing the food grain production
using green revolution technology.
Correct Answer: A
Your Answer: Unanswered
Explanation

Solution (a)

The Great Leap Forward (GLF) campaign initiated in 1958 aimed at industrialising the country
on a massive scale. People were encouraged to set up industries in their backyards.

IASbaba
Web: http://ilp.iasbaba.com/ Score:
Email: ilp@iasbaba.com 0.00 / 200
Page 16
2019 - Test 4-
Exam Title :
History & Cu...
Email : ilbsnaa4@gmail.com
Contact :

In rural areas, communes were started. Under the Commune system, people collectively
cultivated lands. In 1958, there were 26,000 communes covering almost all the farm population.

GLF campaign met with many problems. A severe drought caused havoc in China killing about
30 million people. When Russia had conflicts with China, it withdrew its professionals who had
earlier been sent to China to help in the industrialisation process.

In 1965, Mao introduced the Great Proletarian Cultural Revolution (1966–76) under which
students and professionals were sent to work and learn from the countryside.

QUESTION 22. MTYyNjA0K1NhaSBLaXJhbitpbGJzbmFhNEBnbWFpbC5jb20rUVVFU1RJT04gMjE=


Which of the following organisation releases Human development report?

a) UNDP
b) World Bank
c) World Economic Forum
d) UNESCO
Correct Answer: A
Your Answer: Unanswered
Explanation

Solution (a)

The Human Development Report (HDR) is an annual milestone published by the Human
Development Report Office of the United Nations Development Programme (UNDP).

As of 2013 the last decade saw convergence in human development indicators (HDI) values
globally, although progress was uneven within and between regions. Developing countries'
transformation into major economies with growing political influence has impacted human
development progress.

The Human Development Index (HDI) is a composite statistic (composite index) of life
expectancy, education, and per capita income indicators, which are used to rank countries into
four tiers of human development. A country scores higher HDI when the lifespan is higher, the
education level is higher, and the GDP per capita is higher. The HDI was developed by Pakistani
economist Mahbub ul Haq and Indian economist Amartya Sen which was further used to
measure the country's development by the United Nations Development Program (UNDP)

Do you know?

The Human Development Index (HDI) was created to emphasize that expanding human choices
should be the ultimate criteria for assessing development results. Economic growth is a mean to
that process, but is not an end by itself. The HDI can also be used to question national policy
choices, asking how two countries with the same level of Gross National Income (GNI) per
capita can end up with different human development outcomes.

For example, Malaysia has GNI per capita higher than Chile, but in Malaysia, life expectancy at
birth is about 7 years shorter and expected years of schooling is 3 years shorter than in Chile,
resulting in Chile having a much higher HDI value than Malaysia. These striking contrasts can
stimulate debate about government policy priorities.

IASbaba
Web: http://ilp.iasbaba.com/ Score:
Email: ilp@iasbaba.com 0.00 / 200
Page 17
2019 - Test 4-
Exam Title :
History & Cu...
Email : ilbsnaa4@gmail.com
Contact :

The 2010 Human Development Report introduced an Inequality-adjusted Human Development


Index (IHDI). While the simple HDI remains useful, it stated that "the IHDI is the actual level of
human development (accounting for inequality)", and "the HDI can be viewed as an index of
'potential' human development (or the maximum IHDI that could be achieved if there were no
inequality)".

QUESTION 23. MTYyNjA0K1NhaSBLaXJhbitpbGJzbmFhNEBnbWFpbC5jb20rUVVFU1RJT04gMjI=


Which of the following best describes the meaning of carrying capacity of environment?

a) The situation where resource extraction is above the rate of regeneration of the resource
b) The situation where resource extraction is below the rate of regeneration of the resource
the wastes generated are within the assimilating capacity of the environment
c) It is the ability of the environment to absorb degradation
d) None of the above
Correct Answer: B
Your Answer: Unanswered
Explanation

Solution (b)

The environment performs four vital functions

(i) it supplies resources: resources here include both renewable and non-renewable resources.
Renewable resources are those which can be used without the possibility of the resource
becoming depleted or exhausted. That is, a continuous supply of the resource remains available.
Examples of renewable resources are the trees in the forests and the fishes in the ocean. Non-
renewable resources, on the other hand, are those which get exhausted with extraction and
use, for example, fossil fuel

(ii) it assimilates waste

(iii) it sustains life by providing genetic and bio diversity and

(iv) it also provides aesthetic services like scenery etc.

The environment is able to perform these functions without any interruption as long as the
demand on these functions is within its carrying capacity. This implies that the resource
extraction is not above the rate of regeneration of the resource and the wastes generated are
within the assimilating capacity of the environment. When this is not so, the environment fails
to perform its third and vital function of life sustenance and this results in an environmental
crisis.

Many resources have become extinct and the wastes generated are beyond the absorptive
capacity of the environment. Absorptive capacity means the ability of the environment to absorb
degradation.

QUESTION 24. MTYyNjA0K1NhaSBLaXJhbitpbGJzbmFhNEBnbWFpbC5jb20rUVVFU1RJT04gMjM=

IASbaba
Web: http://ilp.iasbaba.com/ Score:
Email: ilp@iasbaba.com 0.00 / 200
Page 18
2019 - Test 4-
Exam Title :
History & Cu...
Email : ilbsnaa4@gmail.com
Contact :

Arrange the following in the increasing order of contribution to India’s total electricity
generation in 2018.

1. Thermal power

2. Hydel power

3. Nuclear power

4. Renewable energy sources

Choose the correct code

a) 3<4<1<2
b) 3<4<2<1
c) 3<2<4<1
d) 3<2<3<4
Correct Answer: C
Your Answer: Unanswered
Explanation

Solution (c)

Sector Contribution

Thermal power 64.8%

Hydel power 13.2%

Nuclear Power 2%

Renewable Energy resources 20.1%

QUESTION 25. MTYyNjA0K1NhaSBLaXJhbitpbGJzbmFhNEBnbWFpbC5jb20rUVVFU1RJT04gMjQ=


Consider the following statements regarding employment status in India.

1. Worker-population ratio gives the proportion of population that is actively contributing to the
production of goods and services of a country.

2. Higher the ratio, lower the engagement of people in the economic activities.

Choose the correct code

a) 1 only
b) 2 only
c) Both 1 and 2

IASbaba
Web: http://ilp.iasbaba.com/ Score:
Email: ilp@iasbaba.com 0.00 / 200
Page 19
2019 - Test 4-
Exam Title :
History & Cu...
Email : ilbsnaa4@gmail.com
Contact :

d) None of the above


Correct Answer: A
Your Answer: Unanswered
Explanation

Solution (a)

Worker-population ratio is an indicator which is used for analysing the employment situation in
the country.

This ratio is useful in knowing the proportion of population that is actively contributing to the
production of goods and services of a country.

If the ratio is higher, it means that the engagement of people is greater; if the ratio for a
country is medium, or low, it means that a very high proportion of its population is not involved
directly in economic activities.

QUESTION 26. MTYyNjA0K1NhaSBLaXJhbitpbGJzbmFhNEBnbWFpbC5jb20rUVVFU1RJT04gMjU=


Which of the following are not the members of BIMSTEC?

1. Laos

2. Myanmar

3. Maldives

4. China

5. Thailand

Choose the correct code

a) 1,2 and 3 only


b) 1,3 and 4 only
c) 1 and 4 only
d) 1 and 3 only
Correct Answer: B
Your Answer: Unanswered
Explanation

Solution (b)

The Bay of Bengal Initiative for Multi-Sectoral Technical and Economic Cooperation (BIMSTEC)
is an international organisation of seven nations of South Asia and South East Asia, housing 1.5
billion people and having a combined gross domestic product of $2.5 trillion (2014).

The BIMSTEC member states—Bangladesh, India, Myanmar, Sri Lanka, Thailand, Bhutan, and
Nepal—are among the countries dependent on the Bay of Bengal.

Leadership is rotated in alphabetical order of country names. The permanent secretariat is in


Dhaka.

IASbaba
Web: http://ilp.iasbaba.com/ Score:
Email: ilp@iasbaba.com 0.00 / 200
Page 20
2019 - Test 4-
Exam Title :
History & Cu...
Email : ilbsnaa4@gmail.com
Contact :

Do you know?

• The 4th BIMSTEC summit is scheduled to be held at Nepal in 2018.

QUESTION 27. MTYyNjA0K1NhaSBLaXJhbitpbGJzbmFhNEBnbWFpbC5jb20rUVVFU1RJT04gMjY=


Which of the following is/are included in foreign exchange reserves?

1. Bank notes

2. Government Securities

3. Bonds

4. Treasury bills

Choose the correct code

a) 1,2 and 4 only


b) 1 only
c) 1,2 and 3 only
d) All of the above
Correct Answer: D
Your Answer: Unanswered
Explanation

Solution (d)

Foreign exchange reserves are used to back liabilities and influence monetary policy. This refers
to any foreign money held by a central bank, such as the Reserve Bank of India. These reserves
can include banknotes, deposits, bonds, treasury bills and other governmental securities. These
assets serve many purposes but are most significantly held to ensure that a central government
agency has backup funds if their national currency rapidly devalues or becomes all together
insolvent.

It is a common practice in countries around the world for their central bank to hold a significant
amount of reserves in their foreign exchange. Most of these reserves are held in the U.S. dollar,
since it is the most traded currency in the world.

The foreign exchange reserves can also be held in the British Pound (GBP), the Euro (EUR), the
Chinese yuan (CNY) or the Japanese yen (JPY) as well.

QUESTION 28. MTYyNjA0K1NhaSBLaXJhbitpbGJzbmFhNEBnbWFpbC5jb20rUVVFU1RJT04gMjc=


Which of the following are the reports given by International Monetary Fund?

1. Fiscal Monitor

2. World economic outlook

3. Global Financial Stability report

IASbaba
Web: http://ilp.iasbaba.com/ Score:
Email: ilp@iasbaba.com 0.00 / 200
Page 21
2019 - Test 4-
Exam Title :
History & Cu...
Email : ilbsnaa4@gmail.com
Contact :

4. World Development report

Choose the correct code

a) 1,2 and 4 only


b) 1 and 2 only
c) 1 and 3 only
d) All except 4
Correct Answer: D
Your Answer: Unanswered
Explanation

Solution (d)

World development report is published annually by World bank group.

The theme of 2018 report is “LEARNING to Realize Education’s Promise”

QUESTION 29. MTYyNjA0K1NhaSBLaXJhbitpbGJzbmFhNEBnbWFpbC5jb20rUVVFU1RJT04gMjg=


After the 12th five year plan, the government has introduced three year action agenda for the
development of Indian economy. What is the time period of this agenda?

a) 2017-2020
b) 2018-2021
c) 2019-2022
d) 2020-2023
Correct Answer: A
Your Answer: Unanswered
Explanation

Solution (a)

On 1st January 2015, the National Institution for Transforming India or NITI Aayog came into
existence as the government’s premier think tank. The Prime Minister’s Office advised the NITI
Aayog to prepare Fifteen Year Vision, Seven Year Strategy and Three Year Action Agenda
documents. Accordingly, the present document is being published to recommend policy changes
and programmes for action from 2017-18 to 2019-20, the last three years of the Fourteenth
Finance Commission.

The Vision, Strategy and Action Agenda exercise represents a departure from the Five Year
Plan process, followed with a handful of discontinuities until the fiscal year 2016-17. The 12th
Five Year Plan was the last of these plans. It has been felt that with an increasingly open and
liberalized economy, we needed to rethink the tools and approaches to conceptualizing the
development process.

The proposed shift represents an important step in this direction. The Action Agenda has been
prepared as an integral part of the exercise leading to the Vision and Strategy document. It has
been fast tracked and released first, keeping in view that with the start of fiscal year 2016-17, it
is of immediate relevance for policy implementation.

IASbaba
Web: http://ilp.iasbaba.com/ Score:
Email: ilp@iasbaba.com 0.00 / 200
Page 22
2019 - Test 4-
Exam Title :
History & Cu...
Email : ilbsnaa4@gmail.com
Contact :

The Three Year Action Agenda offers ambitious proposals for policy changes within a relatively
short period. It is understood that while some may be fully implemented during the three-year
period, implementation of others would continue into the subsequent years.

QUESTION 30. MTYyNjA0K1NhaSBLaXJhbitpbGJzbmFhNEBnbWFpbC5jb20rUVVFU1RJT04gMjk=


What causes the planets to rotate within their respective orbits?

a) Their great size and spherical shape


b) Gravitation alone
c) Gravitation and inertia
d) The rotation of sun on its own axis
Correct Answer: C
Your Answer: Unanswered
Explanation

Solution (c)

The Solar System was formed from a rotating cloud of gas and dust which spun around a newly
forming star, our Sun, at its center. The planets all formed from this spinning disk-shaped cloud,
and continued this rotating course around the Sun after they were formed. The gravity of the
Sun keeps the planets in their orbits. They stay in their orbits because there is no other force in
the Solar System which can stop them.

Gravity is the primary force that controls the orbit of the planets around the sun. While each
planet has its own gravity based on the size of the planet and the speed at which it travels, orbit
is based on the gravity of the sun. The sun's gravity is just strong enough to keep the planets
pulled toward it to create an orbit pattern but not strong enough to pull the planets into the
sun. This is similar to the effect of the Earth on the orbit of the moon and satellites. The lesser
gravity of the planets also helps to keep the planets from falling toward the sun.

The physical law that states that objects in motion have a tendency to remain in motion also
plays a role in keeping the planets in orbit. The gravity of the sun and the planets works
together with the inertia to create the orbits and keep them consistent. The gravity pulls the
sun and the planets together, while keeping them apart. The inertia provides the tendency to
maintain speed and keep moving. The planets want to keep moving in a straight line because of
the physics of inertia. However, the gravitational pull wants to change the motion to pull the
planets into the core of the sun. Together, this creates a rounded orbit as a form of compromise
between the two forces.

QUESTION 31. MTYyNjA0K1NhaSBLaXJhbitpbGJzbmFhNEBnbWFpbC5jb20rUVVFU1RJT04gMzA=


Consider the following statements about meteors and meteorites

1. Meteor is an interplanetary body which enters the earth's atmosphere and explodes in mid-
air as a ball of fire.

2. Meteorite is also an interplanetary body which travels through the earth’s atmosphere lands
on the surface of the earth without exploding in mid-air.

IASbaba
Web: http://ilp.iasbaba.com/ Score:
Email: ilp@iasbaba.com 0.00 / 200
Page 23
2019 - Test 4-
Exam Title :
History & Cu...
Email : ilbsnaa4@gmail.com
Contact :

Choose the correct code

a) 1 Only
b) 2 only
c) Both 1 and 2
d) None of the above
Correct Answer: C
Your Answer: Unanswered
Explanation

Solution (c)

A meteor is the flash of light that we see in the night sky when a small chunk of interplanetary
debris burns up as it passes through our atmosphere. "Meteor" refers to the flash of light
caused by the debris, not the debris itself.

The debris is called a meteoroid. A meteoroid is a piece of interplanetary matter that is smaller
than a kilometre and frequently only millimetres in size. Most meteoroids that enter the Earth's
atmosphere are so small that they vaporize completely and never reach the planet's surface.

If any part of a meteoroid survives the fall through the atmosphere and lands on Earth, it is
called a meteorite. Although the vast majority of meteorites are very small, their size can range
from about a fraction of a gram (the size of a pebble) to 100 kilograms (220 lbs) or more (the
size of a huge, life-destroying boulder).

Asteroids are generally larger chunks of rock that come from the asteroid belt located between
the orbits of Mars and Jupiter.

Comets are asteroid-like objects covered with ice, methane, ammonia, and other compounds
that develop a fuzzy, cloud-like shell called a coma and sometimes a visible tail whenever they
orbit close to the Sun.

QUESTION 32. MTYyNjA0K1NhaSBLaXJhbitpbGJzbmFhNEBnbWFpbC5jb20rUVVFU1RJT04gMzE=


The twelve constellations in the sky which are referred to as Zodiac are

a) Group of stars
b) Signs of roman gods
c) Imaginary region that encompass the path of the planets
d) Group of exo-planets
Correct Answer: A
Your Answer: Unanswered
Explanation

Solution (a)

In western astrology, there are 12 signs but standing for 12 periods of a year. According to
natural distribution, stars are divided into many regions of different sizes, each called a
constellation. Connecting all bright stars in a constellation with lines, different images in the
shape of animals and objects are formed. People named each constellation according to its

IASbaba
Web: http://ilp.iasbaba.com/ Score:
Email: ilp@iasbaba.com 0.00 / 200
Page 24
2019 - Test 4-
Exam Title :
History & Cu...
Email : ilbsnaa4@gmail.com
Contact :
shape. The International Astronomical Union divided the sky into 88 constellations with precise
boundaries, making every star belonging to a particular constellation.

Seen from Earth, the sun moves slowly in the Celestial Sphere and passes through
constellations, forming a large circle for a year. This circle is called Ecliptic. The Ecliptic is
divided into twelve equal portions (each equivalent to 30 degrees); each portion was named
after the closest constellation. All these twelve portions were called Ecliptic Constellations,
according to which western horoscope theories developed.

The astrologists divide a year is into 12 periods, during each period the sun being in a
constellation area. So everyone has a corresponding zodiacal sign according to the period his /
her birthday lies in. The 12 signs are Aries, Taurus, Gemini, Cancer, Leo, Virgo, Libra, Scorpio,
Sagittarius, Capricorn, Aquarius and Pisces. People believe that different signs of the zodiac
present different characteristics and talents.

QUESTION 33. MTYyNjA0K1NhaSBLaXJhbitpbGJzbmFhNEBnbWFpbC5jb20rUVVFU1RJT04gMzI=


The time taken by the sun to orbit once around the centre of its galaxy is called

a) Cosmic year
b) Solar year
c) Parsec
d) Light year
Correct Answer: A
Your Answer: Unanswered
Explanation

Solution (a)

The galactic year, also known as a cosmic year, is the duration of time required for the Sun to
orbit once around the center of the Milky Way Galaxy.

Estimates of the length of one orbit range from 225 to 250 million terrestrial years. The Solar
System is traveling at an average speed of 828,000 km/h (230 km/s) or 514,000 mph (143 mi/s)
within its trajectory around the galactic center, a speed at which an object could
circumnavigate the Earth's equator in 2 minutes and 54 seconds.

A light-year is a unit of distance. It is the distance that light can travel in one year. Light moves
at a velocity of about 300,000 kilometres (km) each second. So in one year, it can travel about
10 trillion km. More precisely, one light-year is equal to 9,500,000,000,000 kilometres.

The period of time required for the earth to make one complete revolution around the sun,
measured from one vernal equinox to the next and equal to 365 days, 5 hours, 48 minutes,
45.51 seconds is known as Solar year. Also known as astronomical year or tropical year.

The parsec is a unit of length used to measure large distances to astronomical objects outside
the Solar System.

QUESTION 34. MTYyNjA0K1NhaSBLaXJhbitpbGJzbmFhNEBnbWFpbC5jb20rUVVFU1RJT04gMzM=

IASbaba
Web: http://ilp.iasbaba.com/ Score:
Email: ilp@iasbaba.com 0.00 / 200
Page 25
2019 - Test 4-
Exam Title :
History & Cu...
Email : ilbsnaa4@gmail.com
Contact :

When heated Magma flow onto Earth's surface through fracture, cracks and volcanoes, it is
known as?

a) Lava
b) Molten magma
c) Core magma
d) Mantle
Correct Answer: A
Your Answer: Unanswered
Explanation

Solution (a)

Magma is composed of molten rock and is stored in the Earth’s crust.

Lava is magma that reaches the surface of our planet through a volcano vent.

QUESTION 35. MTYyNjA0K1NhaSBLaXJhbitpbGJzbmFhNEBnbWFpbC5jb20rUVVFU1RJT04gMzQ=


Which of the following is not an intrusive volcanic landform?

a) Batholith
b) Dyke
c) Lacolith
d) Cinder cone
Correct Answer: D
Your Answer: Unanswered
Explanation

Solution (d)

The lava that is released during volcanic eruptions on cooling develops into igneous rocks. The
cooling may take place either on reaching the surface or also while the lava is still in the crustal
portion.

IASbaba
Web: http://ilp.iasbaba.com/ Score:
Email: ilp@iasbaba.com 0.00 / 200
Page 26
2019 - Test 4-
Exam Title :
History & Cu...
Email : ilbsnaa4@gmail.com
Contact :

Depending on the location of the cooling of the lava, igneous rocks are classified as volcanic
rocks (cooling at the surface) and plutonic rocks (cooling in the crust). The lava that cools
within the crustal portions assumes different forms. These forms are called intrusive forms.

A large body of magmatic material that cools in the deeper depth of the crust develop in the
form of large domes is known as batholith.

Lacoliths are large dome-shaped intrusive bodies with a level base and connected by a pipe-like
conduit from below. It resembles the surface volcanic domes of composite volcano, only these
are located at deeper depths.

When the lava makes its way through cracks and the fissures developed in the land, it solidifies
almost perpendicular to the ground. It gets cooled in the same position to develop a wall-like
structure. Such structures are called dykes.

QUESTION 36. MTYyNjA0K1NhaSBLaXJhbitpbGJzbmFhNEBnbWFpbC5jb20rUVVFU1RJT04gMzU=


Which of the following are the immediate effects of Earthquake?

1. Soil liquefaction

2. Avalanches

3. Structural Collapse

4. Tsunami

5. Floods from dam and levee failures

Choose the correct code

a) 1,2,3 and 5 only


b) 1,2,3 and 4 only

IASbaba
Web: http://ilp.iasbaba.com/ Score:
Email: ilp@iasbaba.com 0.00 / 200
Page 27
2019 - Test 4-
Exam Title :
History & Cu...
Email : ilbsnaa4@gmail.com
Contact :

c) 2,3 4 and 5 only


d) All of the above
Correct Answer: D
Your Answer: Unanswered
Explanation

Solution (d)

Earthquake is a natural hazard. The following are the immediate hazardous effects of

earthquake:

1) Ground Shaking

2) Differential ground settlement

3) Land and mud slides

4) Soil liquefaction

5) Ground lurching

6) Avalanches

7) Ground displacement

8) Floods from dam and levee failures

9) Fires

10) Structural collapse

11) Falling objects

12) Tsunami

The first six listed above have some bearings upon landforms, while others may be considered
the effects causing immediate concern to the life and properties of people in the region.

The effect of tsunami would occur only if the epicentre of the tremor is below oceanic waters
and the magnitude is sufficiently high. Tsunamis are waves generated by the tremors and not an
earthquake in itself. Though the actual quake activity lasts for a few seconds, its effects are
devastating provided the magnitude of the quake is more than 5 on the Richter scale.

QUESTION 37. MTYyNjA0K1NhaSBLaXJhbitpbGJzbmFhNEBnbWFpbC5jb20rUVVFU1RJT04gMzY=


Consider the following statements regarding laws and morals.

1. Laws are concerned with external acts of man and not motives Whereas Morality Concerned
with both the external acts and internal motives.

2. Law is subjective whereas morality is objective

Choose the incorrect code

IASbaba
Web: http://ilp.iasbaba.com/ Score:
Email: ilp@iasbaba.com 0.00 / 200
Page 28
2019 - Test 4-
Exam Title :
History & Cu...
Email : ilbsnaa4@gmail.com
Contact :

a) 1 only
b) 2 only
c) Both 1 and 2
d) None of the above
Correct Answer: B
Your Answer: Unanswered
Explanation

Solution (b)

Some people believe that a constitution merely consists of laws and that laws are one thing,
values and morality, quite another. Therefore, we can have only a legalistic, not a political
philosophy approach to the Constitution.

It is true that all laws do not have a moral content, but many laws are closely connected to our
deeply held values. For example, a law might prohibit discrimination of persons on grounds of
language or religion. Such a law is connected to the idea of equality. Such a law exists because
we value equality. Therefore, there is a connection between laws and moral values.

Some differences between laws and morality

1) Laws are concerned with external acts of man and not motives Whereas Morality Concerned
with both the external acts and internal motives.

2) Law is the concern of the state. Morality is the concern of conscience.

3) Law is concerned with a part of man’s life. Morality is concerned with the whole of man’s
life.

4) Violation of law is punishable by the state whereas violation of Morality is not punishable by
the state.

5) Law is definite and precise. Morality is vague and indefinite.

6) Law is objective. Morality is subjective.

7) Law acts within the territory of the state. Morality is universal.

8) A legal wrong may be morally right. A moral wrong may be legally right.

QUESTION 38. MTYyNjA0K1NhaSBLaXJhbitpbGJzbmFhNEBnbWFpbC5jb20rUVVFU1RJT04gMzc=


Which of the following correctly explains the nature of Indian federalism?

a) Constitutionally symmetric
b) Constitutionally asymmetric
c) Partial symmetric
d) Partially symmetric and partially asymmetric
Correct Answer: B
Your Answer: Unanswered
Explanation

IASbaba
Web: http://ilp.iasbaba.com/ Score:
Email: ilp@iasbaba.com 0.00 / 200
Page 29
2019 - Test 4-
Exam Title :
History & Cu...
Email : ilbsnaa4@gmail.com
Contact :

Solution (b)

The Indian constitution, by introducing the articles concerning Jammu and Kashmir (Art. 370)
and the North-East (Art. 371), anticipates the very important concept of asymmetric federalism.
The Constitution has created a strong central government. But despite this unitary bias of the
Indian Constitution, there are important constitutionally embedded differences between the
legal status and prerogatives of different sub-units within the same federation.

Unlike the constitutional symmetry of American federalism, Indian federalism has been
constitutionally asymmetric. To meet the specific needs and requirements of some sub-units, it
was always part of the original design to have a unique relationship with them or to give them
special status.

For example, the accession of Jammu and Kashmir to the Indian union was based on a
commitment to safeguard its autonomy under Article 370 of the Constitution. This is the only
State that is governed by its own constitution. Similarly, under Article 371A, the privilege of
special status was also accorded to the North-Eastern State of Nagaland.

This Article not only confers validity on pre-existing laws within Nagaland, but also protects
local identity through restrictions on immigration. Many other States too, are beneficiaries of
such special provisions. According to the Indian Constitution, then, there is nothing bad about
this differential treatment.

QUESTION 39. MTYyNjA0K1NhaSBLaXJhbitpbGJzbmFhNEBnbWFpbC5jb20rUVVFU1RJT04gMzg=


The constitution of India has largely remained intact with its basic philosophy since
Independence. Which of the following arguments support this case?

1. Our constitution is robust and its basic framework is very much suited to our country.

2. The Constitution makers were very farsighted and provided for many solutions for future
situations.

Choose the correct code

a) 1 only
b) 2 only
c) Both 1 and 2
d) None of the above
Correct Answer: C
Your Answer: Unanswered
Explanation

Solution (c)

It is not uncommon for nations to rewrite their constitutions in response to changed


circumstances or change of ideas within the society or even due to political upheavals. The
Soviet Union had four constitutions in its life of 74 years (1918, 1924, 1936 and 1977).

In 1991, the rule of the Communist Party of Soviet Union came to an end and soon the Soviet
federation disintegrated. After this political upheaval, the newly formed Russian federation
adopted a new constitution in 1993.

IASbaba
Web: http://ilp.iasbaba.com/ Score:
Email: ilp@iasbaba.com 0.00 / 200
Page 30
2019 - Test 4-
Exam Title :
History & Cu...
Email : ilbsnaa4@gmail.com
Contact :

The Constitution of India was adopted on 26 November 1949. Its implementation formally
started from 26 January 1950. More than sixty-eight years after that, the same constitution
continues to function as the framework within which the government of our country operates.

It is true that we have inherited a very robust Constitution. The basic framework of the
Constitution is very much suited to our country.

It is also true that the Constitution makers were very farsighted and provided for many
solutions for future situations. But no constitution can provide for all eventualities. No
document can be such that it needs no change.

Do you know?

Our Constitution accepts the necessity of modifications according to changing needs of the
society. In the actual working of the Constitution, there has been enough flexibility of
interpretations. Both political practice and judicial rulings have shown maturity and flexibility
in implementing the Constitution. These factors have made our Constitution a living document
rather than a closed and static rulebook.

QUESTION 40. MTYyNjA0K1NhaSBLaXJhbitpbGJzbmFhNEBnbWFpbC5jb20rUVVFU1RJT04gMzk=


Which of the following statements hold true about the nature of Indian constitution?

1. It is a statement of the philosophy of the country and is unalterable.

2. It is placed above the ordinary law of the country.

3. It is a sacred document therefore any talk of changing it is against democracy.

Choose the correct code

a) 1 and 2 only
b) 2 and 3 only
c) 2 only
d) 1 and 3 only
Correct Answer: C
Your Answer: Unanswered
Explanation

Solution (c)

The provisions of the constitution naturally reflect efforts to tackle the problems that the society
is facing at the time of making of the constitution. At the same time, the constitution must be a
document that provides the framework of the government for the future as well. Therefore, the
constitution has to be able to respond to the challenges that may arise in the future.

In this sense, the constitution will always have something that is contemporary and something
that has a more durable importance.

Constitution is not a frozen and unalterable document. It is a document made by human beings
and may need revisions, changes and re-examination.

IASbaba
Web: http://ilp.iasbaba.com/ Score:
Email: ilp@iasbaba.com 0.00 / 200
Page 31
2019 - Test 4-
Exam Title :
History & Cu...
Email : ilbsnaa4@gmail.com
Contact :

It is true that the constitution reflects the dreams and aspirations of the concerned society. It
must also be kept in mind that the constitution is a framework for the democratic governance of
the society.

In this sense, it is an instrument that societies create for themselves. The makers of the Indian
Constitution placed the Constitution above ordinary law and expected that the future
generations will respect this document.

Do you know?

The makers of the Indian Constitution recognised that in the future, this document may require
modifications. Even at the time of writing the Constitution, they were aware that on many
matters there were differences of opinion. Whenever society would veer toward any particular
opinion, a change in the constitutional provisions would be required.

Thus, the Indian Constitution is a combination of both the approaches mentioned above: that
the constitution is a sacred document and that it is an instrument that may require changes
from time to time.

In other words, our Constitution is not a static document, it is not the final word about
everything; it is not unalterable.

QUESTION 41. MTYyNjA0K1NhaSBLaXJhbitpbGJzbmFhNEBnbWFpbC5jb20rUVVFU1RJT04gNDA=


Which of the following were the reasons for failure of Revolt of 1857?

1. Different groups had different ideologies and they were not fighting for the same cause.

2. British had superior arms and resources as compared to Indians.

3. British had faster communication system.

4. During the revolt British were fighting many wars in the neighbouring countries like China,
Burma and Afghanistan so they couldn’t concentrate on India only.

Select the code from below:

a) 1,2 and 3
b) 2,3 and 4
c) 1,3 and 4
d) All of the above
Correct Answer: A
Your Answer: Unanswered
Explanation

Solution (a)

Causes of Failure of the Revolt:

• The resources of the British Government were far superior to those of the rebels.
Luckily, for the British, the Crimean and the Chinese wars had been concluded by 1856 that
helped the British Government to concentrate the entire energy on India.

IASbaba
Web: http://ilp.iasbaba.com/ Score:
Email: ilp@iasbaba.com 0.00 / 200
Page 32
2019 - Test 4-
Exam Title :
History & Cu...
Email : ilbsnaa4@gmail.com
Contact :

• Electric telegram improved the communication channel of the British and information
could be communicated very quickly.

• The people of India could not be inspired by the spirit of nationalism to resist the
foreign soldiers for a long period. No doubt the princes joined the revolution to regain their lost
prestige, the taluqdars jointed it to get back their privileges, and the peasants fought in it for
their economic discontent yet in a positive sense there was no great ideal to unite all Indians in
a common platform.

• The revolt thus could not be prolonged. The rebels could not organize a united military
front against the British army. They fought in separate groups. In moments of need, they could
not combine all their forces which worked as strength to their enemies.

• Lack of able and genius leaders to guide the destiny of the country resulted in a
negative result of the nation-wide revolt. The revolt was spontaneous, the rebels were many in
number but unfortunately the leaders were no great military generals. They fought desperately
with their limited capacities.

• The sudden out -break of the revolt created an uncertain situation in the country for
which the people were not mentally prepared. They were quite ignorant about their role in that
emergency.

QUESTION 42. MTYyNjA0K1NhaSBLaXJhbitpbGJzbmFhNEBnbWFpbC5jb20rUVVFU1RJT04gNDE=


Who of the following did not participate in the revolt of 1857?

1. Sikh Regiment

2. Nizam of Hyderabad

3. Raja of Mysore

4. King of Nepal

5. Western educated Middle class Indians

Select the code from following:

a) 1,2 and 5
b) 1,3,4 and 5
c) 1,2,3 and 5
d) All of the above
Correct Answer: D
Your Answer: Unanswered
Explanation

Solution (d)

The British Government did not allow the fire of revolt to spread in a large part of India.

Punjab and Bombay presidency was untouched. Sikh Regiment played an important role in the
suppression of revolt.

IASbaba
Web: http://ilp.iasbaba.com/ Score:
Email: ilp@iasbaba.com 0.00 / 200
Page 33
2019 - Test 4-
Exam Title :
History & Cu...
Email : ilbsnaa4@gmail.com
Contact :

The Nizam of Hyderabad, the Bengum of Bhopal, the King of Nepal and the Maratha leader
Sindhia extended their helping hands to the British.

The modern educated Indians looked at the revolt as backward looking. They had faith in the
British Government and believed that they can bring about a change in society and modernize
it. Because of this they did not support the revolt.

New Zamindars also supported British in their endeavours as their legitimacy was based on the
British rule.

QUESTION 43. MTYyNjA0K1NhaSBLaXJhbitpbGJzbmFhNEBnbWFpbC5jb20rUVVFU1RJT04gNDI=


Who of the following leaders died during the revolt of 1857?

1. Bahadur Shah Zafar II

2. Rani Lakshmibai

3. Begum Hazrat Mahal

4. Tatya Tope

5. Nana Sahib

Select the code from following:

a) 1,2 and 3
b) 2,4 and 5
c) 2 and 4
d) All of the above
Correct Answer: C
Your Answer: Unanswered
Explanation

Solution (c)

The old Mughal emperor Bahadur Shah II was taken prisoner to Rangoon. The British
Commander-in-Chief Colin Campbell with much difficulties captured Lucknow. The guerrilla
type of war continued in the interior part of Oudh. Begum Hazrat Mahal escaped to Nepal.

On 17th June, 1857 while fighting Maharani Laxmi Bai died, Tantia Tope kept the fighting on for
a pretty long time and at last caught and hanged. Nana Saheb was defeated and fled away to
the dense forests of Nepal to escape death. Kunwar Singh became the victim in a battle field.
Many others either died or fled to the dense forest regions of Nepal. The great revolt subsided.

QUESTION 44. MTYyNjA0K1NhaSBLaXJhbitpbGJzbmFhNEBnbWFpbC5jb20rUVVFU1RJT04gNDM=


Which of the following personalities have called the Revolt of 1857, the First war of Indian
Independence?

IASbaba
Web: http://ilp.iasbaba.com/ Score:
Email: ilp@iasbaba.com 0.00 / 200
Page 34
2019 - Test 4-
Exam Title :
History & Cu...
Email : ilbsnaa4@gmail.com
Contact :

a) Bipan Chandra
b) Outram
c) V D Savarkar
d) Mahatma Gandhi
Correct Answer: C
Your Answer: Unanswered
Explanation

Solution (c)

It was Vinayak Damodar (Veer) Savarkar, who used this title, “Indian war of independence” for
writing a book on the Revolt of 1857. Interestingly, this book was banned under the British rule,
and was firstly published in Netherlands.

QUESTION 45. MTYyNjA0K1NhaSBLaXJhbitpbGJzbmFhNEBnbWFpbC5jb20rUVVFU1RJT04gNDQ=


The Regulating Act 1773 was an Act of the Parliament of Great Britain intended to overhaul the
management of the East India Company's rule in India. Which of the following statements
regarding this Act are correct?

1. It established the Supremacy of Bengal over the other presidencies.

2. It disallowed all Presidencies in India to give orders to commence hostilities or sign peace
treaties.

3.It established a Supreme Court at Fort William, Calcutta.

Select the code from following:

a) 1 and 2
b) 2 and 3
c) 1 and 3
d) All of the above
Correct Answer: C
Your Answer: Unanswered
Explanation

Solution (c)

The Act of 1773 recognized the political functions of the company, because it asserted for the
first time right of the parliament to dictate the form of government. It was the first attempt of
British government to centralize the administrative machinery in India. The act set up a written
constitution for the British possession in India in place of arbitrary rule of the company. A
system was introduced to prevent the Governor-General from becoming autocratic.

This act unequivocally established the supremacy of the Presidency of Bengal over the others.
In matters of foreign policy, the Regulating Act of 1773 made the presidencies of Bombay and
Madras, subordinate to the Governor General and his council. Now, no other presidency could
give orders for commencing hostilities with the Indian Princes, declare a war or negotiate a
treaty. It established a supreme court at Fort William, Calcutta and India’s modern
Constitutional History began.

IASbaba
Web: http://ilp.iasbaba.com/ Score:
Email: ilp@iasbaba.com 0.00 / 200
Page 35
2019 - Test 4-
Exam Title :
History & Cu...
Email : ilbsnaa4@gmail.com
Contact :

QUESTION 46. MTYyNjA0K1NhaSBLaXJhbitpbGJzbmFhNEBnbWFpbC5jb20rUVVFU1RJT04gNDU=


Which of the following British Act introduced an open competition for Indian Civil Service?

a) Charter Act 1813


b) Charter Act 1833
c) Charter Act 1853
d) Government of India Act 1858
Correct Answer: C
Your Answer: Unanswered
Explanation

Solution (c)

The previous charter act of 1833 had laid down that the Court of Directors should nominate
annually 4 times as many candidates as there were vacancies, from whom one should be
selected by competitive examination. The charter act of 1833 also provided the Haileybury
college of London should make quota to admit the future civil servants. However, this system of
an open competition was never effectively operated. A committee under the chairmanship of
Lord Macaulay had prepared the regulations in this context. The report said that:

• Haileybury should cease to be maintained as higher education college for the ICS

• There should be a broad general education rather than specialized education for the
ICS recruits

• The recruitment should be based upon an open competitive examination to bring out
the best candidates and not through mere superficial knowledge

• The appointments should be subject to a period of probation.

Charter Act of 1853 deprived the Court of Directors of its right of Patronage to Indian
appointments and now it was to be exercised under the regulations. This was the Birth of Civil
Services which was thrown in 1854 for open competition.

QUESTION 47. MTYyNjA0K1NhaSBLaXJhbitpbGJzbmFhNEBnbWFpbC5jb20rUVVFU1RJT04gNDY=


Which of the following were the features of Doctrine of Lapse Policy?

1. As per policy an adopted son of a ruler could not be proclaimed as the heir to the throne.

2. An adopted son will not inherit the personal property and estates of his foster father.

3. An adopted son would not be entitled to any pension given to his father by the British
Government.

Select the code from following:

a) 1 and 2
b) 2 and 3

IASbaba
Web: http://ilp.iasbaba.com/ Score:
Email: ilp@iasbaba.com 0.00 / 200
Page 36
2019 - Test 4-
Exam Title :
History & Cu...
Email : ilbsnaa4@gmail.com
Contact :

c) 1 and 3
d) All of the above
Correct Answer: C
Your Answer: Unanswered
Explanation

Solution (c)

Features of Doctrine of Lapse

• According to this, any princely state under the direct or indirect (as a vassal) control of
the East India Company where the ruler did not have a legal male heir would be annexed by the
company.

• This was not introduced by Lord Dalhousie even though it was he who documented it,
and used it widely to acquire territories for the British.

• As per this, any adopted son of the Indian ruler could not be proclaimed as heir to the
kingdom. The adopted son would only inherit his foster father’s personal property and estates.

• The adopted son would also not be entitled to any pension that his father had been
receiving or to any of his father’s titles.

• This challenged the Indian ruler’s long-held authority to appoint an heir of their choice.

QUESTION 48. MTYyNjA0K1NhaSBLaXJhbitpbGJzbmFhNEBnbWFpbC5jb20rUVVFU1RJT04gNDc=


Which of the following was distributed during the revolt of 1857 to create awareness?

a) Sindoor and leaves


b) Chapati and Lotus
c) Red and Green cloth
d) Broken sticks
Correct Answer: B
Your Answer: Unanswered
Explanation

Solution (b)

Chapati and Lotus became symbol of revolt of 1857.

Chapatis were distributed in villages over night to create awareness. People exchanged
chapattis. This created an environment of fear in the British ranks.

Lotus flower were thrown in the cantonment areas to gather support from other sepoys. Also a
message was sent – ‘Sab laal ho jayega’ i.e. everything will turn red.

QUESTION 49. MTYyNjA0K1NhaSBLaXJhbitpbGJzbmFhNEBnbWFpbC5jb20rUVVFU1RJT04gNDg=


For which of the following purposes, Peel Commission was constituted by the British
Government?

IASbaba
Web: http://ilp.iasbaba.com/ Score:
Email: ilp@iasbaba.com 0.00 / 200
Page 37
2019 - Test 4-
Exam Title :
History & Cu...
Email : ilbsnaa4@gmail.com
Contact :

a) Reorganisation of Army after revolt of 1857


b) Address the issue of corruption in East India Company.
c) To recommend measures for conducting exam for Civil Services in India
d) To return the annexed states to rulers after revolt of 1857
Correct Answer: A
Your Answer: Unanswered
Explanation

Solution (a)

In order to avoid another Mutiny in the Sepoy rankings, the British government constituted Peel
Commission to suggest measures to reorganize the army.

The Peel Commission was comprised of high-ranking officers of the British and Indian armies
and headed by the Secretary of State of War, Major-General Peel, with the purpose of policy
creation in post-Mutiny India.

It suggested several measures like – increasing the ratio of Europeans in the Indian army,
making regiments on caste lines initiating the policy of divide and rule, not promoting Indians
to the higher ranks.

QUESTION 50. MTYyNjA0K1NhaSBLaXJhbitpbGJzbmFhNEBnbWFpbC5jb20rUVVFU1RJT04gNDk=


Which of the following statements are the correct reasons of Vellore Mutiny?

1. There was a racial prejudice and Indian sepoys were considered inferior.

2. Wearing ear rings and caste marks were prohibited.

3. The sepoys were asked to shave the chin and trim the moustache.

4. It was a popular belief that all the soldiers would be slowly converted to Christianity.

Select the code from the following:

a) 1 and 4
b) 1,3 and 4
c) 1,2 and 3
d) All of the above
Correct Answer: D
Your Answer: Unanswered
Explanation

Solution (d)

Several causes are attributed to the Vellore Mutiny (1806). Indian sepoys had to experience
numerous difficulties when they went to serve in the Company’s army. The sepoys were forced
to serve under the Company since their earlier patrons (the native chieftains) were all
disappearing from the scene.

The strict discipline, practice, new weapons, new methods and uniforms were all new to the
sepoys. Anything new appears to be difficult and wrong for a man who is well-settled in the old

IASbaba
Web: http://ilp.iasbaba.com/ Score:
Email: ilp@iasbaba.com 0.00 / 200
Page 38
2019 - Test 4-
Exam Title :
History & Cu...
Email : ilbsnaa4@gmail.com
Contact :
way of life for a long-time. Sir John Cradock, the commander-in-chief, with the -approval of
Lord-William Bentinck, the Governor of Madras, introduced a new form of turban, resembling a
European hat. Wearing ear rings and caste marks were also prohibited.

The sepoys were asked to shave the chin and to trim the moustache. The sepoys felt that these
were designed to insult them and their religious and social traditions. There was also a popular
belief that this was the beginning of a process by which all of them would be converted to
Christianity. The English treated the Indian sepoys as their inferior. There was the racial
prejudice.

QUESTION 51. MTYyNjA0K1NhaSBLaXJhbitpbGJzbmFhNEBnbWFpbC5jb20rUVVFU1RJT04gNTA=


During 17th and 18th Century, the East India company had a monopoly in the eastern trade.
Bribes were often given to monarchs to have their political backing. Which of the following
statements are correct about the relations between east India company and the crown of
England?

1. In 17th century company gave huge loans to the crown.

2. Charles II granted series of charters, empowering company to build forts, raise army and
make peace and war with the eastern powers.

3. Company was allowed to administer justice to all Englishmen and others living in English
settlements.

Select the code from the following:

a) 1 and 2
b) 2 and 3
c) 1 and 3
d) All of the above
Correct Answer: D
Your Answer: Unanswered
Explanation

Solution (d)

Self explanatory.

QUESTION 52. MTYyNjA0K1NhaSBLaXJhbitpbGJzbmFhNEBnbWFpbC5jb20rUVVFU1RJT04gNTE=


Which of the following Organisations was established mainly to counter the ongoing social
reform movements led by protagonists such as Raja Ram Mohun Roy and Henry Derozio?

a) Paramhansa Mandali
b) Ramakrishna Mission
c) Adi Brahm Samaj
d) Dharma Sabha
Correct Answer: D
Your Answer: Unanswered

IASbaba
Web: http://ilp.iasbaba.com/ Score:
Email: ilp@iasbaba.com 0.00 / 200
Page 39
2019 - Test 4-
Exam Title :
History & Cu...
Email : ilbsnaa4@gmail.com
Contact :
Explanation

Solution (d)

Dharma Sabha

Dharma Sabha was formed in 1829 in Calcutta by Raja Radhakant Deb. The organization was
established mainly to counter the ongoing social reform movements led by protagonists such as
Raja Ram Mohun Roy and Henry Derozio. More specifically, the impetus of forming the
organization came from a new law enacted by the colonial Government which banned the
practice of sati in the country; the focus of the new association was to repel the law which was
seen as an intrusion by the Government into the religious affairs of the indigenous people by
some sections of the Hindu community.

QUESTION 53. MTYyNjA0K1NhaSBLaXJhbitpbGJzbmFhNEBnbWFpbC5jb20rUVVFU1RJT04gNTI=


Battle of Plassey between The Nawab of Bengal ‘Siraj ud Daulah’ and English East India
company made company a major contender of political power in India. Consider the following
statements:

1. Siraj ud Daulah was defeated treacherously and there was hardly any fight in Battle of
Plassey.

2. British put Mir Jafar as a puppet nawab in place of Siraj ud Daulah.

3. Mir Jafar was an able administrator who tried to maintain public discipline and tried to free
himself from the influence of the British.

Which of the above statements are correct:

a) 1 and 2
b) 2 and 3
c) 1 and 3
d) All of the above
Correct Answer: A
Your Answer: Unanswered
Explanation

Solution (a)

British put Mir Jafar as the Puppet nawab. However, Mir Jafar was not able to satisfy the greed
of his new British bosses. Mir Jafar was humiliated several times by the British and then finally
removed from the throne and a new Nawab Mir Qasim was made to sit there.

Mir Qasim was an able administrator and tried to free himself from the influence of British. This
finally culminated in the battle of Buxar.

QUESTION 54. MTYyNjA0K1NhaSBLaXJhbitpbGJzbmFhNEBnbWFpbC5jb20rUVVFU1RJT04gNTM=

IASbaba
Web: http://ilp.iasbaba.com/ Score:
Email: ilp@iasbaba.com 0.00 / 200
Page 40
2019 - Test 4-
Exam Title :
History & Cu...
Email : ilbsnaa4@gmail.com
Contact :

Fourth Anglo-Mysore war was a decisive war between British and Mysore. Tipu sultan was
killed defending the city. Which of the following statements are correct about fourth Anglo-
Mysore war?

1. Mysore was attacked from all four sides by British, Marathas and the Nizam of Hyderabad.

2. Tipu’s troops were outnumbered 4:1.

3. British annexed almost all of Mysore. The core area was restored to the eldest son of Tipu
whose ancestors ruled till 1947.

Select the correct code from the following:

a) 1 and 2
b) 2 and 3
c) 1 and 3
d) All of the above
Correct Answer: A
Your Answer: Unanswered
Explanation

Solution (a)

The Fourth Anglo-Mysore War (1799) saw the death of Tipu Sultan and further reductions in
Mysorean territory. Mysore's alliance with the French was seen as a threat to the East India
Company and Mysore was attacked from all four sides. Tipu's troops were outnumbered 4:1 in
this war. Mysore had 35,000 soldiers, whereas the British commanded 60,000 troops. The
Nizam of Hyderabad and the Marathas launched an invasion from the north. The British won a
decisive victory at the Siege of Seringapatam (1799). Tipu was killed during the defence of the
city. Much of the remaining Mysorean territory was annexed by the British, the Nizam and the
Marathas. The remaining core, around Mysore and Srirangapattana, was restored to the Indian
prince belonging to the Wodeyar dynasty, whose forefathers had been the actual rulers before
Hyder Ali became the de facto ruler. The Wodeyars ruled the remnant state of Mysore until
1947, when it joined the Union of India.

QUESTION 55. MTYyNjA0K1NhaSBLaXJhbitpbGJzbmFhNEBnbWFpbC5jb20rUVVFU1RJT04gNTQ=


Which of the following Acts ended the monopoly of trade of East India company in India?

a) Charter Act of 1793


b) Charter Act of 1813
c) Charter Act of 1833
d) Government of India Act 1858
Correct Answer: B
Your Answer: Unanswered
Explanation

Solution (b)

IASbaba
Web: http://ilp.iasbaba.com/ Score:
Email: ilp@iasbaba.com 0.00 / 200
Page 41
2019 - Test 4-
Exam Title :
History & Cu...
Email : ilbsnaa4@gmail.com
Contact :

The earlier charter act of 1793 had given the East India Company a monopoly to trade with East
for a period of 20 years. However, the rise of Napoleon Bonaparte had brought hard days to the
businessmen of England.

Charter act of 1813 ended the monopoly of the East India Company in India, however the
company’s monopoly in trade with china and trade in tea with India was kept intact. Thus, trade
with India for all commodities except Tea was thrown open to all British subjects. This lasted till
1833 when the next charter abolished the trade of the company.

QUESTION 56. MTYyNjA0K1NhaSBLaXJhbitpbGJzbmFhNEBnbWFpbC5jb20rUVVFU1RJT04gNTU=


Consider the following statements regarding Permanent Settlement method of land revenue:

1. It was introduced in Bengal and Bihar by Warren Hastings.

2. The zamindars were made owners of the land and ownership was made hereditary and
transferrable.

3. Cultivators were reduced to low status of mere tenants.

4. Zamindars were to give 50% of the rental they derived to the state, keeping 50% to
themselves.

Which of the above statements are correct?

a) 2 and 3
b) 1,2 and 3
c) 2,3 and 4
d) All of the above
Correct Answer: A
Your Answer: Unanswered
Explanation

Solution (a)

The Permanent Settlement of Bengal was brought into effect by the East India Company headed
by the Governor-General Lord Cornwallis in 1793. This was basically an agreement between the
company and the Zamindars to fix the land revenue. First enacted in Bengal, Bihar and Odisha,
this was later followed in northern Madras Presidency and the district of Varanasi. Cornwallis
thought of this system inspired by the prevailing system of land revenue in England where the
landlords were the permanent masters of their holdings and they collected revenue from the
peasants and looked after their interests. He envisaged the creation of a hereditary class of
landlords in India. This system was also called the Zamindari System.

Zamindars were to give 10/11 of the rental they derived from the peasantry to the state,
keeping 1/11 for them.

QUESTION 57. MTYyNjA0K1NhaSBLaXJhbitpbGJzbmFhNEBnbWFpbC5jb20rUVVFU1RJT04gNTY=

IASbaba
Web: http://ilp.iasbaba.com/ Score:
Email: ilp@iasbaba.com 0.00 / 200
Page 42
2019 - Test 4-
Exam Title :
History & Cu...
Email : ilbsnaa4@gmail.com
Contact :

The revolt of 1857 came as a bitter shock to the British empire. Since it started as an army
revolt, key changes were made in the Army to avoid such a situation again. Which of the
following statements are correct regarding the changes made in the Indian Army?

1. The proportion of Europeans to Indians in the army was increased.

2. European troops were kept in key geographical and military positions.

3. Policy of ‘divide and rule’ was implemented by making battalions on the basis of caste and
religion.

4. The older policy of excluding Indians from officer corps was abandoned and loyal Indians
were given higher posts.

Select the code from the following:

a) 2,3 and 4
b) 1,2 and 3
c) 1,3 and 4
d) All of the above
Correct Answer: B
Your Answer: Unanswered
Explanation

Solution (b)

4th option is wrong because the older policy of excluding Indians from officer corps was strictly
continued and not Indian could rise over the position of subedar.

QUESTION 58. MTYyNjA0K1NhaSBLaXJhbitpbGJzbmFhNEBnbWFpbC5jb20rUVVFU1RJT04gNTc=


Consider the following statements about Tattvabodhini Sabha:

1. It was founded by Rabindranath Tagore.

2. The objective of Tattvabodhini Sabha was to encourage religious inquiries and disseminate
the essence of Upanishads.

Which of the above statements are correct?

a) 1 only
b) 2 only
c) Both 1 and 2
d) Neither 1 nor 2
Correct Answer: B
Your Answer: Unanswered
Explanation

Solution (b)

It was founded by Debendranath Tagore in 1838. The Sabha was formed to propagate the ideas
of Raja Rammohun Roy. It was initially known as Adi Brahmo Samaj. The objective of

IASbaba
Web: http://ilp.iasbaba.com/ Score:
Email: ilp@iasbaba.com 0.00 / 200
Page 43
2019 - Test 4-
Exam Title :
History & Cu...
Email : ilbsnaa4@gmail.com
Contact :
Tattvabodhini Sabha was to encourage religious inquiries and disseminate the essence of
Upanishads.

QUESTION 59. MTYyNjA0K1NhaSBLaXJhbitpbGJzbmFhNEBnbWFpbC5jb20rUVVFU1RJT04gNTg=


Paika rebellion has been recently recognized by the Indian government as the first war of
Independence. Which of the following statements is/are correct regarding the rebellion?

1. Paikas were peasant militias of the Odisha rulers which rendered military service to the
throne during war.

2. Paikas rebelled peacefully against the British rule using the constitutional methods of
requests and petitions against discriminatory policies.

Select the code from below:

a) 1 only
b) 2 only
c) Both 1 and 2
d) Neither 1 nor 2
Correct Answer: A
Your Answer: Unanswered
Explanation

Solution (a)

Paika Rebellion of Odisha

Pre-dating what has been popularly regarded as the fist war of independence in 1857, the Paika
Bidroha (Paika Rebellion) of 1817 in Odisha briefly shook the foundations of British rule in the
eastern part of India. Paikas were essentially the peasant militias of the Gajapati rulers of
Odisha who rendered military service to the king during times of war while taking up
cultivation during times of peace. They unfurled the banner of rebellion against the British
under the leadership of Baxi Jagandhu Bidyadhara as early as 1817 to throw off the British
yoke.

Rulers of Khurda were traditionally the custodians of Jagannath Temple and ruled as the deputy
of lord Jagannath on earth. They symbolised the political and cultural freedom of the people of
Odisha. The British, having established their sway over Bengal Province and Madras Province
to the north and south of Odisha, occupied it in 1803.The Gajapati King of Odisha Mukunda
Deva-ll was a minor then and initial resistance by Jai Rajguru, the custodian of Mukunda Deva-
II, was put down brutally and Jai Rajguru was torn apart alive. A few years later, it was the
Paikas under Baxi Jagabandhu, the hereditary chief of the militia army of the Gajapati King,
who rose in rebellion, taking support of tribals and other sections of society. The rebellion
started in March 1817 and spread quickly. Though Paikas played a larger role in the rebellion
against the British, it was by no means a rebellion by a small group of people belonging to a
particular class. The tribals of Ghumusar (part of present day Ganjam and Kandhmal Districts)
and other sections of the population actively took part in it. In fact, the Paika Bidroha got the
opportune moment to spread when 4OOtribals of Ghumsar entered Khurda protesting against
the British rule. The Paikas attacked British symbols of power, setting ablaze police stations,
administrative offices and the treasury during their march towards Khurda, from where the
British fled.

IASbaba
Web: http://ilp.iasbaba.com/ Score:
Email: ilp@iasbaba.com 0.00 / 200
Page 44
2019 - Test 4-
Exam Title :
History & Cu...
Email : ilbsnaa4@gmail.com
Contact :

QUESTION 60. MTYyNjA0K1NhaSBLaXJhbitpbGJzbmFhNEBnbWFpbC5jb20rUVVFU1RJT04gNTk=


One of the earliest revolts against the British was led by the Fakirs and Sanyasis, popularly
known as the Sanyasi rebellion. Where did it occur?

a) Bengal
b) Madras
c) Bombay
d) Punjab
Correct Answer: A
Your Answer: Unanswered
Explanation

Solution (a)

Sanyasi Rebellion

The Sannyasi rebellion or Sannyasi revolt were the activities of sannyasis and fakirs (Hindu and
Muslim ascetics, respectively) in Bengal against the East India Company rule in the late 18th
century. It is also known as the Sannyasi rebellion which took place around Murshidabad and
Baikunthupur forests of Jalpaiguri. Historians have not only debated what events constitute the
rebellion, but have also varied on the significance of the rebellion in Indian history. While some
refer to it as an early war for India's independence from foreign rule, since the right to collect
tax had been given to the British East India Company after the Battle of Buxar in 1764, others
categorize it as acts of violent banditry following the depopulation of the province in the Bengal
famine of 1770.

QUESTION 61. MTYyNjA0K1NhaSBLaXJhbitpbGJzbmFhNEBnbWFpbC5jb20rUVVFU1RJT04gNjA=


Which of the following planets is the hottest?

a) Mercury
b) Venus
c) Jupiter
d) Mars
Correct Answer: B
Your Answer: Unanswered
Explanation

Solution (b)

Mercury is the planet that is closest to the sun and therefore gets more direct heat, but even it
isn't the hottest. Venus is the second planet from the sun and has a temperature that is
maintained at 462 degrees Celsius, no matter where you go on the planet. It is the hottest
planet in the solar system.

The main reason for the highest temperature of Venus is the presence of a large amount of
Green house gases.

IASbaba
Web: http://ilp.iasbaba.com/ Score:
Email: ilp@iasbaba.com 0.00 / 200
Page 45
2019 - Test 4-
Exam Title :
History & Cu...
Email : ilbsnaa4@gmail.com
Contact :

QUESTION 62. MTYyNjA0K1NhaSBLaXJhbitpbGJzbmFhNEBnbWFpbC5jb20rUVVFU1RJT04gNjE=


Which of the following statements correctly explains a ‘Cordillera’?

a) It is a massive mountain range with two or more parallel mountain ranges.


b) It is a huge volcanic depression formed due to collapse of a volcano crater.
c) It is lake formed in mountain valleys.
d) It is lake formed in depressions of rocky deserts.
Correct Answer: A
Your Answer: Unanswered
Explanation

Solution (a)

A Cordillera is a system of mountain ranges often consisting of a number of more or less


parallel chains. Eg. Rockies of North America, Himalayas of India etc.

Think

• Caldera

QUESTION 63. MTYyNjA0K1NhaSBLaXJhbitpbGJzbmFhNEBnbWFpbC5jb20rUVVFU1RJT04gNjI=


‘Insular shelf’ means

a) A continental shelf allocated to landlocked countries.


b) The shelf surrounding an island.
c) The shelf formed out of deposition.
d) The shelf formed by fall in sea level.
Correct Answer: B
Your Answer: Unanswered
Explanation

Solution (b)

The continental shelf is an underwater landmass which extends from a continent, resulting in
an area of relatively shallow water known as a shelf sea. Much of the shelves were exposed
during glacial periods and interglacial periods.

The shelf surrounding an island is known as an insular shelf.

Do you know?

Economic significance of continental shelf

• Marine food comes almost entirely from continental shelves;

• They provide the richest fishing grounds;

• They are potential sites for polymetallic nodules, petroleum.

IASbaba
Web: http://ilp.iasbaba.com/ Score:
Email: ilp@iasbaba.com 0.00 / 200
Page 46
2019 - Test 4-
Exam Title :
History & Cu...
Email : ilbsnaa4@gmail.com
Contact :

THINK!

• International Seabed Authority

QUESTION 64. MTYyNjA0K1NhaSBLaXJhbitpbGJzbmFhNEBnbWFpbC5jb20rUVVFU1RJT04gNjM=


Which of the following statements regarding Rift Valleys are correct?

1. They are formed due to Continental – continental divergence.

2. They have flat bottom topography.

3. Himalayan rivers make Rifts Valleys in their old age.

Select the code from following:

a) 1 and 2
b) 2 and 3
c) 1 and 3
d) All of the above
Correct Answer: A
Your Answer: Unanswered
Explanation

Solution (a)

Rift Valley

A rift valley is linear-shaped lowland between several highlands or mountain ranges created by
the action of a geologic rift or fault. A rift valley is formed on a divergent plate boundary, a
crustal extension or spreading apart of the surface, which is subsequently further deepened by
the forces of erosion. When the tensional forces were strong enough to cause the plate to split
apart, a center block dropped between the two blocks at its flanks, forming a graben. The drop
of the center creates the nearly parallel steeply dipping walls of a rift valley when it is new.
That feature is the beginning of the rift valley, but as the process continues, the valley widens,
until it becomes a large basin that fills with sediment from the rift walls and the surrounding
area. One of the best known examples of this process is the East African Rift.

Note: Himalayan rivers do not form a rift valley. Rift valley is a secondary landform and is
formed because of endogenic forces and not due to erosion.

QUESTION 65. MTYyNjA0K1NhaSBLaXJhbitpbGJzbmFhNEBnbWFpbC5jb20rUVVFU1RJT04gNjQ=


Tanganyika, Malawi and Dead sea are examples of

a) Man – made lake


b) Crater lakes
c) Rift Valley Lakes
d) Volcanic Lakes

IASbaba
Web: http://ilp.iasbaba.com/ Score:
Email: ilp@iasbaba.com 0.00 / 200
Page 47
2019 - Test 4-
Exam Title :
History & Cu...
Email : ilbsnaa4@gmail.com
Contact :
Correct Answer: C
Your Answer: Unanswered
Explanation

Solution (c)

Rift valley lakes

• A rift valley is formed when two blocks of earth move apart letting the ‘in between’
block slide downwards. Or, it’s a sunken land between two parallel faults.

• Rift valleys are deep, narrow and elongated. Hence the lakes formed along rift valleys
are also deep, narrow and very long.

• Water collects in troughs (Valley in the rift) and their floors are often below sea level.

• The best-known example is the East African Rift Valley which runs through Zambia,
Malawi, Tanzania, Kenya and Ethiopia, and extends along the Red Sea to Israel and Jordan over
a total distance of 3,000 miles.

• It includes such lakes as Lakes Tanganyika, Malawi, Rudolf, Edward, Albert, as well as
the Dead Sea1,286 feet below mean sea level, the world’s lowest lake.

QUESTION 66. MTYyNjA0K1NhaSBLaXJhbitpbGJzbmFhNEBnbWFpbC5jb20rUVVFU1RJT04gNjU=


Consider the following pairs.

Plateaus Major mineral

1. Kimberley Gold

2. Katanga Copper

3. Laurentian Iron ore

4. Mexican Silver

Which of the above pairs is/are correctly matched?

a) 1 and 3 only
b) 1 and 4 only
c) 2 and 4 only
d) All the above
Correct Answer: D
Your Answer: Unanswered
Explanation

Solution (d)

Kimberley Plateau

• Lies in the northern part of Australia.

• This plateau is made of volcanic eruption.

IASbaba
Web: http://ilp.iasbaba.com/ Score:
Email: ilp@iasbaba.com 0.00 / 200
Page 48
2019 - Test 4-
Exam Title :
History & Cu...
Email : ilbsnaa4@gmail.com
Contact :

• Many minerals like iron, gold, lead, zinc, silver and diamond are found here.

• Diamond is also found here.

Katanga Plateau

• It is lying in Congo.

• It is famous for copper production.

• Other minerals like Cobalt, Uranium, Zinc, Silver, Gold and Tin are also mined here.

Laurentian Plateau

• Lying in the eastern part of Canada, it is a part of Canadian Shield.

• Fine quality of iron-ore is found here.

Mexican Plateau

• It is called as ‘Mineral Store’. Different types of metallic minerals like silver, copper
etc. are obtained from here.

• World’s biggest silver mine Chihuahua is situated in the plateau.

Do you know?

Patagonian Plateau

• It is a Piedmont plateau (Arid Landforms) lying in southern part of Argentina.

• It is a rain shadow desert plateau.

• It is an important region for sheep rearing.

THINK!

• Types of plains

QUESTION 67. MTYyNjA0K1NhaSBLaXJhbitpbGJzbmFhNEBnbWFpbC5jb20rUVVFU1RJT04gNjY=


Which of the following are the necessary conditions for the formation of deltas?

1. The river must have large load.

2. Presence of large lakes on the river course.

3. Presence of shallow adjoining sea or continental shelf.

4. Strong currents running at right angles to the mouth of the river.

Select the correct answer using the codes given below.

a) 1 and 4 only
b) 1 and 3 only

IASbaba
Web: http://ilp.iasbaba.com/ Score:
Email: ilp@iasbaba.com 0.00 / 200
Page 49
2019 - Test 4-
Exam Title :
History & Cu...
Email : ilbsnaa4@gmail.com
Contact :

c) 1, 2 and 3 only
d) All the above
Correct Answer: B
Your Answer: Unanswered
Explanation

Solution (b)

The following are necessary conditions for formation of river delta

• The river must have large load. This will be possible if there is active erosion in the
upper and middle stages.

• There should not be extensive deposition in the middle stage e.g. presence of lake in
between or high evaporation rate (first).

• The river’s load must be deposited faster than it can be removed by the action of
currents and tides i.e no strong current should be at right angle to the mouth of the river.

• Presence of shallow adjoining sea or continental shelf.

• The velocity of a river must be sufficiently low to allow most of its load to be deposited
in the river’s mouth.

Do you know?

A delta is formed by a combination of two processes:

• load-bearing capacity of a river is reduced as a result of the check to its speed as it


enters a sea or lake, and

• clay particles carried in suspension in the river coagulate in the presence of salt water
and are deposited.

• The finest particles are carried farthest to accumulate as bottom-set beds.

THINK!

• Types of deltas.

QUESTION 68. MTYyNjA0K1NhaSBLaXJhbitpbGJzbmFhNEBnbWFpbC5jb20rUVVFU1RJT04gNjc=


Primary (P) Waves generated during an earthquake. What type of waves are P Waves?

a) Transverse
b) Electromagnetic
c) Sound
d) Longitudinal
Correct Answer: D
Your Answer: Unanswered
Explanation

Solution (d)

IASbaba
Web: http://ilp.iasbaba.com/ Score:
Email: ilp@iasbaba.com 0.00 / 200
Page 50
2019 - Test 4-
Exam Title :
History & Cu...
Email : ilbsnaa4@gmail.com
Contact :

P and S waves

A wave is a vibration that transfers energy from one place to another without transferring
matter (solid, liquid or gas). Light and sound both travel in this way.

Energy released during an earthquake travels in the form of waves around the Earth. Two types
of seismic wave exist, P- and S-waves. They are different in the way that they travel through the
Earth.

P-waves (P stands for primary) arrive at the detector first. They are longitudinal waves which
mean the vibrations are along the same direction as the direction of travel. Other examples of
longitudinal waves include sound waves and waves in a stretched spring.

S-waves (S stands for secondary) arrive at the detector of a seismometer second. They are
transverse waves which mean the vibrations are at right angles to the direction of travel. Other
examples of transverse waves include light waves and water waves.

QUESTION 69. MTYyNjA0K1NhaSBLaXJhbitpbGJzbmFhNEBnbWFpbC5jb20rUVVFU1RJT04gNjg=


Which of the following statements are correct regarding ‘Shield Type’ Volcanoes?

1. They are formed by solidification of very fluid Basaltic lava.

2. These volcanoes blast violently, throwing lava and projectiles to great heights.

3. They have steep slopes.

Select the code from following:

a) 1 only
b) 2 and 3
c) 1 and 3
d) None of the above
Correct Answer: A
Your Answer: Unanswered
Explanation

Solution (a)

Shield Type Volcanic Landforms

• The Hawaiian volcanoes are the most famous examples.

• These volcanoes are mostly made up of basalt, a type of lava that is very fluid when
erupted.

• These volcanoes are not steep.

• They become explosive if somehow water gets into the vent; otherwise, they are less
explosive.

• Example: Mauna Loa (Hawaii).

IASbaba
Web: http://ilp.iasbaba.com/ Score:
Email: ilp@iasbaba.com 0.00 / 200
Page 51
2019 - Test 4-
Exam Title :
History & Cu...
Email : ilbsnaa4@gmail.com
Contact :

QUESTION 70. MTYyNjA0K1NhaSBLaXJhbitpbGJzbmFhNEBnbWFpbC5jb20rUVVFU1RJT04gNjk=


Which of the following landforms are associated with Oceanic – Oceanic plate convergence?

1. Oceanic trench

2. Oceanic ridge

3. Volcanic island chain

4. Folding mountains

Select the code from following:

a) 1 and 2
b) 2,3 and 4
c) 1 and 3
d) All of the above
Correct Answer: C
Your Answer: Unanswered
Explanation

Solution (c)

Note: Study the following table very carefully. If you can understand and remember the
following table, you will be able to explain the formation of all the secondary reliefs.

Plate Events
Plate boundary seafloor examples
movement observed

Ridge forms at
spreading
Forms by centre. Plate Mid atlantic
Ocean -
seafloor area increases. ridge, east
ocean
spreading Many small pacific rise
volcanoes and
earthquakes
Divergent plate
apart
boundaries
Continent drifts
New ocean
apart, ocean
basin may
Continent - may intrude. East African
form as the
continent Formation of rift.
continent
rift valleys and
split
block mountains

IASbaba
Web: http://ilp.iasbaba.com/ Score:
Email: ilp@iasbaba.com 0.00 / 200
Page 52
2019 - Test 4-
Exam Title :
History & Cu...
Email : ilbsnaa4@gmail.com
Contact :

Dense oceanic
lithosphere
plunges
beneath less
dense
continental
crust.
Earthquake
traces path of
Destroyed
down moving Western
Ocean - at
plate as it South
continent subduction
descends into america
zones
asthenosphere.
A trench is
formed.
Subducted plate
partially melts
and magma
rises to form
continental
volcanoes.
Convergent plate
together
boundary
Denser crust
plunges into
lighter crust
Destroyed
and is
Ocean - at
subducted Aleutians
ocean subduction
forming a
zone
curved trench
and a volcanic
arc.

Collision
between masses
of gigantic
continental
lithosphere.
Continent- Himalayas,
NA Neither mass is
continent alps
subducted.
Plate edges are
compressed,
folded and
uplifted

IASbaba
Web: http://ilp.iasbaba.com/ Score:
Email: ilp@iasbaba.com 0.00 / 200
Page 53
2019 - Test 4-
Exam Title :
History & Cu...
Email : ilbsnaa4@gmail.com
Contact :

A transform
fault is formed
Neither where plates San
Transform plate Past each
created nor move past each Andreas
boundary other
destroyed other. Strong fault.
earthquakes
along the fault

QUESTION 71. MTYyNjA0K1NhaSBLaXJhbitpbGJzbmFhNEBnbWFpbC5jb20rUVVFU1RJT04gNzA=


Which of the following changes were made in the Constituent Assembly by the Indian
Independence Act 1947?

1. The Assembly was made a fully sovereign body and the act empowered the Assembly to
abrogate or alter any law made by the British Parliament in relation to India.

2. Two separate functions were assigned to the Assembly, that is, making of a constitution for
free India and enacting of ordinary laws for the country.

Select the code from following:

a) 1 only
b) 2 only
c) Both 1 and 2
d) Neither 1 nor 2
Correct Answer: C
Your Answer: Unanswered
Explanation

Solution (c)

The Indian Independence Act of 1947 made the following three changes in the position of the
Assembly:

1. The Assembly was made a fully sovereign body, which could frame any Constitution it
pleased. The act empowered the Assembly to abrogate or alter any law made by the British
Parliament in relation to India.

2. The Assembly also became a legislative body. In other words, two separate functions were
assigned to the Assembly, that is, making of a constitution for free India and enacting of
ordinary laws for the country. These two tasks were to be performed on separate days. Thus,
the Assembly became the first Parliament of free India (Dominion Legislature). Whenever the
Assembly met as the Constituent body it was chaired by Dr. Rajendra Prasad and when it met as
the legislative body, it was chaired by G V Mavlankar. These two functions continued till
November 26, 1949, when the task of making the Constitution was over.

QUESTION 72. MTYyNjA0K1NhaSBLaXJhbitpbGJzbmFhNEBnbWFpbC5jb20rUVVFU1RJT04gNzE=


Which of the following Major Committees was not headed by Jawaharlal Nehru?

IASbaba
Web: http://ilp.iasbaba.com/ Score:
Email: ilp@iasbaba.com 0.00 / 200
Page 54
2019 - Test 4-
Exam Title :
History & Cu...
Email : ilbsnaa4@gmail.com
Contact :

a) Union Powers Committee


b) Union Constitution Committee
c) Advisory Committee on Fundamental Rights
d) States Committee
Correct Answer: C
Your Answer: Unanswered
Explanation

Solution (c)

Major Committees

1. Union Powers Committee – Jawaharlal Nehru

2. Union Constitution Committee – Jawaharlal Nehru

3. Provincial Constitution Committee – Sardar Patel

4. Drafting Committee – Dr. B.R. Ambedkar

5. Advisory Committee on Fundamental Rights, Minorities and Tribal and Excluded Areas –
Sardar Patel. This committee had the following sub-committes:

(a) Fundamental Rights Sub-Committee – J.B. Kripalani

(b) Minorities Sub-Committee – H.C. Mukherjee

(c) North-East Frontier Tribal Areas and Assam Excluded & Partially Excluded Areas Sub-

Committee – Gopinath Bardoloi

(d) Excluded and Partially Excluded Areas (Other than those in Assam) Sub-Committee – A.V.
Thakkar

6. Rules of Procedure Committee – Dr. Rajendra Prasad

7. States Committee (Committee for Negotiating with States) – Jawaharlal Nehru

8. Steering Committee – Dr. Rajendra Prasad

QUESTION 73. MTYyNjA0K1NhaSBLaXJhbitpbGJzbmFhNEBnbWFpbC5jb20rUVVFU1RJT04gNzI=


Which of the following terms were added to the Preamble by 44th Constitutional Amendment?

1. Socialist

2. Secular

3. Integrity

Select the code from following:

a) 1 and 2

IASbaba
Web: http://ilp.iasbaba.com/ Score:
Email: ilp@iasbaba.com 0.00 / 200
Page 55
2019 - Test 4-
Exam Title :
History & Cu...
Email : ilbsnaa4@gmail.com
Contact :

b) 2 and 3
c) 1 and 3
d) None of the above
Correct Answer: D
Your Answer: Unanswered
Explanation

Solution (d)

All of the above terms were added by 42nd Constitutional Amendment rather than 44th.

QUESTION 74. MTYyNjA0K1NhaSBLaXJhbitpbGJzbmFhNEBnbWFpbC5jb20rUVVFU1RJT04gNzM=


Article 3 of the Constitution authorizes the parliament to form a new state by separation of
territory from any state or by uniting two or more states or parts of states or by uniting any
territory to a part of any state. Which of the following statements regarding this are correct?

1. A bill contemplating the above changes can be introduced in the Parliament only with the
prior recommendation of the President.

2. The President has to refer the bill to the state legistature concerned for expressing its views
within a specified period.

3. The president is bound to act according to the recommendation of the States concerned in
case of change of boundary of the State.

Select the code from following:

a) 1 and 2
b) 2 and 3
c) 1 and 3
d) All of the above
Correct Answer: A
Your Answer: Unanswered
Explanation

Solution (a)

Article 3 authorises the Parliament to:

• form a new state by separation of territory from any state or by uniting two or more
states or parts of states or by uniting any territory to a part of any state,

• increase the area of any state,

• diminish the area of any state,

• alter the boundaries of any state, and

• alter the name of any state.

IASbaba
Web: http://ilp.iasbaba.com/ Score:
Email: ilp@iasbaba.com 0.00 / 200
Page 56
2019 - Test 4-
Exam Title :
History & Cu...
Email : ilbsnaa4@gmail.com
Contact :

However, Article 3 lays down two conditions in this regard: one, a bill contemplating the above
changes can be introduced in the Parliament only with the prior recommendation of the
President; and two, before recommending the bill, the President has to refer the same to the
state legistature concerned for expressing its views within a specified period.

Further, the power of Parliament to form new states includes the power to form a new state or
union territory by uniting a part of any state or union territory to any other state or union
territory.

The President (or Parliament) is not bound by the views of the state legislature and may either
accept or reject them, even if the views are received in time.

QUESTION 75. MTYyNjA0K1NhaSBLaXJhbitpbGJzbmFhNEBnbWFpbC5jb20rUVVFU1RJT04gNzQ=


Which of the following rights have been conferred only to Indian Citizens by the Constitution?

1. Right against discrimination on grounds of religion, race, caste, sex or place of birth (Article
15).

2. Right to equality of opportunity in the matter of public employment (Article 16).

3. Right to freedom of speech and expression, assembly, association, movement, residence and
profession (Article 19).

4. Cultural and educational rights (Articles 29 and 30).

Select the code from following:

a) 1,2 and 3
b) 2,3 and 4
c) 1,3 and 4
d) All of the above
Correct Answer: D
Your Answer: Unanswered
Explanation

Solution (d)

Constitution confers the following rights and privileges on the citizens of India (and denies the
same to aliens):

• Right against discrimination on grounds of religion, race, caste, sex or place of birth
(Article 15).

• Right to equality of opportunity in the matter of public employment (Article 16).

• Right to freedom of speech and expression, assembly, association, movement,


residence and profession (Article 19).

• Cultural and educational rights (Articles 29 and 30).

• Right to vote in elections to the Lok Sabha and state legislative assembly.

IASbaba
Web: http://ilp.iasbaba.com/ Score:
Email: ilp@iasbaba.com 0.00 / 200
Page 57
2019 - Test 4-
Exam Title :
History & Cu...
Email : ilbsnaa4@gmail.com
Contact :

• Right to contest for the membership of the Parliament and the state legislature.

• Eligibility to hold certain public offices, that is, President of India, Vice-President of
India, judges of the Supreme Court and the high courts, governor of states, attorney general of
India and advocate general of states.

Think

• Which of the Fundamental Rights are given to Aliens?

QUESTION 76. MTYyNjA0K1NhaSBLaXJhbitpbGJzbmFhNEBnbWFpbC5jb20rUVVFU1RJT04gNzU=


Consider the following statements about ‘E-Way Bill’ System

1. It offers the technological framework to track intra-state as well as inter-state movements of


goods under the Goods and Services Tax (GST) regime

2. One e-way bill will be valid throughout the country for the movement of goods and it does
away with the separate transit pass for every state

Select the correct statements

a) 1 Only
b) 2 Only
c) Both 1 and 2
d) Neither 1 nor 2
Correct Answer: C
Your Answer: Unanswered
Explanation

Solution (c)

E-Way Bill

• An electronic way bill or ‘e-way bill’ system offers the technological framework to track
intra-state as well as inter-state movements of goods of value exceeding Rs 50,000, for sales
beyond 10 km in the new Goods and Services Tax (GST) regime.

• The E-way bill must be raised before the goods are shipped and should include details
of the goods, their consignor, recipient and transporter.

• Under the e-way bill system, there will be no need for a separate transit pass for every
state — one e-way bill will be valid throughout the country for the movement of goods.

• Every E-way bill generated by a sender or buyer of goods is to be automatically


updated in the outward sales return (GSTR1) of the supplier162604

• Whether goods are transported on one’s own or hired conveyance, by air, rail or road,
the E-way bill has to be generated.

• Where the goods are handed over to a transporter for conveyance by road and neither
the consignor nor the consignee has generated the E-way Bill, the transporter becomes liable to
generate it.

IASbaba
Web: http://ilp.iasbaba.com/ Score:
Email: ilp@iasbaba.com 0.00 / 200
Page 58
2019 - Test 4-
Exam Title :
History & Cu...
Email : ilbsnaa4@gmail.com
Contact :

• When the consignor or transporter generates the E-way bill, the recipient for the
consignment has to either accept or reject it on the portal. If no action is taken by the recipient
in 72 hours, it shall be taken as accepted.

Source: http://pib.nic.in/PressReleseDetail.aspx?PRID=1541576

QUESTION 77. MTYyNjA0K1NhaSBLaXJhbitpbGJzbmFhNEBnbWFpbC5jb20rUVVFU1RJT04gNzY=


Consider the following statements with respect to ‘Alternative Investment Funds (AIFs)’ in India

1. It refers to any privately pooled investment fund from Indian sources only

2. Venture Capital Fund is a type of AIF

3. They are not taxable under the Income Tax Act

Select the correct statements

a) 1 and 3
b) 1 and 2
c) 2 Only
d) 1, 2 and 3
Correct Answer: C
Your Answer: Unanswered
Explanation

Solution (c)

Alternative Investment Funds

· In India, alternative investment funds (AIFs) are defined in Regulation 2(1)(b) of Securities
and Exchange Board of India (Alternative Investment Funds) Regulations, 2012.

· It refers to any privately pooled investment fund, (whether from Indian or foreign sources),
in the form of a trust or a company or a body corporate or a Limited Liability Partnership (LLP)
which are not presently covered by any Regulation of SEBI governing fund management (like,
Regulations governing Mutual Fund or Collective Investment Scheme)nor coming under the
direct regulation of any other sectoral regulators in India-IRDA, PFRDA, RBI.

· Hence, in India, AIFs are private funds which are otherwise not coming under the
jurisdiction of any regulatory agency in India.

Types of AIFs

· Category I AIF are those AIFs with positive spill over effects on the economy, for which
certain incentives or concessions might be considered by SEBI or Government of India

· Category II AIF are those AIFs for which no specific incentives or concessions are given

· Category III AIF are funds that are considered to have some potential negative
externalities in certain situations and which undertake leverage to a great extent

They are taxable.

IASbaba
Web: http://ilp.iasbaba.com/ Score:
Email: ilp@iasbaba.com 0.00 / 200
Page 59
2019 - Test 4-
Exam Title :
History & Cu...
Email : ilbsnaa4@gmail.com
Contact :

THINK!

· Green Growth Equity Fund (GGEF)

· National Investment Fund

· National Investment and Infrastructure Fund

Source: http://pib.nic.in/PressReleseDetail.aspx?PRID=1541570

QUESTION 78. MTYyNjA0K1NhaSBLaXJhbitpbGJzbmFhNEBnbWFpbC5jb20rUVVFU1RJT04gNzc=


‘Vidya Lakshmi Portal’ caters to

a) Massive open online course (MOOCs)


b) Educational Loans
c) Female Education
d) PhDs for Minority Women Graduates
Correct Answer: B
Your Answer: Unanswered
Explanation

Solution (b)

Vidya Lakshmi Portal

· It was launched by the Government to ensure that students can avail loans easily through
single window system of banks for education loans.

· It has been developed under the guidance of Department of Financial Services (Ministry of
Finance), Department of Higher Education (Ministry of Human Resource Development) and
Indian Banks Association (IBA)

Source: http://www.pib.nic.in/Pressreleaseshare.aspx?PRID=1541568

QUESTION 79. MTYyNjA0K1NhaSBLaXJhbitpbGJzbmFhNEBnbWFpbC5jb20rUVVFU1RJT04gNzg=


Which of the following is correct about ‘Article 335’ of the Indian Constitution’?

a) It also allows the president to prolong the life of the Lok Sabha one year at time.
b) It empowers the central government to make funding allocations within the centre-state
financial arrangements
c) It deals with the Claims of Scheduled Castes and Scheduled Tribes to services and posts
and the requirement of the state to acknowledge the claims of the SCs/STs while ‘making
appointments to posts and services’.
d) It provides the president with powers to declare a state of emergency in the nation
Correct Answer: C
Your Answer: Unanswered
Explanation

IASbaba
Web: http://ilp.iasbaba.com/ Score:
Email: ilp@iasbaba.com 0.00 / 200
Page 60
2019 - Test 4-
Exam Title :
History & Cu...
Email : ilbsnaa4@gmail.com
Contact :

Solution (c)

The main focus of Article 335 of the Constitution is the requirement of the state to acknowledge
the claims of the SCs/STs while ‘making appointments to posts and services’. However, Article
335 also states that the acknowledgement of such claims shall be consistent with the concerns
of efficiency.

THINK!

· M. Nagaraj v. Union of India, 2006

Source: https://www.financialexpress.com/opinion/what-backwardness-not-all-sc-st-are-
historically-backward/1273036/

QUESTION 80. MTYyNjA0K1NhaSBLaXJhbitpbGJzbmFhNEBnbWFpbC5jb20rUVVFU1RJT04gNzk=


Consider the following statements about ‘Public Financial Management System (PFMS)’

1. It is a web-based online software application developed and implemented by the Office of


Controller General of Accounts (CGA)

2. The ambit of PFMS coverage includes Central Sector and Centrally Sponsored Schemes as
well as other expenditures including the Finance Commission Grants

3. Use of PFMS is mandatory for all the Central Sector Schemes of the Government of India

Select the correct statements

a) 1 and 2
b) 2 and 3
c) 1 and 3
d) All of the above
Correct Answer: D
Your Answer: Unanswered
Explanation

Solution (d)

The Public Financial Management System (PFMS),earlier known as Central Plan Schemes
Monitoring System (CPSMS), is a web-based online software application developed and
implemented by the Office of Controller General of Accounts (CGA).

The primary objective of PFMS is to facilitate sound Public Financial Management System for
Government of India (GoI) by establishing an efficient fund flow system as well as a payment
cum accounting network. PFMS provides various stakeholders with a real time, reliable and
meaningful management information system and an effective decision support system, as part
of the Digital India initiative of GoI.

Finance Ministry launched the mandatory use of Public Finance Management System (PFMS)
for all Central Sector Schemes, stating that PFMS would ensure that the benefits of the various
Government Schemes reach to the last mile.

IASbaba
Web: http://ilp.iasbaba.com/ Score:
Email: ilp@iasbaba.com 0.00 / 200
Page 61
2019 - Test 4-
Exam Title :
History & Cu...
Email : ilbsnaa4@gmail.com
Contact :

Read More - http://pib.nic.in/newsite/PrintRelease.aspx?relid=171984

The PFMS Scheme has been rolled-out by the Controller General of Accounts (CGA) at the
behest of Finance Ministry, Department of Expenditure as a cherished Public Finance
Management (PFM) reform in the country. The Scheme aims at promoting transparency and
bringing about tangible improvements in the overall Central Government Financial
Management as well as implementation of various Central Government Schemes across the
country. The ambit of PFMS coverage includes Central Sector and Centrally Sponsored
Schemes as well as other expenditures including the Finance Commission Grants.

Source: http://pib.nic.in/PressReleseDetail.aspx?PRID=1541657

QUESTION 81. MTYyNjA0K1NhaSBLaXJhbitpbGJzbmFhNEBnbWFpbC5jb20rUVVFU1RJT04gODA=


Consider the following statements about ‘Jal Marg Vikas Project (JMVP)’

1. It is being executed on Brahmaputra-Barak river systems to connect northeast to the rest of


India

2. World Bank is providing the technical and investment support for the implementation of the
project

Select the correct statements

a) 1 Only
b) 2 Only
c) Both 1 and 2
d) Neither 1 nor 2
Correct Answer: B
Your Answer: Unanswered
Explanation

Solution (b)

Jal Marg Vikas Project (JMVP)

· JMVP on NW-1 is being implemented with the financial and technical support of the World
Bank.

· The Project entails development of fairway with 3 meters depth between Varanasi and
Haldia (Phase-I) covering a distance of 1380 km.

· The project falls in Uttar Pradesh, Bihar, Jharkhand and West Bengal

Benefits

· It will provide an alternative mode of environment-friendly and cost-effective transport.

· It will contribute to bringing down logistics costs in the country

· It will boost infrastructure development like multi-modal and inter-modal terminals, Roll
on-Roll off (Ro-Ro) facilities, ferry services and navigation aids.

IASbaba
Web: http://ilp.iasbaba.com/ Score:
Email: ilp@iasbaba.com 0.00 / 200
Page 62
2019 - Test 4-
Exam Title :
History & Cu...
Email : ilbsnaa4@gmail.com
Contact :

Source: http://pib.nic.in/PressReleseDetail.aspx?PRID=1541662

QUESTION 82. MTYyNjA0K1NhaSBLaXJhbitpbGJzbmFhNEBnbWFpbC5jb20rUVVFU1RJT04gODE=


‘RISECREEK’ was in recently in news. What is it?

a) A newly developed microprocessor


b) A new technology to tackle coral bleaching
c) A newly formed island Hvar, Croatia
d) A supersonic airplane developed by China
Correct Answer: A
Your Answer: Unanswered
Explanation

Solution (a)

News: Computer scientists and a student team from the IIT- Madras have developed a industry-
standard microprocessors (RISECREEK).

About

· It can be adapted by others, as the design is open source

· It can meet the demands of defence and strategic equipment such as NAVIC (Indian
Regional Navigation Satellite) and Internet of Things (IoT) electronics.

· The initial batch of 300 chips named RISECREEK, produced under Project Shakti.

· Project Shakti started in 2014 as an IIT-M initiative.

Source: https://www.thehindu.com/sci-tech/technology/iit-madras-powers-up-a-desi-
chip/article24609946.ece

QUESTION 83. MTYyNjA0K1NhaSBLaXJhbitpbGJzbmFhNEBnbWFpbC5jb20rUVVFU1RJT04gODI=


Consider the following statements with respect to ‘Indian star tortoises’

1. It is listed as ‘Critically Endangered’ in the red list of endangered species of the IUCN

2. It is endemic to Western Ghats Only

3. It is liable for confiscation under the Customs Act, 1962

Select the correct statements

a) 1 and 3
b) 2 and 3
c) 3 Only
d) 2 and 3

IASbaba
Web: http://ilp.iasbaba.com/ Score:
Email: ilp@iasbaba.com 0.00 / 200
Page 63
2019 - Test 4-
Exam Title :
History & Cu...
Email : ilbsnaa4@gmail.com
Contact :
Correct Answer: C
Your Answer: Unanswered
Explanation

Solution (c)

Indian star tortoises

· It is categorised as ‘Vulnerable’ in the red list of endangered species of the IUCN.

· It is listed in the Schedule IV of the Wild Life (Protection) Act, 1972 and prohibited from
export under the Foreign Trade Policy.

· The species are liable for confiscation under the Customs Act, 1962.

· It has been included on Appendix II of the CITES

· Native: India (Andhra Pradesh, Karnataka, Orissa, Tamil Nadu); Pakistan; Sri Lanka

· It naturally inhabits scrub forests, grasslands, and some coastal scrublands of arid and
semi-arid regions throughout its wide range, but also commonly inhabits human-dominated
landscapes.

Source: https://www.thehindu.com/news/cities/Visakhapatnam/1125-indian-star-
tortoises-seized-from-three-train-passengers/article24610332.ece

QUESTION 84. MTYyNjA0K1NhaSBLaXJhbitpbGJzbmFhNEBnbWFpbC5jb20rUVVFU1RJT04gODM=


What is ‘Starry Sky-2’, recently in the news?

a) Electric plane tested by NASA


b) Hypersonic flight vehicle developed by China
c) Solar-powered two-seater aircraft designed by Japan
d) Space observatory launched by China
Correct Answer: B
Your Answer: Unanswered
Explanation

Solution (b)

Xingkong-2/Starry Sky-2

· It is a Chinese hypersonic aircraft which could carry nuclear warheads and penetrate any
current generation anti-missile defence systems

Source: https://www.thehindu.com/todays-paper/tp-life/china-tests-hypersonic-aircraft/
article24618690.ece?utm_source=tp-life&utm_medium=sticky_footer

QUESTION 85. MTYyNjA0K1NhaSBLaXJhbitpbGJzbmFhNEBnbWFpbC5jb20rUVVFU1RJT04gODQ=


The ‘8888 Uprising’ is associated with

IASbaba
Web: http://ilp.iasbaba.com/ Score:
Email: ilp@iasbaba.com 0.00 / 200
Page 64
2019 - Test 4-
Exam Title :
History & Cu...
Email : ilbsnaa4@gmail.com
Contact :

a) Democratic Republic of Congo


b) Myanmar
c) Tunisia
d) Sudan
Correct Answer: B
Your Answer: Unanswered
Explanation

Solution (b)

8888 Uprising

· August 8 marks the 30th anniversary of the people’s uprising in Myanmar.

· The ‘8888’ uprising (or the eighth day of August 1988) is one of Myanmar’s most important
historic days in the context of the pro-democracy movement

· It was a series of nationwide protests, marches and civil unrest.

· ‘8888’ was a people’s movement that challenged the then ruling Burma Socialist
Programme Party’s grip on political, economic and social affairs which led the country into
extreme poverty

· The protests began as a student movement and were organised largely by university
students at the Rangoon Arts and Sciences University and the Rangoon Institute of Technology
(RIT).

Source: https://www.thehindu.com/opinion/op-ed/thirty-years-after-the-8888-uprising/
article24617719.ece

QUESTION 86. MTYyNjA0K1NhaSBLaXJhbitpbGJzbmFhNEBnbWFpbC5jb20rUVVFU1RJT04gODU=


Consider the following statements about ‘Lisbon Treaty’

1. It introduced the European Economic Community (EEC)

2. It paved the way for the euro, the common currency

3. It established the Court of Justice of the European Union

Select the correct statements

a) 1 and 2
b) 1 and 3
c) 3 Only
d) None of the above
Correct Answer: D
Your Answer: Unanswered
Explanation

Solution (d)

IASbaba
Web: http://ilp.iasbaba.com/ Score:
Email: ilp@iasbaba.com 0.00 / 200
Page 65
2019 - Test 4-
Exam Title :
History & Cu...
Email : ilbsnaa4@gmail.com
Contact :

Lisbon Treaty

· It updated regulations for the European Union, establishing a more centralized leadership
and foreign policy, a proper process for countries that wish to leave the Union, and a
streamlined process for enacting new policies.

· It was signed by the 27 member states of the European Union and officially took effect in
December of 2009, two years after it was signed.

· It amends two existing treaties, the Treaty of Rome and the Maastricht Treaty.

Treaty of Rome (1957)

· It introduced the European Economic Community (EEC), reduced customs regulations


between member countries, and facilitated a single market for goods and the set of policies for
transporting them.

· It is also known as the Treaty on the Functioning of the European Union (TFEU).

Maastricht Treaty (1992)

· It established the three pillars of the European Union and paved the way for the euro, the
common currency.

· It is also known as the Treaty on European Union (TEU).

Court of Justice of the European Union was formed in 1952.

Source: https://www.thehindu.com/opinion/op-ed/threats-to-eu-values/
article24617328.ece

QUESTION 87. MTYyNjA0K1NhaSBLaXJhbitpbGJzbmFhNEBnbWFpbC5jb20rUVVFU1RJT04gODY=


Which of the following India State shares its border with ‘Rakhine province’ of Myanmar?

a) Nagaland
b) Manipur
c) Mizoram
d) None of the above
Correct Answer: D
Your Answer: Unanswered
Explanation

Solution (d)

Rakhine does not border with India.

IASbaba
Web: http://ilp.iasbaba.com/ Score:
Email: ilp@iasbaba.com 0.00 / 200
Page 66
2019 - Test 4-
Exam Title :
History & Cu...
Email : ilbsnaa4@gmail.com
Contact :

Source: https://www.thehindu.com/opinion/op-ed/thirty-years-after-the-8888-uprising/
article24617719.ece

QUESTION 88. MTYyNjA0K1NhaSBLaXJhbitpbGJzbmFhNEBnbWFpbC5jb20rUVVFU1RJT04gODc=


The ‘Pacific Ring of Fire’ wraps which of the following countries?

1. Canada

2. New Zealand

3. Japan

4. Chile

Select the correct cod:

a) 1, 2 and 3
b) 2, 3 and 4
c) 1, 3 and 4
d) All of the above
Correct Answer: D
Your Answer: Unanswered
Explanation

Solution (d)

IASbaba
Web: http://ilp.iasbaba.com/ Score:
Email: ilp@iasbaba.com 0.00 / 200
Page 67
2019 - Test 4-
Exam Title :
History & Cu...
Email : ilbsnaa4@gmail.com
Contact :

Source: https://www.thehindu.com/todays-paper/tp-international/indonesia-
earthquake-toll-rises-to-98/article24618747.ece

QUESTION 89. MTYyNjA0K1NhaSBLaXJhbitpbGJzbmFhNEBnbWFpbC5jb20rUVVFU1RJT04gODg=


Consider the following statements about ‘Scrub Typhus’

1. It is found only in the tropical regions

2. It is transmitted through mites

3. It is one of the causes for acute encephalitis syndrome

Select the correct statements

a) 1 and 2
b) 2 and 3
c) 1 and 3
d) All of the above
Correct Answer: B
Your Answer: Unanswered

IASbaba
Web: http://ilp.iasbaba.com/ Score:
Email: ilp@iasbaba.com 0.00 / 200
Page 68
2019 - Test 4-
Exam Title :
History & Cu...
Email : ilbsnaa4@gmail.com
Contact :
Explanation

Solution (b)

Scrub typhus

· It is a form of typhus caused by the intracellular parasite Orientia tsutsugamushi.

· It has particularly been shown to be the most common cause of acute encephalitis
syndrome in Bihar and now in UP.

· Scrub typhus is transmitted by some species of trombiculid mites which are found in areas
of heavy scrub vegetation.

· No licensed vaccines are available.

Tsutsugamushi Triangle

· Scrub typhus is endemic to a part of the world known as the tsutsugamushi triangle (after
O. tsutsugamushi).

· This extends from northern Japan and far-eastern Russia in the north, to the territories
around the Solomon Sea into northern Australia in the south, and to Pakistan and Afghanistan
in the west.

· It may also be endemic in parts of South America, too.

Source: https://www.thehindu.com/news/national/scrub-typhus-is-key-encephalitis-
cause-in-eastern-up-study/article24627299.ece

QUESTION 90. MTYyNjA0K1NhaSBLaXJhbitpbGJzbmFhNEBnbWFpbC5jb20rUVVFU1RJT04gODk=


UMANG app is associated with

a) Citizen engagement platform for direct citizen participation in governance


b) Access to the pan-India e-Gov services from the Central, State, Local Bodies, and Agencies
of government
c) Online filing of Income Tax Returns
d) Pilgrimage and Tourism
Correct Answer: B
Your Answer: Unanswered
Explanation

Solution (b)

UMANG (Unified Mobile Application for New-age Governance)

· It is developed by National e-Governance Division (NeGD), MeitY.

· UMANG provides a single platform for all Indian Citizens to access pan India e-Gov
services ranging from Central to Local Government bodies and other citizen centric services.

· It provides a unified approach where citizens can install one application to avail multiple
government services.

IASbaba
Web: http://ilp.iasbaba.com/ Score:
Email: ilp@iasbaba.com 0.00 / 200
Page 69
2019 - Test 4-
Exam Title :
History & Cu...
Email : ilbsnaa4@gmail.com
Contact :

Source: http://pib.nic.in/PressReleseDetail.aspx?PRID=1541941

QUESTION 91. MTYyNjA0K1NhaSBLaXJhbitpbGJzbmFhNEBnbWFpbC5jb20rUVVFU1RJT04gOTA=


Consider the following statements about ‘Pradhan Mantri Suraksha Bima Yojana (PMSBY)’

1. It is available to people between 18 and 50 years of age with bank accounts

2. It is a pure term insurance policy which covers only mortality

Select the correct statements

a) 1 Only
b) 2 Only
c) Both 1 and 2
d) Neither 1 nor 2
Correct Answer: D
Your Answer: Unanswered
Explanation

Solution (d)

Pradhan Mantri Suraksha Bima Yojana (PMSBY)

· It was launched with a view to enhance the level of insurance penetration in the country
and to provide insurance cover to common people especially poor and the Under-privileged
Sections of the society.

· It is available to people between 18 and 70 years of age with bank accounts.

· It has an annual premium of ₹12 exclusive of taxes.

· The GST is exempted on Pradhan Mantri Suraksha Bima Yojana.

· The amount will be automatically debited from the account.

· The accident insurance scheme will have one year cover from June 1 to May 31 and would
be offered through banks and administered through public sector general insurance companies.

Risk Coverage:

· Death - Rs 2 Lakh

· Total and irrecoverable loss of both eyes or loss of use of both hands or feet or loss of sight
of one eye and loss of use of hand or foot - Rs 2 Lakh

· Total and irrecoverable loss of sight of one eye or loss of use of one hand or foot – Rs.1
Lakh.

Source: http://pib.nic.in/PressReleseDetail.aspx?PRID=1541986

IASbaba
Web: http://ilp.iasbaba.com/ Score:
Email: ilp@iasbaba.com 0.00 / 200
Page 70
2019 - Test 4-
Exam Title :
History & Cu...
Email : ilbsnaa4@gmail.com
Contact :
QUESTION 92. MTYyNjA0K1NhaSBLaXJhbitpbGJzbmFhNEBnbWFpbC5jb20rUVVFU1RJT04gOTE=
Under the Foreigners (Restricted Areas) Order, 1963 Restricted Area Permit (RAP) is required
to visit which of the following states?

1. Jammu and Kashmir

2. Nagaland

3. Manipur

Select the correct code:

a) 1 Only
b) 2 and 3
c) All of the above
d) None of the above
Correct Answer: D
Your Answer: Unanswered
Explanation

Solution (d)

Restricted Area Permit (RAP)

Under the Foreigners (Restricted Areas) Order, 1963, the following areas have been declared as
`Restricted’ Areas -

· Andaman & Nicobar Islands - Entire Union Territory

· Sikkim - Part of the State

BUT NOW,

· Union Home Ministry decided to lift restrictions from A&N as the government intends to
boost tourism.

· Foreigners no longer need a Restricted Areas Permit to visit 29 inhabited islands in the
Andaman and Nicobar chain.

THINK!

· Protected Area Permit (PAP)

· Free Movement Regime (FMR)

Source: https://www.thehindu.com/news/national/access-to-andamans-eased/
article24627284.ece

QUESTION 93. MTYyNjA0K1NhaSBLaXJhbitpbGJzbmFhNEBnbWFpbC5jb20rUVVFU1RJT04gOTI=


‘Spodoptera frugiperda’ was in news recently. What is it?

a) An invasive thorny shrub

IASbaba
Web: http://ilp.iasbaba.com/ Score:
Email: ilp@iasbaba.com 0.00 / 200
Page 71
2019 - Test 4-
Exam Title :
History & Cu...
Email : ilbsnaa4@gmail.com
Contact :

b) An invasive agricultural pest


c) A flower which is used in cosmetics
d) None of the above
Correct Answer: B
Your Answer: Unanswered
Explanation

Solution (b)

Fall Armyworm

News: An invasive agricultural pest Fall Armyworm (Spodoptera frugiperda) was discovered in
Karnataka

About

· A major maize pest in North America, the Fall Armyworm arrived in Africa in 2016.

· The Karnataka finding is the first report of the pest in Asia.

· It feeds on around 100 different crops, such as vegetables, rice, and sugarcane.

· They practice cannibalism.

The term "armyworm" can refer to several species, often describing the large-scale invasive
behavior of the species' larval stage.

Source: https://www.thehindu.com/news/national/karnataka/alarm-as-deadly-maize-
pest-seen-in-karnataka/article24636285.ece

QUESTION 94. MTYyNjA0K1NhaSBLaXJhbitpbGJzbmFhNEBnbWFpbC5jb20rUVVFU1RJT04gOTM=


Consider the following statements about Strategic Trade Authorization (STA)

1. It allows a controlled item to be exported under defined conditions without a transaction-


specific license.

2. India is only Asian country to be granted the Strategic Trade Authorisation-1 (STA-1) status
by the US.

Select the correct statement

a) 1 Only
b) 2 Only
c) Both 1 and 2
d) Neither 1 nor 2
Correct Answer: A
Your Answer: Unanswered
Explanation

Solution (a)

IASbaba
Web: http://ilp.iasbaba.com/ Score:
Email: ilp@iasbaba.com 0.00 / 200
Page 72
2019 - Test 4-
Exam Title :
History & Cu...
Email : ilbsnaa4@gmail.com
Contact :

STA

· It allows a controlled item to be exported under defined conditions without a transaction-


specific license. The defined conditions for any license exception allow for an audit trail to
ensure the U.S. Government’s ability to enforce the controls.

· The STA permits the export of a defined set of items on the Commerce Control List to
Allies and certain other friendly countries without a specific license.

· India has been granted the Strategic Trade Authorisation, or STA-1, status that placed it
with a group of 36 countries, mostly NATO allies. Japan and South Korea are the only Asian
countries in this group.

· It would allow the transfer of more sensitive defence technologies and dual use
technologies to India and without the risk of any proliferation

· STA-1 provides India greater supply chain efficiency, both for defence and for other high-
tech products, that will increase activity with US systems, the interoperability of the systems,
and it will reduce time and resources needed to get licensing approved.

Source: https://www.thehindu.com/news/international/gokhale-in-us-ahead-of-22-talks/
article24635755.ece

QUESTION 95. MTYyNjA0K1NhaSBLaXJhbitpbGJzbmFhNEBnbWFpbC5jb20rUVVFU1RJT04gOTQ=


Consider the following statements about ‘Niryat Mitra’

1. It provides wide range of information required to undertake international trade

2. It is developed by the Federation of Indian Export Organisations (FIEO)

3. It works internally to map the ITC HS code of other countries with that of India

Select the correct code:

a) 1 and 2
b) 2 and 3
c) 1 and 3
d) All of the above
Correct Answer: D
Your Answer: Unanswered
Explanation

Solution (d)

Niryat Mitra

· It is launched by the Ministry of Commerce & Industry.

· It is developed by the Federation of Indian Export Organisations (FIEO)

IASbaba
Web: http://ilp.iasbaba.com/ Score:
Email: ilp@iasbaba.com 0.00 / 200
Page 73
2019 - Test 4-
Exam Title :
History & Cu...
Email : ilbsnaa4@gmail.com
Contact :

· It provides wide range of information required to undertake international trade right from
the policy provisions for export and import, applicable GST rate, available export incentives,
tariff, preferential tariff, market access requirements – SPS and TBT measures.

· It works internally to map the ITC HS code of other countries with that of India and
provides all the required data without the users bothering about the HS code of any country.

· ITC (HS) codes are better known as Indian Trade Clarification (ITC) and are based on
Harmonized System (HS) of Coding. It was adopted in India for import-export operations.
Indian custom uses an eight digit ITC (HS) code to suit the national trade requirements.

Source: http://pib.nic.in/PressReleseDetail.aspx?PRID=1542140

QUESTION 96. MTYyNjA0K1NhaSBLaXJhbitpbGJzbmFhNEBnbWFpbC5jb20rUVVFU1RJT04gOTU=


Which of the following is a ‘UNESCO World Heritage Mixed Site’?

a) Khangchendzonga National Park


b) Ajanta Caves
c) Rock Shelters of Bhimbetka
d) Manas Wildlife Sanctuary
Correct Answer: A
Your Answer: Unanswered
Explanation

Solution (a)

Khangchendzonga National Park (KNP) in Sikkim is the only ‘mixed’ heritage site from India.

A ‘mixed site’ exhibits qualities of both natural and cultural significance.

India now has 35 sites, including 27 cultural properties, seven natural sites and one mixed site,
notified as World Heritage Sites.

News: Khangchendzonga Biosphere Reserve has become the 11th Biosphere Reserve from
India that has been included in the UNESCO designated World Network of Biosphere Reserves
(WNBR).

Source: https://timesofindia.indiatimes.com/home/environment/the-good-earth/
kanchenjunga-biosphere-reserve-gets-entry-into-the-unescos-global-list/
articleshow/65327646.cms

QUESTION 97. MTYyNjA0K1NhaSBLaXJhbitpbGJzbmFhNEBnbWFpbC5jb20rUVVFU1RJT04gOTY=


‘Red Corner Notice’ is concerned with which of the following statements?

a) To provide warnings and intelligence about persons who have committed criminal offences
and are likely to repeat these crimes in other countries
b) To collect additional information about a person’s identity, location or activities in relation
to a crime

IASbaba
Web: http://ilp.iasbaba.com/ Score:
Email: ilp@iasbaba.com 0.00 / 200
Page 74
2019 - Test 4-
Exam Title :
History & Cu...
Email : ilbsnaa4@gmail.com
Contact :

c) To seek or provide information on modus operandi, objects, devices and concealment


methods used by criminals
d) To seek the location and arrest of wanted persons with a view to extradition or similar
lawful action
Correct Answer: D
Your Answer: Unanswered
Explanation

Solution (d)

Source: https://www.thehindu.com/news/national/india-hands-over-extradition-request-
for-nirav-modi-to-uk-authorities/article24601882.ece

IASbaba
Web: http://ilp.iasbaba.com/ Score:
Email: ilp@iasbaba.com 0.00 / 200
Page 75
2019 - Test 4-
Exam Title :
History & Cu...
Email : ilbsnaa4@gmail.com
Contact :
QUESTION 98. MTYyNjA0K1NhaSBLaXJhbitpbGJzbmFhNEBnbWFpbC5jb20rUVVFU1RJT04gOTc=
Joint Comprehensive Plan of Action refers to which of the following

a) The Chilcot Report


b) India-Iran Nuclear Deal
c) The Heart of Asia - Istanbul Process
d) None of the above
Correct Answer: D
Your Answer: Unanswered
Explanation

Solution (d)

The Joint Comprehensive Plan of Action (JCPOA) known commonly as the Iran deal, is an
international agreement on the nuclear program of Iran reached in Vienna on 14 July 2015
between Iran, the P5+1 (the five permanent members of the United Nations Security Council—
China, France, Russia, United Kingdom, United States—plus Germany) and the European
Union.

Source: https://www.thehindu.com/news/national/india-iran-pledge-to-maintain-trade-
levels/article24436444.ece

QUESTION 99. MTYyNjA0K1NhaSBLaXJhbitpbGJzbmFhNEBnbWFpbC5jb20rUVVFU1RJT04gOTg=


Which of the following schedules contains provisions for Anti-Defection?

a) Eighth
b) Tenth
c) Sixth
d) Ninth
Correct Answer: B
Your Answer: Unanswered
Explanation

Solution (b)

The 10th Schedule to the Constitution, popularly referred to as the ‘Anti-Defection Law,' was
inserted by the 52nd Amendment in 1985.

QUESTION 100. MTYyNjA0K1NhaSBLaXJhbitpbGJzbmFhNEBnbWFpbC5jb20rUVVFU1RJT04gOTk=


‘Portuguese man-of-war’ was recently in news. It is associated with

a) Blockchain Technology
b) Algorithm Trading
c) Artificial Intelligence
d) None of the above
Correct Answer: D

IASbaba
Web: http://ilp.iasbaba.com/ Score:
Email: ilp@iasbaba.com 0.00 / 200
Page 76
2019 - Test 4-
Exam Title :
History & Cu...
Email : ilbsnaa4@gmail.com
Contact :
Your Answer: Unanswered
Explanation

Solution (d)

Portuguese man-of-war is a jelly-like marine organism. It is commonly known as ‘bluebottle’ or


‘floating terror’.

While most jellyfish stings are harmless to humans and cause only a mild irritation, species like
the bluebottle are venomous and can cause harm on contact. Even a dead bluebottle washed up
on shore can deliver a sting.

Source: https://www.thehindu.com/news/national/other-states/portuguese-man-of-war-
spotted-on-goa-beach/article24604555.ece

IASbaba
Web: http://ilp.iasbaba.com/ Score:
Email: ilp@iasbaba.com 0.00 / 200
Page 77
2019 - Test 4-
Exam Title :
History & Cu...
Email : ilbsnaa4@gmail.com
Contact :
Review in Hindi
QUESTION 1.
मुग़ल�सा�ा�य�के�संदभ��म���न�न�ल�खत�कथन��पर��वचार�क��जए।�

1. औरंगजेब�क��संक�ण��मान�सकता�वाली�नी�तय��को�ही�केवल�मुग़ल�सा�ा�य�के�पतन�का�मु�य�कारण�माना�जाता�है।�

2. बहा�र�शाह�ने��ह���सामंत��और�राजा��के���त�अ�धक�स�ह�णु�रवैया�अपनाया।�

3. बहा�र�शाह�के�शासनकाल�के�दौरान�अ�धक��ह���मं�दर�तोड़े�गए।�

सही�कूट�का�चयन�क��जए:

a) केवल�1 और�2
b) केवल�2 और�3
c) केवल�2
d) केवल�3
Correct Answer: C
Your Answer:
Explanation

Solution (c)

अठारव��शता�द��के�पूवा����के�दौरान�मुग़ल�सा�ा�य�का�पतन�और��वखंडन��आ।�इस�महान�सा�ा�य�के�पतन�क�����या��का�अ�ययन
बताता�है��क�म�यकालीन�भारत�के�सामा�जक, आ�थ�क�और�राजनी�तक�ढ़ांचे�के�कुछ�दोष�और�कमजो�रयां, दे श�क��अं�ेजी�ई�ट�इं�डया�कंपनी
के�हाथ��पराधीनता�हेतु��ज�मेदार�थी।�

औरंगजेब�के�लंबे�और�मजबूत�शासनकाल�के�दौरान�सा�ा�य�क��एकता�और���थरता��हल�गई; तथा�प�उसक��अनेक�हा�नकारक�नी�तय��के
बावजूद, 1707 म��उसक��मृ�यु�के�समय�मुग़ल��शासन�काफ��द��था�और�मुगल�सेना�काफ��मजबूत�थी।�इसके�अ�त�र�, मुगल�वंश�का�दे श
म��अभी�भी�स�मान��कया�जाता�था।�

औरंगजेब�क��मृ�यु�के�प�ात�65 वष�य�बहा�र�शाह�स�ा�म��आया।�उसने�समझौते�और�समाधान�क��नी�त�अपनाई, और�इसम��औरंगजेब�के


�ारा�अपनाई�गई�कुछ�संक�ण��नी�तय��और�उपाय��के�बदलने�का�संकेत��दखाई��दया।�उसने��ह���सरदार��और�राजा��के���त�अ�धक�स�ह�णु
रवैया�अपनाया।�बहा�र�शाह�के�शासनकाल�के�दौरान�कोई��ह���मं�दर�नह��तोडा�गया।�

QUESTION 2.
यूरोपीय�लोग���ारा�भारत�प�ंचने�वाला�समु���माग��खोजे�जाने�का��न�न�ल�खत�म��से�कौन�सा�कारण�सही�माना�जा�सकता�है?

1. यूरोपीय�लोग�भारत�प�ंचने�वाला�एक�वैक��पक�माग��खोजना�चाहते�थे��य��क�लाल�सागर�से�होकर�जाने�वाले�समु���माग��पर�अरब�
का��नयं�ण�था।�

2. वे�पुनजा�गरण�क��भावना�से��भा�वत�थे��जसने�उ�ह��नए�माग��क��खोज�हेतु��ो�सा�हत��कया।�

3. जहाज��नमा�ण�और�नौवहन�म���ए��वकास�ने�उ�ह��पूव��के�अ�ात��थान��तक�प�ँचने�हेतु�साह�सक�समु���या�ाएं�करने�को��ो�सा�हत
�कया।�

सही�कूट�का�चयन�क��जए:

a) केवल�1 और�2
b) केवल�2 और�3
c) केवल�1 और�3
d) उपयु���सभी�
Correct Answer: D
Your Answer:

IASbaba
Web: http://ilp.iasbaba.com/ Score:
Email: ilp@iasbaba.com 0.00 / 200
Page 78
2019 - Test 4-
Exam Title :
History & Cu...
Email : ilbsnaa4@gmail.com
Contact :
Explanation

Solution (d)

सातव��शता�द��म��रोमन�सा�ा�य�के�पतन�के�प�ात, अरब��ने��म���और�फारस�म��अपना�वच��व��था�पत�कर��लया।�यूरोपीय�लोग��और�भारत
के�बीच�सीधे�संपक��म���गरावट�आई�तथा�इसके�साथ�मसाल�, कैलको�(छ�ट�का�कपड़ा), रेशम�और�अनेक�ब�मू�य�र�न��जैसी�भारतीय�व�तु�
(�जनक��भारी�माँग�थी) तक�उनक��आसान�प�ँच��भा�वत��ई।�

1453 म�, कु�तुतू�नया�ऑ�ोमन�तुक��के�अधीन�हो�गया�जो�उभार�पर�थे।�भारत�से�जाने�वाला�सामान�अरब�मु��लम�म�य�थ��से�होकर�यूरोपीय


मं�डय��म��जाता�था।�लाल�सागर�माग��रा�य�का�एका�धकार�हो�गया��जससे�मु��लम�शासक��ने�बड़ी�मा�ा�म��राज�व�क���ा��त�क�।�भारत
तक�जाने�वाले��थलमाग��पर�भी�अरब��का��नयं�ण�था।�इन�प�र��थ�तय��म�, यूरोपीय�लोग�भारत�तक�जाने�वाले�सीधे�समु���माग��क��खोज�को
उ�सुक�थे।�प��व��शता�द��का�यूरोप�पुनजा�गरण�क���गर�त�म��था��जसने�अ�वेषण�का�आ�ान��कया।�ठ�क�उसी�समय, यूरोप�ने�जहाज-
�नमा�ण�और�नौवहन�म��भारी��ग�तयां�क��।�अत:, संपूण��यूरोप�म��पूव��के�अ�ात��थान��तक�प�ँचने�हेतु�साह�सक�समु���या�ाएं�करने
का�उ�साह�था�।

QUESTION 3.
भारत�म��पुत�गा�लय��के�संदभ��म���न�न�ल�खत�कथन��पर��वचार�क��जए।�

1. वा�को�डी�गामा�भारतीय�तट�पर�जहाज��ारा�प�ँचने�वाला�पहला�यूरोपीय�����था।�

2. कालीकट�म��फै����का��नमा�ण�वा�को�डी�गामा��ारा��कया�गया�था।�

3. कालीकट, को��चन�और�क�ूर�भारत�म��पुत�गा�लय��के�शासन�के�समय�मह�वपूण���ापा�रक�क���थे।�

सही�कूट�का�चयन�क��जए:

a) केवल�1 और�2
b) केवल�2 और�3
c) केवल�1 और�3
d) उपयु���सभी�
Correct Answer: B
Your Answer:
Explanation

Solution (b)

पुत�गाली�ना�वक�बथ�लोमेव��डयाज़�ने�1487 म��अ��का�के�आशा�अंतरीप�का�च�कर�लगाया�और�पूव��तट�के�साथ-साथ�आगे�बढ़ा; उसे�पुरा


�व�ास�था��क�लंबे�समय�से��ती�ीत�भारत�क��ओर�जाने�वाला�समु���माग��खोज��लया�गया�है।��क�तु�इसके�दस�वष��प�ात�ही�पुत�गाली�जहाज़�
का�एक�बेड़ा�1497 म��भारत�क��ओर�बढ़ा�और��यारह�से�भी�कम�माह�के�समय�म��मई�1498 म��भारत�प�ँचा।�

वा�को�डी�गामा�1498 म��भारत�प�ँचा।�कालीकट�के��ह���शासक�जमो�रन�(समु�थरी) ने�वा�को�डी�गामा�का��म�वत��वागत��कया।�वा�को


डी�गामा�भारत�म��तीन�माह��का।�जब�वह�पुत�गात�लौटा, तब�अपने�साथ�काफ��साजो-साम�ी�ले�गया�और�उस�सामान�को�यूरोपीय�बाजार�म�
बड़े�मुनाफे�पर�बेचा।�बाद�म�, पे�ो�अ�वारेज�का�ल�ने��सतंबर�1500 म��मसाल��के��ापार�हेतु�समु���या�ा�क�।�उसने�कालीकट�म��एक
फै����क���थापना�क��।

�या�आप�जानते�ह��?

वा�को�डी�गामा�1501 म��एक�बार��फर�भारत�आया।�जब�वा�को�डी�गामा�ने�वा�ण��यक�लालच�को�कड़ी�श�ुता�के�साथ�जोड़ा�और�अरब��के
जहाज़��पर�कहर�ढाया, तब�जमो�रन�ने�अरब��ापा�रय��को�पुत�गा�लय��के�प��म��अपने�से��र�करने�से�इंकार�कर��दया।�इस��कार�जमो�रन�के
साथ�उसके�संबंध��म��पूरी�टू ट�आ�गई।�वा�को�डी�गामा�ने�क�ूर�म��एक��ापा�रक�फै�����था�पत�क�।�धीरे-धीरे, कालीकट, को�ची�और�क�ूर
पुत�गा�लय��के�मह�वपूण���ापा�रक�क���बन�गए�।

QUESTION 4.
�न�न�ल�खत�म��से��कसने��लू�वाटर�नी�त�को�अपनाया?

IASbaba
Web: http://ilp.iasbaba.com/ Score:
Email: ilp@iasbaba.com 0.00 / 200
Page 79
2019 - Test 4-
Exam Title :
History & Cu...
Email : ilbsnaa4@gmail.com
Contact :

a) वा�को�डी�गामा�
b) अ�फ़ा�सो�डी�अलबुकक��
c) �ां�स�को�डी�अ�मीडा�
d) नीनो�डी�कु�हा�
Correct Answer: C
Your Answer:
Explanation

Solution (c)

1505 म�, पुत�गाल�के�राजा�ने�तीन�वष��के�काय�काल�हेतु�भारत�म��एक�गवन�र�के��नयु���क��और�उसे�पुत�गाली��हत��क��र�ा�हेतु�पया��त�बल


मुहैया�करवाए।�

नव�नयु��गवन�र��ां�स�को�डी�अ�मीडा�को�भारत�म��पुत�गा�लय��क����थ�त�को�समे�कत�करने�तथा�अदन, होमु�ज�व�मल�का�पर��नयं�ण
�था�पत�कर�मु��लम��ापार�को�न��करने�को�कहा�गया।�

उसे�अ�जा�दवा, कोचीन, क�ूर�और��कलवा�म���कल��का��नमा�ण�करने�क��भी�सलाह�द��गई।��क�तु�अ�मीडा�को�जमो�रन�का��वरोध�झेलना


पड़ा�और�उसे��म���के�ममलूक�सुलतान�से�खतरा�था।�वे�नस�के��ापा�रय��(�जनका�लाभदायक��ापार�पुत�गा�लय��के�ह�त�ेप�के�चलते�खतरे
म��था) से�उ�सा�हत�होकर��म���वा�सय��ने�पुत�गा�लय��को�रोकने�के��लए�लाल�सागर�म��एक�जहाजी�बेड़ा�तैनात��कया।�

1507 म�, पुत�गा�लय��का�जहाजी�दल��म���वा�सय��और�गुजरात�क��संयु��नौसेना��ारा�द�व�के�तट�से��र�एक�नौसै�नक�लड़ाई�म�


हरा��दया�गया�तथा�अ�मीडा�का�पु��मारा�गया।�अगले�वष�, अ�मीडा�ने�दोन��नौसेना��को�पूण�तया�कुचलकर�अपनी�हार�का�बदला��लया।
अ�मीडा�का�सपना�पुत�गा�लय��को��ह�द�महासागर�का��वामी�बनाने�का�था�। उसक��नी�त�को��लू�वाटर�नी�त�( काट� ज़��स�टम�) के
नाम�से�जाना�जाता�है�।

QUESTION 5.
�न�न�ल�खत�म��से�कौन-से�����को�पूव��म��पुत�गा�लय��का�वा�त�वक�सं�थापक�माना�जाता�है?

a) �ां�स�को�डी�अ�मीडा�
b) अ�फ़ा�सो�डी�अलबुकक��
c) नीनो�डी�कु�हा�
d) वा�को�डी�गामा�
Correct Answer: B
Your Answer:
Explanation

Solution (b)

अलबुकक��, जो�पुत�गाली�गवन�र�के��प�म��अ�मीडा�का�उ�रा�धकारी�था, पूव��म��पुत�गा�लय��क��श���का�वा�त�वक�सं�थापक�था,


तथा�उसने�यह�काय��अपनी�मृ�यु�से�पूव��अंजाम��दया।�उसने�समु��से�आने-जाने�वाले�सभी�माग��पर�अपने�अ�े��था�पत�कर�पुत�गाल�हेतु��ह�द
महासागर�पर��नय��ण�कायम�कर��लया।�

पूव��अ��का, लाल�सागर�से��र, होमु�ज, मालाबार�और�मल�का�म��पुत�गा�लय��क��मजबूत�उप��थ�त�थी।�अलबुकक��के�अधीन�पुत�गा�लय��ने


अ�य�जहाज़��के��लए�पर�मट��व�था�लागू�कर�और��े��के��मुख�जहाज-�नमा�ण�के����पर��नयं�ण�कायम�कर�अपने�मजबूत��ठकान��को�और
बढ़ाया।�खाड़ी�और�लाल�सागर�के��े���म��जहाज-�नमा�ण�हेतु�लकड़ी�क��अनुपल�धता�ने�भी�पुत�गा�लय��को�उनके�उ�े �य��म��मदद�क�।�

अलबुकक��ने�1510 म��बीजापुर�के�सु�तान�से�आसानी�से�गोवा�ह�थया��लया; बीजापुर�के�सु�तान�का�सव��मुख�बंदरगाह�“ एले�ज�डर�महान


के�बाद�से�यूरोपीय��नयं�ण�म��आने�वाला��थम�भारतीय��े��बना�” ।�उसके�शासन�क��सबसे�रोचक�बात�सती��था�क��समा��त�थी।�

QUESTION 6.
�न�न�ल�खत�म��से�कौन�भारत�म��तंबाकू�और�काजू�क��फसल�लेकर�आये?

IASbaba
Web: http://ilp.iasbaba.com/ Score:
Email: ilp@iasbaba.com 0.00 / 200
Page 80
2019 - Test 4-
Exam Title :
History & Cu...
Email : ilbsnaa4@gmail.com
Contact :

a) पुत�गाली�
b) डच�
c) अं�ेज�
d) �ांसीसी�
Correct Answer: A
Your Answer:
Explanation

Solution (a)

पुत�गाली�पु�ष��को�(जो�समु���या�ा�पर�आए�थे�और�भारत�ही�बस�गए�थे) अलबुकक��के�समय�से�ही��थानीय�म�हला��से��ववाह�करने�को
�ो�सा�हत��कया�गया।�

गोवा�तथा�उ�र�के�सूबे�म��उ�ह�ने��वयं�को��ाम�भू�वा�मय��के��प�म��जमा��लया�तथा��ाय: नई�सड़क��और��स�चाई�सु�वधा��का��नमा�ण��कया,
तंबाकू�और�काजू�अथवा�ना�रयल�क��बेहतर��क�म��जैसी�नई�फसल��क��शु�आत�क��तथा�साथ�ही�अपनी�जहाजी�र��सय��क��मांग
को�पूरा�करने�के��लए�ना�रयल�के�पेड़��क��एक�बड़ी�सं�या�म��पौध�लगा��।

गोवा�और�कोचीन�जैसे�शहर��म�, वे��ापारी�होने�के�साथ-साथ�द�तकार��और��श��पय��के��प�म��बस�गए।�

QUESTION 7.
भारत�म��अं�ेज��के�संदभ��म���न�न�ल�खत�कथन��पर��वचार�क��जए।�

1. अं�ेज��को�अपनी�पहली�फै����सूरत�म���था�पत�करने�हेतु�जहाँगीर�ने�अनुम�त��दान�क��थी।�

2. उ�ह��जहाँगीर��ारा�भारतीय�बंदरगाह��पर��न:शु�क��ापार�करने�का�सुनहरा�फरमान(golden Farman) �दया�गया�था।�

सही�कूट�का�चयन�क��जए:

a) केवल�1
b) केवल�2
c) 1 और�2 दोन��
d) उपरो��म��से�कोई�नह��
Correct Answer: A
Your Answer:
Explanation

Solution (c)

31 �दसंबर�1600 को�रानी�ए�लज़ाबेथ��थम�ने�‘ गवन�र�एंड�कंपनी�ऑफ़�मच�ट्स�ऑफ़�लंदन��े �ड�ग�इनटू �द�ई�ट�इंडीज�’ नामक


कंपनी�को��व�श���ापार�अ�धकार��वाला�एक�चाट� र�जारी��कया।�आरंभ�म�, प��ह�वष��का�एका�धकार��दया�गया�था��जसे�मई�1609 म��एक
नया�चाट� र�जारी�कर�अ�न��त�काल�के��लए�बढ़ा��दया�गया।�कै�टे न�हॉ�क�स�अ�ैल�1609 म���वयं�जहाँगीर�के�दरबार�म��प�ँचा।��क�तु
पुत�गा�लय��के��वरोध�के�कारण�सूरत�म��फै�����था�पत�करने�का�मंसूबा�सफल�न�हो�सका�और�हॉ�क�स�नवंबर�1611 म��आगरा�छोड़कर�चला
गया।�

1611 म�, अं�ेज��ने�भारत�के�द��ण-पूव��तट�मसुलीप�नम�पर��ापार�आरंभ�कर��दया�था�और�आगे�चलकर�1616 म��वहाँ�फै���


�था�पत�क��।

1612 म��जाकर�कै�टे न�थॉमस�बे�ट�ने�पुत�गा�लय��को�सूरत�के�तट�से��र�हराया; इससे��भा�वत�होकर�जहाँगीर�ने�अं�ेज��को�1613 म�


सूरत�म��थॉमस�अ�डवथ��के�अधीन�फै�����था�पत�करने�क��अनुम�त��दान�क��।�1632 म��गोलकुंडा�के�सु�तान��ारा�जारी��कए�गए�सु
नहरे�फरमान��ारा�कंपनी�क����थ�त�और�भी�मजबूत�हो�गई�।�500 पैगोडा���त�वष��के�भुगतान�पर�उ�ह�ने�गोलकुंडा�के�बंदरगाह��पर�मु�
�ापर�का��वशेषा�धकार�हा�सल�कर��लया।�

�या�आप�जानते�ह��?

IASbaba
Web: http://ilp.iasbaba.com/ Score:
Email: ilp@iasbaba.com 0.00 / 200
Page 81
2019 - Test 4-
Exam Title :
History & Cu...
Email : ilbsnaa4@gmail.com
Contact :

मसुलीप�नम�प�रषद�के�एक�सद�य����टश��ापारी��ां�सस�डे�को�1639 म��चाँद�गरी�के�शासक�से�म�ास�म��एक��कलेबंद�फै����के��नमा�ण
क��अनुम�त��ा�त��ई�जो�आगे�चलकर�फोट� �स�ट�जॉज��बना�और�इसने�द��ण�भारत�म��अं�ेज�ब��तय��के�मु�यालय�के�तौर�पर
मसुलीप�नम�का��थान��लया।�

QUESTION 8.
�न�न�ल�खत�म��से�कौन-सी�अं�ेजी�कंपनी�के�मै�ना�काटा��क��मु�य�शत��थी?

1. कंपनी�को�व�तु��के�प�रवहन�के��लए�द�तक�जारी�करने�क��अनुम�त�थी।�

2. कंपनी�को�कलक�ा�के�आसपास�अ�धक�जमीन��कराए�पर�दे ने�क��अनुम�त�थी।�

3. बॉ�बे�म��ढाले�गए�कंपनी�के��स�क��का�संपूण��मुग़ल�सा�ा�य�म��मु�ा�के��प�म���चलन।�

सही�कूट�का�चयन�क��जए:

a) केवल�1 और�2
b) केवल�2 और�3
c) केवल�1 और�3
d) उपयु���सभी�
Correct Answer: D
Your Answer:
Explanation

Solution (d)

1715 म�, जॉन�सम�न�के�नेतृ�व�म��एक��मशन�मुग़ल�बादशाह�फ�� ��सयर�के�दरबार�म��प�ँचा�और�तीन���स��फरमान�हा�सल��कए, �जनसे


कंपनी�को�बंगाल, गुजरात�और�हैदराबाद�म��अनेक�मू�यवान��वशेषा�धकार��ा�त��ए।��ा�त��कए�गए�इन�फरमान��को�कंपनी�के�मै�ना�काटा��क�
सं�ा�द��गई।�इनक��मह�वपूण��शत��थी—

1. बंगाल�म�, कंपनी�के�आयात��और��नया�त��को�अ�त�त��सीमा�शु�क��से�छू ट�द��गई�थी��जनका�अनुमा�नत�वा�ष�क�भुगतान�3,000


�पए�(जैसा��क�पहले�तय��आ�था) बनता�था।�
2. कंपनी�को�ऐसी�व�तु��के�प�रवहन�के��लए�द�तक�जारी�करने�क��अनुम�त�थी।�
3. कंपनी�को�कलक�ा�के�आसपास�अ�धक�जमीन���कराए�पर�दे ने�क��अनुम�त�थी।�
4. हैदराबाद�म�, कंपनी�ने�अपने�शु�क�मु���ापार�क���वतं�ता�के�वत�मान�अ�धकार�बरक़रार�रखे�और�उसे�केवल�म�ास�को��च�लत
�कराए�का�भुगतान�करना�था।�
5. सूरत�म�, 10,000 �पए�के�वा�ष�क�भुगतान�के�बदले�ई�ट�इं�डया�कंपनी�को�सभी�शु�क��से�छू ट�दे �द��गई�थी।�
6. यह�आदे श�जारी��कया�गया��क�बॉ�बे�म��ढाले�गए�कंपनी�के��स�क��को�संपूण��मुग़ल�सा�ा�य�क��मु�ा�माना�जाएगा।�

QUESTION 9.
कना�टक�यु���के�संदभ��म���न�न�ल�खत�कथन��पर��वचार�क��जए।�

1. सभी�कना�टक�यु�, यूरोप�म��ऑ���या�के�उ�रा�धकार�क��लड़ाई�के�कारण�हो�रही�आं�ल-�ांसीसी���त�पधा��का��व�तार�थे।�

2. ए�स�ला�चैपल�सं�ध���तीय�कना�टक�यु��से�संबं�धत�है।�

3. वां�डवाश�क��लड़ाई�तृतीय�कना�टक�यु��से�संबं�धत�है।�

सही�कूट�का�चयन�क��जए:

a) केवल�1 और�2
b) केवल�2 और�3
c) केवल�3

IASbaba
Web: http://ilp.iasbaba.com/ Score:
Email: ilp@iasbaba.com 0.00 / 200
Page 82
2019 - Test 4-
Exam Title :
History & Cu...
Email : ilbsnaa4@gmail.com
Contact :

d) केवल�1 और�3
Correct Answer: C
Your Answer:
Explanation

Solution (c)

कना�टक�यूरोपीय�लोग���ारा�कोरोमंडल�तट�और�इसके�पृ���दे श��को��दया�गया�नाम�था�। �थम�कना�टक�यु��(1740-1748) यू


रोप�म��ऑ���या�के�उ�रा�धकार�क��लड़ाई�के�कारण�हो�रही�आं�ल-�ांसीसी���त�पधा��का��व�तार�था�।

�थम�कना�टक�यु��1748 म��उस�समय�समा�त��आ�जब�ए�स�ला�चैपल�सं�ध�पर�ह�ता�र�कर�ऑ���या�के�उ�रा�धकार�क��लड़ाई�को
समा�त��कया�गया।�सं�ध�क��शत��के�अनुसार, म�ास�को�अं�ेज��को�पुन: स�प��दया�गया�तथा��ांसी�सय��को�बदले�म��उ�री�अमे�रका�म��उनके
�े��वापस��मले।�

��तीय�कना�टक�यु��(1749-1754)

��तीय�कना�टक�यु��क��पृ�भू�म�भारत�म��हो�रही���त�पधा��से��ा�त��ई�।��ांसीसी�गवन�र�डु �ले��जसने��थम�कना�टक�यु��म���ांसीसी
बल��का�सफल�नेतृ�व��कया�था, ने�अं�ेज��को�हराने�के��लए��थानीय�वंश��के��ववाद��म��ह�त�ेप�के�मा�यम�से�द��ण�भारत�म��अपनी�श��
और��ांसीसी��भाव�को�बढ़ाने�क��को�शश�क�।�

डु �ले�क��नी�त�से�होने�वाले�भारी��व�ीय�नुकसान��से��च��तत�हो��ांसीसी�अ�धका�रय��ने�डु �ले�को�1754 म��वापस�बुला��लया।��ांसीसी�गवन�र-


जनरल�के�तौर�पर�भारत�म��डु �ले�का��थान�गोडे��ने��लया।�गोडे��ने�अं�ेज��के�साथ�वाता���क��नी�त�अपनाई�और�उनके�साथ�एक�सं�ध
कर�ली�। अं�ेज�और��ांसीसी��थानीय�राजकुमार��के�झगड़��म��ह�त�ेप�न�करने�पर�सहमत��ए।�साथ�ही, दोन��प���का�सं�ध�के�समय�उनके
अ�धकार�म��रहे��े���पर�अ�धकार�बना�रहने��दया�गया।�

तृतीय�कना�टक�यु��(1758-1763)

यूरोप�म�, जब�ऑ���या�1756 म���सले�सया��ा�त�करना�चाहता�था, स�त�वष�य�यु��(1756-63) आरंभ�हो�गया।���टे न�और��ांस�एक�बार


�फर�एक-�सरे�के�आमने-सामने�थे।�1758 म�, काउंट�डी�लाली�के�अधीन��ांसीसी�सेना�ने�स�ट�डे�वड�और��वजयनगरम�के�अं�ेजी��कल��पर
1758 म��अ�धकार�कर��लया।�अब, अं�ेज�आ�ामक�हो�गए�और�उ�ह�ने�एड�मरल�डी’आचे�के�अधीन�मसुलीप�नम�म����ांसीसी�बेड़े�को�महान
��त�प�ंचाई।�

वां�डवाश�क��लड़ाई: तीसरे�कना�टक�यु��क��सबसे��नणा�यक�लड़ाई�अं�ेज��ने�22 जनवरी�1760 को�त�मलनाडु �के�वां�डवाश�म�


जीती।�अं�ेज�जनरल�आयर�कूट�ने�आथ�र�डी�लाली�के�नेतृ�व�वाली��ांसीसी�सेना�को�पूण�त: परा�जत�कर��दया�और�ब�सी�को�बंद��बना��लया।�

QUESTION 10.
�न�न�म��से�भारत�म���ांसी�सय��पर�अं�ेजी��भु�व�का�कारण�कहा�जा�सकता�है?

1. �नजी�उ�म�होने�के�कारण�अं�ेजी�कंपनी��ांसीसी�कंपनी�क��अपे�ा�ती���नण�य�ले�सकती�थी��य��क��ांसीसी�कंपनी��ांसीसी�सरकार
�ारा��नयं��त�और��व�नय�मत�क��जाती�थी।�

2. अं�ेजी�नौसेना, �ांसीसी�नौसेना�से��े��थी।�

3. अं�ेज��ने�अपनी��े�ीय�मह�वाकां�ा�(territorial ambition) को�अपने�वा�ण��यक��हत��पर�वरीयता�द���जससे�अं�ेजी


कंपनी�के�पास�धन�क��कमी�हो�गई।�

सही�कूट�का�चयन�क��जए:

a) केवल�1 और�2
b) केवल�2 और�3
c) केवल�1 और�3
d) उपयु���सभी�
Correct Answer: A
Your Answer:

IASbaba
Web: http://ilp.iasbaba.com/ Score:
Email: ilp@iasbaba.com 0.00 / 200
Page 83
2019 - Test 4-
Exam Title :
History & Cu...
Email : ilbsnaa4@gmail.com
Contact :
Explanation

Solution (a)

अं�ेज�कंपनी�एक��नजी�उ�म�थी—इसने�लोग��के�बीच�एक�उ�साह�और�आ�म�व�ास�क��भावना�पैदा�क�।�सरकार�के�कम��नय��ण�के�चलते,
यह�कंपनी�आव�यकता�पड़ने�पर�सरकार�के�अनुमोदन�क���ती�ा��कए��बना�ती�ता�से��नण�य�ले�सकती�थी।��सरी�ओर��ांसीसी�कंपनी�एक
सरकारी�कंपनी�थी।�इसका��नयं�ण�और��व�नयमन��ांस�क��सरकार�के�हाथ�म��था�यह�सरकारी�नी�तय��और��नण�य-�नमा�ण�म���वलंब��से��घरी
थी।�

अं�ेजी�नौसेना, �ांसीसी�नौसेना�से��े��थी।; इसने�भारत�म���ांस�के��नय��ण�वाले��थान��का��ांस�से�स�पक��काटने�म��मदद�क�।�

अं�ेज��के�पास�तीन�मह�वपूण���थान�थे�नामत: कलक�ा, बॉ�बे�और�म�ास�जब�क��ांसी�सय��के�पास�केवल�पां�डचेरी�था।�

�ांसी�सय��ने�अपने�वा�ण��यक��हत��को�अपने��े�गत��हत��के�अधीन�कर��दया, �जससे��ांसीसी�कंपनी�के�पास�कोष��क��कमी�हो�गई।�

अपने�सा�ा�यवाद��इराद��के�बावजूद, अं�ेज��ने�कभी�भी�अपने�वा�ण��यक��हत��क��अनदे खी�नह��क�।�इस�लए�उनके�पास�हमेशा�कोष


उपल�ध�थे�और�इस��कार�उनक��बेहतर��व�ीय���थ�त�ने�उ�ह��उनके��वरो�धय��के��व���यु��म���नणा�यक�मदद�द�।�

भारत�म��अं�ेज��क��सफलता�का�एक�अ�य��मुख�कारक�उनके�पास�बेहतर�कमांडर��का�होना�था।�सर�आयर�कूट, मेजर����ं गर�लॉर�स, रॉबट�


�लाइव�और�अ�य�अनेक�कमांडर��क��लंबी�सूची�के�मुकाबले��ांसी�सय��के�पास�केवल�डु �ले�था।�

QUESTION 11.
�न�न�ल�खत�को�सही�काल�मानुसार��व��थत�क��जए।�

1. वा�को�डी�गामा��ारा�भारत�प�ंचने�वाले�समु���माग��क��खोज।�

2. इं��लश�ई�ट�इं�डया�कंपनी�क���थापना।�

3. ई�ट�इं�डया�कंपनी�का�मै�ना�काटा�।�

4. पे�रस�क��सं�ध।�

सही�कूट�का�चयन�क��जए:

a) 1-2-3-4
b) 1-2-4-3
c) 2-1-3-4
d) 2-1-4-3
Correct Answer: A
Your Answer:
Explanation

Solution (a)

वा�को�डी�गामा��ारा�1498 म��भारत�क��ओर�जाने�वाले�समु���माग��क��खोज�क��गई।�

वा�को�डी�गामा�क��1502 म���ई��सरी�भारत�या�ा�का�प�रणाम�कालीकट, कोचीन�और�क�ूर�म���ापा�रक�के����क���थापना�के��प�म�


सामने�आया।�

इं��लश�ई�ट�इं�डया�कंपनी�क���थापना�31 �दसंबर�1600 को�रानी�ए�लज़ाबेथ�जारी�एक�चाट� र�के�ज�रए��ई��जसने�क�पनी�को�15


वष��तक�ई�ट�इंडीज�म���ापार�का�एका�धकार��दान��कया।�

1717 म��, मुगल�स�ाट�फ�� ��सयर�के�फरमान��ज�ह��ई�ट�इं�डया�कंपनी�का�मै�ना�काटा��कहा�जाता�है, ने�कंपनी�को�बंगाल, गुजरात


और�हैदराबाद�म��मह�वपूण���वशेषा�धकार��दान��कए।�

IASbaba
Web: http://ilp.iasbaba.com/ Score:
Email: ilp@iasbaba.com 0.00 / 200
Page 84
2019 - Test 4-
Exam Title :
History & Cu...
Email : ilbsnaa4@gmail.com
Contact :

पे�रस�क��सं�ध�(1763 ) के��ारा��ांसी�सय��को�भारतीय�ब��तय��के�केवल�वा�ण��यक��योग�क��अनुम�त�द��गई�और�इन�ब��तय��क�
�कलेबंद��को���तबं�धत�कर��दया�गया।�

QUESTION 12.
�न�न�ल�खत�यु�म��का�सही�सु�मेलन�क��जए�

घटनाएं� �ा�या�

1. बीदर�क��लड़ाई� A) �ांसीसी�सेना�पर����टश�सेना�क���वजय�

2. वां�डवाश�क��लड़ाई� B) अं�ेज��ने�डच��को�परा�जत��कया�

3. फोट� �स�ट�जॉज�� C) म�ास�पुन: अं�ेज��को�स�पा�गया�

4. ए�स�ला�चैपल�सं�ध� D) पूव��तट�पर�मसुलीप�नम�अं�ेजी�मु�यालय�नह��रहा�

सही�कूट�का�चयन�क��जए:

a) A-1 B-2 C-3 D-4


b) A-2 B-1 C-4 D-3
c) A-3 B-2 C-4 D-1
d) A-4 B-2 C-3 D-1
Correct Answer: B
Your Answer:
Explanation

Solution (b)

�वत: �प��

QUESTION 13.
पानीपत�म��अनेक�यु��होने�के�उपयु���या�कारण�थे?

1. पानीपत�के�चार��ओर�का�भूभाग�समतल�है�जो�घुड़सवार�सेना�(जो�उस�समय�यु��क��मु�य�प��त��आ�करती�थी) से�यु��करने�हेतु
उपयु��था।�

2. इस��े��म��मानसून�क��अव�ध�अ�य��े���क��अपे�ा�लघु�है, �जससे�यु��करना�आसान�था।�

3. �द�ली�से�इसक���नकटता�के�चलते�भारतीय�राजा��के��लए�ह�थयार, सेना�और�खा��साम�ी�क��आप�त��पानीपत�तक�प�ँचाना
आसान�था।�

सही�कूट�का�चयन�क��जए:

a) केवल�1 और�2
b) केवल�2 और�3
c) केवल�1 और�3
d) उपयु���सभी�
Correct Answer: D
Your Answer:
Explanation

IASbaba
Web: http://ilp.iasbaba.com/ Score:
Email: ilp@iasbaba.com 0.00 / 200
Page 85
2019 - Test 4-
Exam Title :
History & Cu...
Email : ilbsnaa4@gmail.com
Contact :

Solution (d)

�य��पानीपत�एक�पसंद�दा�यु���े��था�?

पानीपत�क��अव��थ�त�साम�रक�है।�यु��के�दोन��प���म��से�कोई�एक�प��सामा�यत: उ�र/उ�र-प��म�से�खैबर�दर��से�होकर��द�ली�पर
�नयं�ण��था�पत�करने�के�इरादे �से�आया�करता�था, जो��क�उस�समय�उ�र�भारत�क��राजनी�तक�राजधानी�थी।�

राज�थान�के�म��थल�और�घने�जंगल��से�भरे�उ�र�के�अ�य��े���से�सेना�को�लेकर�आना�ब�त�ही�जो�खम�भरा�और�क�ठन�था।��सरी�ओर,
�द�ली�के�शासक�पानीपत�को�एक�सु�वधाजनक�साम�रक�मैदान�मानते�थे, अत: उ�ह�ने�वहाँ�यु��करना�पसंद��कया।�

1. �द�ली�से�इसक���नकटता�के�चलते�भारतीय�राजा��के��लए�ह�थयार, सेना�और�भोजन�साम�ी�का�प�रवहन�यहाँ�तक�करना�सरल
रहता�था।�साथ�ही�इससे�राजधानी�को���तुत�यु��से�बचाए�रखा�जा�सकता�था।�
2. पानीपत�के�चार��ओर�का��े��समतल�भूभाग�है�जो�घुड़सवार�सेना�(जो�उस�समय�यु��क��मु�य�प��त��आ�करती�थी) के�चलने�हेतु
उपयु��था।�
3. शेर�शाह�सूरी�(1540-45) �ारा���ड��ं क�रोड�के��नमा�ण�के�प�ात�पानीपत�इसी�माग��पर�पड़ता�था।�इससे��वजेता��के��लए�इसे
रा�ता�बनाना�सरल�था।�
4. इस��े��म��मानसून�क��अव�ध�अ�य��े���क��अपे�ा�छोट��है, �जससे�यु��करना�सरल�रहता�है।�
5. इन��े���के�द�तकार/लौहार�यु��से�जुड़ी�साम�ी�के��नमा�ण�म��कुशल�थे�और�इस��कार�दोन��प���क��सेना��के��लए�अपनी�यु�
साम�ी�क��आपू�त��करना�सरल�था।�

QUESTION 14.
मराठा�शासन�म��कर��णाली�के�संदभ��म���न�न�ल�खत�कथन��पर��वचार�क��जए।�

1. चौथ�उन�भू�मय��के�राज�व�अथवा�उपज�पर�लगाया�जाने�वाला�25% वा�ष�क�कर�था�जो�मुगल�शासन�के�अधीन�थी।�

2. सरदे शमुखी, चौथ�के�ऊपर�लगने�वाला�10% अ�त�र��कर�था�जो�नजराने�के��प�म��राजा�को��दया�जाता�था।�

सही�कूट�का�चयन�क��जए:

a) केवल�1
b) केवल�2
c) 1 और�2 दोन��
d) उपरो��म��से�कोई�नह��
Correct Answer: C
Your Answer:
Explanation

Solution (c)

चौथ�एक��नय�मत�कर�अथवा�नजराना�था�जो�भारत�म��मराठा�सा�ा�य��ारा�18 व��सद��क��शु�आत�से�लगाया�जा�रहा�था।�यह�राज�व
अथवा�उपज�पर�लगाया�जाने�वाला�25% वा�ष�क�कर�था�और�इसी�से�इसका�यह�नाम�पड़ा।�यह�उन�भू�मय��पर�लगाया�जाता�था
जो�मुगल�शासन�के�अधीन�थी।�सरदे शमुखी�चौथ�के�ऊपर�लगने�वाला�10% अ�त�र��कर�था�।�यह�नजराने�के��प�म��राजा�को��दया
जाता�था।�

चौथ�के�काय��पर�अलग-अलग�राय�मौजूद�ह�।�एम�जी�रानाडे�के�अनुसार, चौथ�मराठा�रा�य�क��सेना��ारा�सुर�ा�उपल�ध�कराने�के�नाम�पर�ली
जाती�थी�और�इस��कार�यह�लॉड��वै�सली��ारा�भारतीय�रा�य��को����टश��नयं�ण�म��लाने�के��लए�क��गई�सहायक�सं�ध�के�सामान�थी।�

�या�आप�जानते�ह��?

�शवाजी�ने�सव��थम�1665 म��चौथ�क��मांग�क��और�बीजापुर�और�गोलकुंडा�क��द�कन�स�तनत��ने�उसे�1668 म��औरंगजेब��ारा��शवाजी


को�राजा�बनाए�जाने�के�प�ात�8, 00,000 �पए�क��संयु��रा�श�का�भुगतान�करना�आरंभ�कर��दया।�

IASbaba
Web: http://ilp.iasbaba.com/ Score:
Email: ilp@iasbaba.com 0.00 / 200
Page 86
2019 - Test 4-
Exam Title :
History & Cu...
Email : ilbsnaa4@gmail.com
Contact :

1719 म�, मुगल�बादशाह�ने�शा��को�द�कन�के�छ: �ा�त��पर�चौथ�और�सरदे शमुखी�अ�धकार��दान�कर��दए।�इसके�बदले�उसे�बादशाह�हेतु


15,000 सै�नक��का�ज�था�रखना�था।�चौथ�से��मलने�वाले�राज�व�को�चार�भाग��म���वभा�जत��कया�जाता�था�जो�मराठा�शासन�के��व�भ�
पदा�धका�रय��को�जाता�था।�

QUESTION 15.
मुगल�सा�ा�य�के�पतन�के�प�रणाम�व�प�उभरे�रा�य��के�संदभ��म���न�न�ल�खत�कथन��पर��वचार�क��जए।�

1. उ�रा�धकारी�रा�य�वे�मुग़ल�सूबे�थे�जो�सा�ा�य�से�पृथ�क�होने�के�प�ात�रा�य�बन�गए।�

2. नए�रा�य�मुगल��ा�त��पर�सा�ा�य�का��नयं�ण�अ��थर�हो�जाने�से�अ��त�व�म��आए।�

3. �वतं��रा�य�वे�रा�य�थे��जनक���थापना�मुग़ल�सा�ा�य�के��व�ो�हय���ारा�क��गई�थी।�

सही�कूट�का�चयन�क��जए:

a) केवल�1 और�2
b) केवल�2 और�3
c) केवल�1 और�3
d) केवल�1
Correct Answer: D
Your Answer:
Explanation

Solution (d)

मुगल�सा�ा�य�के�पतन�के�प�रणाम�व�प�उभरे�रा�य��को��न�न�ल�खत�तीन�बड़ी��े�णय��म���वभा�जत��कया�जा�सकता�है:

उ�रा�धकारी�रा�य�: ये�वे�मुगल�सूबे�थे�जो�सा�ा�य�से�अलग�होने�के�प�ात�रा�य�बने।�हालां�क�उ�ह�ने�मुगल�सं�भुता�को�चुनौती�नह��द�,
�क�तु�इनके�सूबेदार���ारा�लगभग��वतं��और�वंशानुगत��ा�धकार�क���थापना�ने�इन��े���म���वायत�राज�व�था�के�उभरने�को�दशा�या।�अवध,
बंगाल�और�हैदराबाद�इसके�कुछ�उदाहरण�ह��।

�वतं��सा�ा�य�: ये�रा�य��मुखतया�सूब��पर�मुगल��नयं�ण�कमजोर�पड़�जाने�के�कारण�अ��त�व�म��आए।�मैसूर, केरल�और�राजपूत�रा�य�


इसके�उदाहरण�ह��।

नए�रा�य�: इन�रा�य��क���थापना�मुगल��के��व����व�ोह�करने�वाल���ारा�क��गई�थी।�मराठा, �सख�और�जाट�रा�य�इसके�उदाहरण�ह�।�

QUESTION 16.
�न�न�ल�खत�रा�य��और�उनके�सं�थापक��को�सुमे�लत�क��जए-

रा�य� सं�थापक�

1. हैदराबाद� A) मात�ड�वमा��

2. केरल� B) मु�श�द�कुली�खां�

3. बंगाल� C) बुरहान-उल-मु�क�

4. अवध� D) �नजाम-उल-मु�क�

सही�कूट�का�चयन�क��जए:

IASbaba
Web: http://ilp.iasbaba.com/ Score:
Email: ilp@iasbaba.com 0.00 / 200
Page 87
2019 - Test 4-
Exam Title :
History & Cu...
Email : ilbsnaa4@gmail.com
Contact :

a) A-2 B-3 C-4 D-1


b) A-3 B-1 C-4 D-2
c) A-2 B-1 C-4 D-3
d) A-2 B-1 C-3 D-4
Correct Answer: A
Your Answer:
Explanation

Solution (a)

हैदराबाद�के�आसफ-जाह�घराने�का�सं�थापक��चन�कलीच�खां�था��जसे��नजाम-उल-मु�क�के�नाम�से�जाना�जाता�है।�

अवध�के��वतं��शाही�घराने�का�सं�थापक�सादत�खान�था��जसे�बुरहान-उल-मु�क�के�नाम�से�भी�जानते�ह��।�सादत�खान�एक��शया�था।
उसने�सैयद�बंधु��के��खलाफ��ए�षडयं��म��भाग��लया�था��जसके�प�रणाम�व�प�उसे�बढ़ा��आ�मनसब��मला�था।�बाद�म�, दरबार�से��नकाले
जाने�के�बाद, वह��वतं��रा�य�क���थापना�को��े�रत��आ।�

मु�श�द�कुली�खां�बंगाल�के��वतं��रा�य�का�सं�थापक�था�।�वह�एक��मतावान�शासक�था�और�उसने�बंगाल�को�एक�समृ��रा�य�बनाया।
1727 म��उसका�पु��शुजाउ��न�उसका�उ�रा�धकारी�बना।�उसका�उ�रा�धकारी�सरफराज�खान��बहार�और�घे�रया�के�नायब�सूबेदार�अलीवद�
खान��ारा�1740 म��मार��दया�गया।�इस��कार�स�ा�ह�थयाने�के�प�ात�उसने�वा�ष�क�नजराना�दे कर��वयं�को�मुगल�स�ाट�से��वतं��बना��लया।

मात�ड�वमा��ने�केरल�को��ावणकोर�राजधानी�के�साथ�एक��वतं��रा�य�बना��लया�।�उसने�अपने�रा�य�क��सीमा��का�क�याकुमारी�से
लेकर�कोचीन�तक��व�तार��कया।�उसने�प��मी�प��त�से�अपनी�सेना�का�संगठन��कया�और�अपने�रा�य�को��वक�सत�बनाने�के�अनेक�उपाय
अपनाए।�

QUESTION 17.
�न�न�ल�खत�म��से�कौन-से�मुगल�सा�ा�य�के�पतन�के�उपयु��कारण�माने�जा�सकते�ह�?

1. औरंगजेब�क��धा�म�क�और�द�कन�नी�त�ने�अनेक�शासक��को�नाराज�कर��दया।�

2. अंतहीन�यु�, कृ�ष�म��ठहराव�और��ापार�व�उ�ोग��म���गरावट�ने�शाही�खजाने�को�खाली�कर��दया।�

3. कमजोर�शासक��के��लए��वशाल�सा�ा�य�का�कुशलतापूव�क�संचालन�क�ठन�हो�गया�था।�

सही�कूट�का�चयन�क��जए:

a) केवल�1 और�2
b) केवल�2 और�3
c) केवल�1 और�3
d) उपयु���सभी�
Correct Answer: D
Your Answer:
Explanation

Solution (d)

मुगल�सा�ा�य�के�पतन�के�कारण-

कमजोर�उ�रा�धकारी�- मुगल�सा�ा�य�एक����गत��नरंकुशतावाद��शासन�था�और�इसक��सफलता�इसके�मजबूत�और�यो�य�राजा��पर
�नभ�र�थी।�

उ�रा�धकार�के��न��त��नयम�क��अनुप��थ�त�- उ�रा�धकार�के��लए�लड़ी�जाने�वाली��नरंतर�लड़ाइय��(�ये�ा�धकार�क��अनुप��थ�त) ने
दे शभ���क��क�मत�पर�साझेदारी�को��ो�सा�हत��कया।�

IASbaba
Web: http://ilp.iasbaba.com/ Score:
Email: ilp@iasbaba.com 0.00 / 200
Page 88
2019 - Test 4-
Exam Title :
History & Cu...
Email : ilbsnaa4@gmail.com
Contact :

औरंगजेब�क��धा�म�क�और�द�कन�नी�त�– धा�म�क�नी�त�ने�राजपूत�, �सख�, जाट��और�मराठ��को�नाराज�कर��दया; द�कन�नी�त�ने�स�ाट


को�लंबे�समय�तक�राजधानी�से��र�रखा।�

शासक��और�सामंत��का�पतन�

सेना�म���गरावट�

सा�ा�य�का�काफ��बड़ा�होना�- कमजोर�शासक��के��लए�इतने�बड़े�सा�ा�य�को�कुशलतापूव�क�चलाना�क�ठन�हो�गया�था।�

बाहरी�हमले�- ईरानी�और��रा�नी�रा�य��(ना�दर�शाह, अहमद�शाह�अ�दाली) के�हमल��ने�भयंकर�तबाही�मचाई।�

आ�थ�क��गरावट�- अंतहीन�यु�, कृ�ष�म��ठहराव�और��ापार�व�उ�ोग��म���गरावट�ने�शाही�खजाने�को�खाली�कर��दया।�

यूरोपीय�लोग��का�आगमन�- यूरोपीय�कंप�नय��ने��थानीय�राजनी�त�म��ह�त�ेप��कया��जससे�सा�ा�य�के�पतन�द��दर�तेज�हो�गई।�

जम�दार��क���न�ा��म��प�रवत�न�

जागीरदारी�संकट�

�े�ीय�मह�वाकां�ा��का�उभरना�- अवध, बंगाल, हैदराबाद, मैसूर, केरल, राजपूत�रा�य��और�जाट�रा�य��के�उभरने�ने��वखंडन�क�


���या�को�तेज�कर��दया।�

QUESTION 18.
�न�न�ल�खत�म��से�आधु�नक�भारत�के�इ�तहास�क����स���लैक�होल(black hole) �घ�टना�से�संबं�धत�है?

a) �सराज-उद-दौला�
b) शुजा-उद-दौला�
c) मीर�जाफर�
d) रॉबट� ��लाइव�
Correct Answer: A
Your Answer:
Explanation

Solution (a)

‘ �लैक�होल��घ�टना�’ - माना�जाता�है��क��सराज-उद-दौला�ने�146 अं�ेज����य��को�एक�छोटे �से�कमरे�म��कैद�कर��दया��जसम��से�123


क��दम�घुटने�से�मृ�यु�हो�गई।�

हालां�क�इ�तहासकार�इस�कहानी�पर��व�ास�नह��करते�अथवा�कहते�ह���क�मरने�वाले�लोग��क��सं�या�काफ��कम�थी।�

QUESTION 19.
�लासी�क��लड़ाई�के�संदभ��म���न�न�ल�खत�म��से�कौन-से�कथन�सही�ह�?

1. इस�लड़ाई�ने�संपूण��भारत�म��अं�ेज��क��सव��चता��था�पत�कर�द�।�

2. लड़ाई�के�प�ात�मीर�जाफर�बंगाल�का�नवाब�बन�गया�और�वह�अं�ेज��के��नयं�ण�से�पूरी�तरह�मु��था।�

3. लड़ाई�ने�अब�तक�चले�आ�रहे�सरकार�के��व�प�को�प�रव�त�त�कर��दया।�

सही�कूट�का�चयन�क��जए:

a) केवल�1 और�2
b) केवल�1

IASbaba
Web: http://ilp.iasbaba.com/ Score:
Email: ilp@iasbaba.com 0.00 / 200
Page 89
2019 - Test 4-
Exam Title :
History & Cu...
Email : ilbsnaa4@gmail.com
Contact :

c) केवल�1 और�3
d) उपरो��म��से�कोई�नह��
Correct Answer: D
Your Answer:
Explanation

Solution (d)

म�ास�से�रॉबट� ��लाइव�के�नेतृ�व�म��एक�मजबूत�सेना�के�कलक�ा�प�ँचने�ने�बंगाल�म��अं�ेज��क����थ�त�को�मजबूत�बना��दया।��लाइव�ने�नवाब
के��ो�हय��मीर�जाफर, राय��ल�भ, जगत�सेठ�(बंगाल�का�एक��भावशाली�ब�कर) और�अमीचंद�के�साथ�गु�त�गठबंधन�बना��लया।�

समझौते�के�मुता�बक, मीर�जाफर�को�नवाब�बनाया�जाना�था�जो�इसके�बदले�कंपनी�को�उसक��सेवा��के��लए�पुर�कृत�करेगा�।
�लासी�क��लड़ाई�ने�बंगाल�के�अकूत�संसाधन��को�अं�ेज��के�कदम��म��लाकर�रख��दया।�

�लासी�के�बाद, अं�ेज��ने�बंगाल�के��ापार�और�वा�ण�य�पर�लगभग�एका�धकार��था�पत�कर��लया।�इस��वजय�के�प�रणाम�व�प, मीर


जाफर�बंगाल�का�नवाब�बन�गया�। उसने�कंपनी�को�बड़ी�रकम�दे ने�के�साथ-साथ�24 परगन��क��जम�दारी�भी��दान�क��।

�लासी�क��लड़ाई�का�राजनी�तक�मह�व�इस�लए�है��य��क�इसने�भारत�म�����टश�सा�ा�य�क��न�व�डाली�; इसे�सही�ही�भारत�म�����टश
शासन�क��शु�आत�कहा�जाता�है�।

इस�लड़ाई�ने�बंगाल�म��अं�ेज��क��सव��चता��था�पत�कर�द�।�उनके�मु�य���त�ं ����ांसीसी�बाहर�कर��दए�गए�थे।�उ�ह��एक�सुस��जत
सेना�के�रखरखाव�के��लए��े���क���ा��त��ई�और�उनक����त�ा�कई�गुना�बढ़�गई।��क�तु, सरकार�के��व�प�म��कोई���य��प�रवत�न�नह�
�आ, हालां�क�मामल��का�सव��च��नयं�ण��लाइव�के�पास�था��जसक��सहायता�से�नया�नवाब�मीर�जाफर�अपनी�नई�है�सयत�को�सभालने
हेतु�पूण�तया�उस�पर�आ��त�था�।

कलक�ा�पर����टश�सं�भुता�को�मा�यता��मल�गई�और�अं�ेज��ने�नवाब�के�दरबार�म��एक�रे�जड�ट�क���नयु���कर�द�।�

QUESTION 20.
सरकार�के��ै ध�शासन�के�संदभ��म���न�न�ल�खत�कथन��पर��वचार�क��जए।�

1. यह��लासी�क��लड़ाई�के�तुरंत�प�ात�रॉबट� ��लाइव�के��ारा�लाया�गया�था।�

2. द�वानी�अ�धकार�कंपनी�के�पास�थे�जब�क��नजामत�के�काय��नवाब�के�अधीन�थे।�

सही�कूट�का�चयन�क��जए:

a) केवल�1
b) केवल�2
c) 1 और�2 दोन��
d) उपरो��म��से�कोई�नह��
Correct Answer: D
Your Answer:
Explanation

Solution (d)

ब�सर�क��लड़ाई�के�प�ात, ई�ट�इं�डया�कंपनी�बंगाल�क��वा�त�वक��वामी�बन�गई।�रॉबट� ��लाइव�ने�बंगाल�म���ै ध�शासन�क��शु�आत�क�


अथा�त�दो�का�शासन- �जसम��कंपनी�और�नवाब�थे�।�इसके�अंतग�त�द�वानी�(अथा�त�राज�व�सं�हण) और��नजामत�(अथा�त�पु�लस�और
�या�यक�काय�) दोन��काय��कंपनी�के�अंतग�त�आ�गए�।

कंपनी�ने�द�वान�के�तौर�पर�अपने�द�वानी�अ�धकार��का��योग��कया�और��नजामत�अ�धकार��का��योग�नायब�सूबेदार�को�नामां�कत
कर��कया�।�कंपनी�को�द�वानी�काय��स�ाट�से��मले�और��नजामत�काय��बंगाल�के�सूबेदार�से��ा�त��ए।�

�या�आप�जानते�ह��?

IASbaba
Web: http://ilp.iasbaba.com/ Score:
Email: ilp@iasbaba.com 0.00 / 200
Page 90
2019 - Test 4-
Exam Title :
History & Cu...
Email : ilbsnaa4@gmail.com
Contact :

इस��णाली�से�कंपनी�को�काफ��लाभ�प�ँचा�।�इससे��दखावे�के�तौर�पर�अ�धकार�कठपुतली�भारतीय�शासक�के�पास�रहे�और�सं�भु�श��
कंपनी�के�हाथ��म��रही।�नवाब�शां�त�और��व�था�बनाए�रखने�के��लए��ज�मेदार�था��क�तु�वह�धन�और�बल�के��लए�क�पनी�पर��नभ�र�था
�य��क�कंपनी�ही�सेना�और�राज�व�पर��नयं�ण�रखती�थी।�

QUESTION 21.
�ेट�लीप�फॉरवड��अ�भयान(great leap forward campaign) के��वषय�म���न�न�ल�खत�म��से�कौन-सा�कथन�सही�है?

a) यह�1958 म��आरंभ��कया�गया�चीनी�अ�भयान�है��जसका�उ�े �य�वृहत��तर�पर�दे श�का�औ�ो�गक�करण�करना�था।�


b) यह�1992 म��आरंभ��कया�गया�भारतीय�अ�भयान�है��जसका�उ�े �य�वृहत��तर�पर�दे श�का�औ�ो�गक�करण�करना�था।�
c) यह�1958 म��आरंभ��कया�गया�चीनी�अ�भयान�है��जसका�उ�े �य�उपभो�ा�व�तु��के�आयात�का��थानाप��था।�
d) यह�1958 म��आरंभ��कया�गया�पा�क�तानी�अ�भयान�है��जसका�उ�े �य�ह�रत��ां�त��ौ�ो�गक��का��योग�कर�अनाज�का�उ�पादन�बढ़ाना
था।�
Correct Answer: A
Your Answer:
Explanation

Solution (a)

1958 म��आरंभ��कए�गए��ेट�लीप�फॉरवड��(GLF) का�उ�े �य�वृहत��तर�पर�दे श�का�औ�ो�गक�करण�करना�था।�लोग��को�उनके�घर��म�


उ�ोग�लगाने�के��लए��ो�सा�हत��कया�गया।�

�ामीण��े���म��क�यून�शु���कए�गए।�क�यून��णाली�के�अंतग�त, लोग��ने�सामू�हक��प�से�जमीन��क��जुताई�क�।�1958 म��26,000 क�यून


थे�जो�लगभग�सारी�कृषक�जनसँ�या�को�आ�छा�दत�करते�थे।�

�ेट�लीप�फॉरवड��अ�भयान�म��अनेक�सम�याएं�आई।�एक�गंभीर�सूखे�ने�चीन�म��भयंकर�तबाही�मचाई�और�लगभग�तीन�करोड़�लोग��क��जान
ली।�जब��स�का�चीन�के�साथ�झगड़ा��आ�तो�उसने�पूव��म��औ�ो�गक�करण�क�����या�के��लए�चीन�भेजे�गए�अपने�पेशेवर��को�वापस�बुला
�लया।�

1965 म��माओ�महान�सव�हारा�सां�कृ�तक��ां�त�(1966 – 76) लेकर�आया��जसके�अंतग�त�छा���और�पेशेवर��को�दे श�के��ामीण��ह�स��म�


काय��करने�और�सीखने�के��लए�भेजा�गया।�

QUESTION 22.
�न�न�ल�खत�म��से�कौन-सा�संगठन�मानव��वकास��रपोट� �जारी�करता�है?

a) यूएनडीपी�
b) �व��ब�क�
c) �व��आ�थ�क�मंच�
d) यूने�को�
Correct Answer: A
Your Answer:
Explanation

Solution (a)

मानव��वकास��रपोट� �(HDR) एक�वा�ष�क��काशन�है�जो�सं. रा. �वकास�काय��म�(UNDP) के�मानव��वकास��रपोट� �काया�लय��ारा


�का�शत��कया�जाता�है।�

2013 म��अं�तम�दशक�के�दौरान�मानव��वकास�संकेतक��के�मू�य��म��वै��क��प�से�अ�भसरण�दे खने�को��मला, हालाँ�क��े���के�भीतर�और


उनके�बीच��ग�त�असमान�थी।��वकाशशील�दे श��के�बढ़ते�राजनी�तक��भाव�के�साथ�बड़ी�अथ��व�था�म���पांतरण�ने�मानव��वकास����या�
को��भा�वत��कया।�

IASbaba
Web: http://ilp.iasbaba.com/ Score:
Email: ilp@iasbaba.com 0.00 / 200
Page 91
2019 - Test 4-
Exam Title :
History & Cu...
Email : ilbsnaa4@gmail.com
Contact :

मानव��वकास�सूचकांक�(HDI) जीवन���याशा, �श�ा�और���त�����आय�संकेतक��का�एक�संयु��सूचकांक�है��जनका��योग�दे श��को


मानव��वकास�के�चार��तर��पर�र��क�ग�दे ने�के��लए��कया�जाता�है।�जीवन�अव�ध�लंबी�होने, �श�ा�का��तर�उ�च�होने�और���त�����GDP
उ�च�होने�पर��कसी�दे श�को�उ�च��कोर��ा�त�होता�है।�पा�क�तानी�अथ�शा��ी�महबूब�उल�हक�और�भारतीय�अथ�शा��ी�अम�य��सेन��ारा
�वक�सत��कया�गया�HDI आगे�चलकर�सं. रा. �वकास�काय��म�(UNDP) �ारा�दे श�के��वकास�को�मापने�हेतु��योग��कया�जाने�लगा।�

�या�आप�जानते�ह��?

मानव��वकास�सूचकांक�(HDI) इस�बात�पर�जोर�दे ने�के��लए��वक�सत��कया�गया�था��क��व�ता�रत�होते�मानव��वक�प��को��वकास�के


प�रणाम�मापने�क��अं�तम�कसौट��होना�चा�हए�।�आ�थ�क�संवृ���उस����या�म��एक�साधन�है�न��क�अपने�आप�म��एक�सा�य�। HDI
का��योग�रा�ीय�नी�त�के�चुनाव�पर�सवाल�उठाने�हेतु�भी��कया�जा�सकता�है�तथा�पूछा�जा�सकता�है��क�कैसे���त�����समान�सकल�रा�ीय
आय�(GNI) वाले�दो�दे श��म��मानव��वकास�के��भ�-�भ��प�रणाम��दखाई�दे �सकते�ह�।�

उदाहरण�के��लए, मले�शया�म����त�����GNI �चली�से�अ�धक�है, �क�तु�जीवन���याशा��चली�से�लगभग�7 वष��कम�और��कूल�जाने�के


अनुमा�नत�वष��क��सं�या��चली�से�3 कम�है��जसके�चलते��चली�का�HDI मू�य�मले�शया�से�कह��अ�धक�है।�ये��प���भ�ताएं�नी�त
�ाथ�मकता��के�बारे�म��चचा��को��ो�सा�हत�कर�सकती�ह�।�

2010 क��मानव��वकास��रपोट� �ने�असमानता�समायो�जत�मानव��वकास�सूचकांक�(IHDI) जारी��कया�।�जहाँ�सामा�य�HDI


अभी�भी�उपयोगी�है, यह�कहता�है��क�"IHDI मानव��वकास�का�सही��तर�है�(जो�असमानता�को�भी�सं�ान�म��लेता�है)", तथा�"HDI को
‘संभा�वत’ मानव��वकास�(अथवा�अ�धकतम�IHDI हा�सल��कया�जा�सकता�है�य�द�कोई�भी�असमानता�न�रहे) के�तौर�पर�दे खा�जा�सकता
है"।�

QUESTION 23.
�न�न�म��से�कौन-सा�कथन�पया�वरण�क��वहन��मता�के�अथ��क��सव��म��ा�या�करता�है?

a) वह�अव�था��जसम��संसाधन��के�दोहन�क��दर�उनके�पुनभ�रण�क��दर�से�अ�धक�होती�है।�
b) वह�अव�था��जसम��संसाधन��के�पुनभ�रण�क��दर�उनके�दोहन�क��दर�से�अ�धक�होती�है�तथा�उ�प��अप�श��पया�वरण�क��आ�मसात
करने�क���मता�के�अंतग�त�होता�है।�
c) यह�पया�वरण�क���न�नीकरण�को�समा�हत�करने�क���मता�है।�
d) उपरो��म��से�कोई�नह��
Correct Answer: B
Your Answer:
Explanation

Solution (b)

पया�वरण�चार�मह�वपूण��काय��करता�है-

(i) यह�संसाधन��क��आपू�त��करता�है: संसाधन��म��नवीकरणीय�और�गैर-नवीकरणीय�दोन��शा�मल�होते�ह�।�नवीकरणीय�संसाधन�वे�ह���ज�ह�


समा�त�होने�क��संभावना�के��बना��योग��कया�जा�सकता�है।�अथा�त, संसाधन��क��एक�सतत�आपू�त��उपल�ध�रहती�है।�नवीकरणीय�संसाधन�
के�उदाहरण�ह��वन��के�वृ��और�महासागर�क��मछ�लयाँ।��सरी�ओर, गैर- नवीकरणीय�संसाधन�वे�संसाधन�ह��जो��योग�करने�पर�समा�त�हो
जाते�ह�, जैसे�जीवा�म��धन।�

(ii) यह�अप�श��को�आ�मसात�करता�है�

(iii) यह�आनुवां�शक�य�और�जैव��व�वधता��दान�कर�जीवन�को�बचाए�रखता�है।�

(iv) यह�स�दय�परक�सेवाएं�जैसे�सुंदर�भू��य�आ�द��दान�करता�है।�

पया�वरण�तब�तक�ये�काय���नबा�ध��प�से�करने�म��स�म�होता�है�जब�तक�इन�काय��क��माँग�इसक��वहन��मता�के�भीतर�है।�इसका�अथ��है��क�
संसाधन��के�दोहन�क��दर�उनके�पुनभ�रण�क��दर�से�अ�धक�नह��है�तथा�उ�प��अप�श��पया�वरण�क��आ�मसात�करने�क���मता�के
भीतर�है।�जब�ऐसा�नह��होता, पया�वरण�अपना�जीवन�बनाए�रखने�का�तीसरा�काय��करने�म���वफल�रहता�है�और�इसका�प�रणाम�पया�वरणीय
संकट�के��प�म��सामने�आता�है।�

IASbaba
Web: http://ilp.iasbaba.com/ Score:
Email: ilp@iasbaba.com 0.00 / 200
Page 92
2019 - Test 4-
Exam Title :
History & Cu...
Email : ilbsnaa4@gmail.com
Contact :

अनेक�संसाधन��वलु�त�हो�गए�ह��और�उ�प��अप�श��पया�वरण�क��आ�मसात�करने�क���मता�के�बाहर�चला�गया�है।�अवशोषण��मता�का
अथ��है�पया�वरण�क���न�नीकरण�के�अवशोषण�क���मता।�

QUESTION 24.
�न�न�ल�खत�को�2018 म��भारत�म���ए�कुल��व�ुत�उ�पादन�म��उनके�योगदान�के�अनुसार�आरोही��म�म���व��थत�क��जए।�

1. तापीय�उजा��

2. जलीय�उजा��

3. परमाणु�उजा��

4. नवीकरणीय�उजा��

सही�कूट�का�चयन�क��जए:

a) 3<4<1<2
b) 3<4<2<1
c) 3<2<4<1
d) 3<2<3<4
Correct Answer: C
Your Answer:
Explanation

Solution (c)

�े�� योगदान�

तापीय��व�ुत�श��� 64.8%

जल��वघुत�श��� 13.2%

परमाणु��व�ुत�श��� 2%

नवीकरणीय�उजा����ोत� 20.1%

QUESTION 25.
भारत�म��रोजगार�क����थ�त�के�संदभ��म���न�न�ल�खत�कथन��पर��वचार�क��जए।�

1. ��मक-जनसं�या�अनुपात, जनसं�या�के�उस�अनुपात�को�दशा�ता�है�जो�दे श�म��व�तु��और�सेवा��के�उ�पादन�म��स��य�योगदान�दे �रहा


है।�

2. यह�अनुपात��जतना�उ�च�होगा, आ�थ�क�ग�त�व�धय��म��लोग��क��भागीदारी�उतनी�ही�कम�होगी।�

सही�कूट�का�चयन�क��जए:

a) केवल�1
b) केवल�2

IASbaba
Web: http://ilp.iasbaba.com/ Score:
Email: ilp@iasbaba.com 0.00 / 200
Page 93
2019 - Test 4-
Exam Title :
History & Cu...
Email : ilbsnaa4@gmail.com
Contact :

c) 1 और�2 दोन��
d) उपरो��म��से�कोई�नह��
Correct Answer: A
Your Answer:
Explanation

Solution (a)

��मक-जनसँ�या�अनुपात�एक�संकेतक�है�जो�दे श�म��रोजगार�क����थ�त�के��व�ेषण�के��लए��योग��कया�जाता�है।�

यह�जनसँ�या�के�उस�अनुपात�को�जानने�हेतु�उपयोगी�है�जो�दे श�म��व�तु��और�सेवा��के�उ�पादन�म��स��य�योगदान�दे �रहा�है।�

य�द�यह�अनुपात�उ�च�है, इसका�अथ��है��क�आ�थ�क�ग�त�व�धय��म��लोग��क��भागीदारी�उतनी�ही�अ�धक�है�; य�द��कसी�दे श�म��यह


अनुपात�म�यम�है, अथवा�कम�है, इसका�अथ��है��क�इसक��जनसँ�या�का�एक�उ�च�अनुपात�आ�थ�क�ग�त�व�धय��म����य��शा�मल�नह��है।�

QUESTION 26.
�न�न�ल�खत�म��से�कौन-से�दे श��ब�सटे क�के�सद�य�नह��ह�?

1. लाओस�

2. �यांमार�

3. मालद�व�

4. चीन�

5. थाईल�ड�

सही�कूट�का�चयन�क��जए:

a) केवल�1,2 और�3
b) केवल�1,3 और�4
c) केवल�1 और�4
d) केवल�1 और�3
Correct Answer: B
Your Answer:
Explanation

Solution (b)

बे�ऑफ�बंगाल�इनी�शए�टव�फॉर�म�ट��से�टोरल�टे ��नकल�ए�ड�इकोनॉ�मक�को-ऑपरेशन�(BIMSTEC) द��ण�ए�शया�और�द��ण�पूव�


ए�शया�के�सात�दे श��का�एक�अंतरा��ीय�संगठन�है��जनम��1.5 �ब�लयन�लोग�रहते�ह���जनका�सकल�घरेलु�उ�पाद�संयु���प�से�$2.5 ���लयन
(2014) बैठता�है।�

�ब�सटे क�के�सद�य�दे श— बां�लादे श�, भारत�, �यांमार�, �ीलंका�, थाईल�ड�, भूटान�, और�नेपाल�—ये�सब�वे�दे श�ह��जो�बंगाल�क�
खाड़ी�पर��नभ�र�ह�।�

इसका�नेतृ�व�दे श�के�नाम�के�अ�र�के�आधार�पर�च��य�ढं ग�से�चलता�है।�इसका��थायी�स�चवालय�ढाका�म��है।�

�या�आप�जानते�ह��?

• �ब�सटे क�का�चौथा�स�मलेन�2018 म��नेपाल�म��होना���ता�वत�है।�

QUESTION 27.

IASbaba
Web: http://ilp.iasbaba.com/ Score:
Email: ilp@iasbaba.com 0.00 / 200
Page 94
2019 - Test 4-
Exam Title :
History & Cu...
Email : ilbsnaa4@gmail.com
Contact :

�न�न�ल�खत�म��से��वदे शी�मु�ा��व�नमय�कोष�म��स��म�लत��कया�जाता�है?

1. ब�क�नोट�

2. सरकारी���तभू�तयां�

3. बॉ�ड् स�

4. �े ज़री��ब�स�

सही�कूट�का�चयन�क��जए:

a) केवल�1,2 और�4
b) केवल�1
c) केवल�1,2 और�3
d) उपयु���सभी�
Correct Answer: D
Your Answer:
Explanation

Solution (d)

�वदे शी�मु�ा��व�नमय�कोष��का��योग�दा�य�व��के�समथ�न�और�मौ��क�नी�त�को��भा�वत�करने�के��लए��कया�जाता�है�।�यह�उस��वदे शी
धन�को�संद�भ�त�करता�है�जो��कसी�एक�क���य�ब�क��ारा�धा�रत�है�जैसे�भारत�का��रजव��ब�क।�इन�कोष��म��ब�क�नोट, जमाएं, बांड्स, �े ज़री
�ब�स�और�अ�य�सरकारी���तभू�तयां�शा�मल�हो�सकती�ह��।�ये�प�रस�प�तयाँ�अनेक�उ�े �य��क��पू�त��करती�ह���क�तु�इ�ह��धारण�करने�का
सबसे�मह�वपूण��उ�े �य�यह�सु�न��त�करना�है��क�क���सरकार�क��एक�एज�सी�के�पास�ऐसी�अव�था�हेतु�समथ�न�कोष�ह��जब�उसक��रा�ीय�मु�ा
का�तेजी�से�अवमू�यन�हो�जाता�है�अथवा�यह�एकदम�से��दवा�लया�हो�जाती�है।�

यह���नया�भर�के�दे श���ारा�अपनाया�जाने�वाला�सामा�य�तरीका�है��जसके�तहत�उनके�क���य�ब�क��वदे शी�मु�ा�क��एक�बड़ी�रा�श�रखते�ह�।


इनमे�से�अ�धकाँश�कोष�अमे�रक��डॉलर�म��रखे�जाते�ह���य��क�इस�मु�ा�का��व��म��सवा��धक��ापार��कया�जाता�है।�

�वदे शी�मु�ा��व�नमय�कोष��को����टश�प�ड�(GBP), यूरो�(EUR), चीनी�युआन�(CNY) अथवा�जापानी�येन�(JPY) म��भी�रखा�जा


सकता�है।�

QUESTION 28.
�न�न�ल�खत�म��से�अंतरा��ीय�मु�ा�कोष��ारा�जारी�क��जाने�वाली��रपोट� �ह�?

1. �फ�कल�मॉ�नटर�

2. व�ड��इकनो�मक�आउटलुक�

3. �लोबल�फाइन��सयल��टे �ब�लट���रपोट� �

4. व�ड��डेवलपम�ट��रपोट� �

सही�कूट�का�चयन�क��जए:

a) केवल�1,2 और�4
b) केवल�1 और�2
c) केवल�1 और�3
d) 4 के�अ�त�र��सभी�
Correct Answer: D
Your Answer:
Explanation

IASbaba
Web: http://ilp.iasbaba.com/ Score:
Email: ilp@iasbaba.com 0.00 / 200
Page 95
2019 - Test 4-
Exam Title :
History & Cu...
Email : ilbsnaa4@gmail.com
Contact :

Solution (d)

व�ड��डेवलपम�ट��रपोट� ���तवष���व��ब�क�समूह��ारा��का�शत�क��जाती�है।�2018 क���रपोट� �का��वषय�है�“LEARNING to Realize


Education’s Promise”

QUESTION 29.
12 व��पंचवष�य�योजना�के�प�ात�सरकार, भारतीय�अथ��व�था�के��वकास�के��लए�तीन�वष�य�ए�शन�एज�डे�का��नमा�ण��कया�गया�है।�इस
एज�डे�क��समयाव�ध��या�है?

a) 2017-2020
b) 2018-2021
c) 2019-2022
d) 2020-2023
Correct Answer: A
Your Answer:
Explanation

Solution (a)

1st जनवरी�2015 को�रा���य�भारत�प�रवत�न�सं�थान�अथवा�नी�त�आयोग�दे श�के�एक��मुख��थ�क�ट� क�के��प�म��अ��त�व�म��आया।


�धानमं�ी�काया�लय�ने�नी�त�आयोग�को�प��ह�वष�य����कोण�प�, स�त�वष�य�रणनी�त�तथा�तीन�वष�य�ए�शन�एज�डा��प��तैयार
करने�का�सुझाव��दया�।�तदनुसार, वत�मान��प��चौहदव���व��आयोग�के�अं�तम�तीन�वष��( 2017-18 से�2019-20 ) हेतु�नी�तगत
प�रवत�न�व�ए�शन�सुझाने�के��लए�तैयार��कया�जा�रहा�है।�

���कोण�प�, रणनी�त�तथा�ए�शन�एज�डा�पंचवष�य�योजना����या�से�एक�गमन�का���त�न�ध�व�करते�ह���जसके�प�ात��व��वष��2016-17
तक�चंद�और�चीज��को�बंद��कया�जाएगा।�12 व��पंचवष�य�योजना�इन�योजना��म��अं�तम�थी�।�ऐसा�महसूस��कया�गया�है��क�एक
�नरंतर�खुली�और�उदारीकृत�अथ��व�था�होने�के�चलते�हम���वकास����या�क��अवधारणा�मकता�के�उपकरण��और�उपागम��पर�पुन�व�चार
करने�क��आव�यकता�है।�

��ता�वत�प�रवत�न�इस��दशा�म��एक�मह�वपूण��कदम�का���त�न�ध�व�करता�है।�ए�शन�एज�डा�इस�पूरी�कवायद�के�एक�मह�वपूण��भाग�के�तौर
पर�तैयार��कया�गया�है�जो�आगे�चलकर����कोण�एवं�रणनी�त�प��बनेगा।�इसे�यह�बात�सोचकर�तेजी�से�बनाया�और�जारी��कया�गया�है, �क
�व��वष��2016-17 के�आरंभ�होने�के�साथ�नी�त���या�वयन�हेतु�यह�ता�का�लक��प�से��ासं�गक�होगा।�

तीन�वष�य�ए�शन�एज�डा�अपे�ाकृत�छोट��समयाव�ध�म��नी�तगत�प�रवत�न��हेतु�मह�वाकां�ी���ताव���तुत�करता�है।�यह�माना�जा�रहा�है��क
जहाँ�कुछ�को�तीन�वष�य�अव�ध�म��पूण�तया�काया���वत��कया�जा�सकेगा�वह��अ�य��का�काया��वयन�बाद�के�वष��म��जारी�रहेगा।�

QUESTION 30.
�न�न�म��से�कौन-से�कारण�से�उप�ह�अपनी�क�ीय�अ��पर�घूण�न�करते�ह�?

a) उनका��वशाल�आकार�और�गोलाकार�होना�
b) केवल�गु��वाकष�ण�
c) गु��वाकष�ण�तथा�जड़�व�
d) सूय��का�अपने�अ��पर�घूण�न�
Correct Answer: C
Your Answer:
Explanation

Solution (c)

सौर�मंडल�गैस�और�धूल�के�घूमते��ए�बादल�से�बना�था�जो�इसके�क���म����थत�एक�नव�न�म�त�तारे�(हमारा�सूय�) के�चार��ओर�घूमा।�सभी��ह
इस�घूमते��ए��ड�क�आकार�वाले�बादल�से�बने, और�अपने�बनने�के�प�ात�सूय��के�चार��ओर�घूमना�जारी�रखा।�सूय��का�गु��वाकष�ण�सभी��ह�
को�उनक��क�ा�म���रखता�है।�वे�अपनी�क�ा�म��रहते�ह���य��क�सौर�मंडल�म��कोई�अ�य�बल��व�मान�नह��जो�उ�ह��रोक�सके।�

IASbaba
Web: http://ilp.iasbaba.com/ Score:
Email: ilp@iasbaba.com 0.00 / 200
Page 96
2019 - Test 4-
Exam Title :
History & Cu...
Email : ilbsnaa4@gmail.com
Contact :

गु��वाकष�ण�वह��ाथ�मक�बल�है�जो�सूय��के�चार��ओर�क��क�ा�को��नयं��त�करता�है�।�हालाँ�क�सभी��ह��का�उनके�आकार�के�अनुसार
गु��वाकष�ण�है�और�ग�त�है��जस�पर�ये�आगे�बढ़ते�ह���क�तु�क�ा�सूय��के�गु��वाकष�ण�पर�आधा�रत�है।�सूय��का�गु��वाकष�ण�इतना
श��शाली�है��क�वह��ह��को�अपनी�ओर�धकेल�कर�रख�सके�और�एक�क�ीय��ा�प�का��नमा�ण�कर�सके��क�तु�यह�इतना�श��शाली
नह��है��क�इन��ह��को�सूय��म���गरा�सके।�यह�पृ�वी�के�चं�मा�और�इसके�उप�ह��क��क�ा�पर�पड़ने�वाले��भाव�के�समान�है।��ह��का�कम
गु��वाकष�ण�उ�ह��सूय��क��ओर��गरने�से�रोकने�म��भी�सहायक�है।�

भौ�तक��का��नयम�जो�यह�बताता�है��क�ग�तशील�व�तु��क���वृ�त�ग�त�म��रहने�क��होती�है, भी��ह��को�उनक��क�ा�म��रखने�म��भू�मका
�नभाता�है।�सूय��और��ह��का�गु��वाकष�ण�जड�व�के�साथ��मलकर�क�ा��के��नमा�ण�और�उ�ह��संगत�बनाए�रखने�हेतु�काय��करते�ह��
।�गु��वाकष�ण�सूय��और��ह��को�एक�साथ�ख�चता�है�तथा�उसी�समय�उ�ह��अलग�भी�बनाए�रखता�है।�जड़�व�ग�त�बनाए�रखने�और�घूमते�रहने
का��झान��दान�करता�है।�जड़�व�क��भौ�तक��के�कारण��ह�एक�सीधी�रेखा�म��गमन�करना�चाहते�ह�।�हालां�क, गु��व��ख�चाव�उनक��ग�त�को
प�रव�त�त�करना�चाहता�है�और�उ�ह��सूय��के�क���क��ओर�ख�चना�चाहता�है।�साथ��मलकर, यह�दो�बल��के�बीच�एक�समझौते�के��प�म��एक
गोल�क�ा�का��नमा�ण�करते�ह�।�

QUESTION 31.
उ�का-पात(meteor) और�उ�का-�प�ड(meteorite) के�संदभ��म���न�न�ल�खत�कथन��पर��वचार�क��जए।�

1. उ�का-पात�एक�अ�त��हीय��प�ड�ह��जो�पृ�वी�के�वायुमंडल�म���वेश�करता�ह��और�वायु�म��ही�आग�के�गोले�क��भां�त�न��हो�जाता�ह�।�

2. उ�का-�प�ड�भी�एक�अ�त��हीय��प�ड�है�जो�पृ�वी�के�वायुमंडल�से�गुजरता�है�और�वायु�म��न���ए��बना�पृ�वी�क��सतह�प�ंचता�है।�

सही�कूट�का�चयन�क��जए:

a) केवल�1
b) केवल�2
c) 1 और�2 दोन��
d) उपरो��म��से�कोई�नह��
Correct Answer: C
Your Answer:
Explanation

Solution (c)

एक�उ�कापात�(meteor) �काश�क��एक�चमक�है�जो�हम�रात�को�तब�दे खते�ह��जब�अ�त��हीय�मलबा�वायुमंडल�से�गुजरते�समय�जल


उठता�है�।�"उ�कापात" से�आशय�मलबे�से�उ�प���काश�क��चमक�से�है�न��क�मलबे�से।�

मलबे�को�उ�का�प�ड�( meteoroid ) कहा�जाता�है�।�एक�उ�का�प�ड�अ�त��हीय�पदाथ��का�एक�टु कड़ा�है�जो�एक��कलोमीटर�से�भी


छोटा�होता�है�और�कई�बार�केवल��मलीमीटर�आकार�का�होता�है।�पृ�वी�के�वायुमंडल�म����व��होने�वाले�अ�धकाँश�उ�का�प�ड�इतने�छोटे �होते�ह�
�क�तुरंत�वा�पीकृत�हो�जाते�ह��और�कभी�भी�पृ�वी�क��सतह�तक�नह��प�ँच�पाते।�

य�द�उ�का�प�ड�का�कोई�भाग�बच�जाता�है�और�पृ�वी�के�वायुमंडल�से�गुजरकर�पृ�वी�पर�आ��गरता�है, तो�वह�उ�का�( meteorite


) कहलाता�है।�हालां�क�अ�धकाँश�उ�का��अ�यंत�छोट��होती�ह���क�तु�उनका�आकार�एक��ाम�(कंकड़�के�आकार�क�) से�लेकर�100 �कलो�ाम
(220 lbs) और�उससे�भी�अ�धक�(�व�वंस�करने�म��स�म�गोला�म��जतना) तक�हो�सकता�है।�

�ु��ह�सामा�यत: च�ान��के�बड़े�टु कड़े�होते�ह��जो�मंगल�और�बृह�प�त�क��क�ा�के�बीच���थत��ु��ह�पेट��से�आते�ह��।

धूमकेतू��ु��ह�जैसे��दखने�वाली�व�तु�होती�है�जो�बफ�, मीथेन, अमो�नया�और�अ�य�यौ�गक��से�ढक��होती�है�जो�धुंधले, बादल�जैसे


खोल�को��वक�सत�करती�है��जसे�कोमा�कहा�जाता�है�तथा�यह�जब�भी�सूय��के��नकट��मण�करती�है, एक�पूँछ�के��प�म����यमान�रहती�ह�।�

QUESTION 32.
आकाश�के�बारह�न����ज�ह��रा�श�च�(Zodiac) कहते�ह�, �न�न�ल�खत�म��से��या�से�ह�-

a) तार��का�समूह�

IASbaba
Web: http://ilp.iasbaba.com/ Score:
Email: ilp@iasbaba.com 0.00 / 200
Page 97
2019 - Test 4-
Exam Title :
History & Cu...
Email : ilbsnaa4@gmail.com
Contact :

b) रोमन�दे वता��के��च��
c) �ह��के�माग��को�चार��ओर�से�घेरे��ए�का�प�नक��े��
d) बा���ह��का�समूह�
Correct Answer: A
Your Answer:
Explanation

Solution (a)

प��मी��यो�तषशा���म��बारह��च��होते�ह���क�तु�ये�एक�वष��क��12 अव�धय��का���त�न�ध�व�करते�ह�।��ाकृ�तक��वतरण�के�अनुसार, तार��को�


�व�भ��आकार�के�अनेक��े���म���वभ���कया�जाता�है, �जनम��से���येक�को�न���कहा�जाता�है�।�एक�न���के�सभी�चमक�ले�तार�
को�एक�रेखा�से�जोड़ने�पर�जानवर��और�व�तु��क���व�भ��छ�वयाँ�बनती�ह�।�लोग��ने�आकार�के�अनुसार���येक�न���का�नाम�रखा�है।�अंतरा�
�ीय�खगोलीय�संघ�ने�आकाश�को�88 न����म��बांटा�है�तथा�उनक��सट�क�सीमा��नधा��रत�क��है��जससे���येक�तारा��कसी�न����वशेष�से
जुड़ा�है।�

पृ�वी�से�दे खने�पर, सूय��आकाशीय��े��म��धीरे-धीरे�घूमता��दखाई�दे ता�है�और�न����के�बीच�से�गुजरते��ए�एक�वष��हेतु�एक�बड़े�वृ��का��नमा�ण


करता�है।�यह�वृ���ां�तवृ��कहलाता�है�।��ां�तवृ��को�बारह�बराबर�भाग��(��येक�भाग�30 �ड�ी�के�बराबर) म���वभ���कया�जाता�है;
��येक�भाग�का�नाम�उसके��नकटतम�न���के�नाम�पर�रखा�जाता�है।�ये�सभी�बारह�भाग��ां�तवृ��न���कहलाए��जनके�आधार�पर�प��मी
ज�मकु�डली��स�ांत��वक�सत��ए।�

सूय��के�न����े��म��रहने�क��अव�ध�के��हसाब�से��यो�तषी�एक�वष��को�12 अव�धय��म���वभ��करते�ह��।�अत: ��येक�का�उसके�ज�म


के�समय�के�अनुसार�एक�संबं�धत�रा�श��च��होता�है।�ये�12 �च��ह��मेष, वृष, �मथुन, कक�, �स�ह, क�या, तुला, वॄ��क, धनु, मकर, कु�भ,
मीन।�लोग��का�मानना�है��क��भ��रा�शय��वाले�लोग��क���वशेषताएं�और���तभा��भ�-�भ��होती�है।�

QUESTION 33.
सूय���ारा�अपनी�आकाशगंगा�के�क���के�चार��ओर�च�कर�लगाने�म��लगने�वाला�समय�कहलाता�है?

a) कॉ��मक�वष��
b) सौर�वष��
c) पारसेक�
d) �काश�वष��
Correct Answer: A
Your Answer:
Explanation

Solution (a)

आकाशगंगा�वष���जसे�कॉ��मक�वष��के�नाम�से�भी�जाना�जाता�है, वह�अव�ध�है�जो�सूय��को��म�क��वे�गैले�सी�के�क���का�च�कर�पूरा
करने�हेतु�लगती�है।�

एक�च�कर�क��लंबाई�अनुमानत: 225 से�250 �म�लयन�पा�थ�व�वष��होती�है।�सौर�मंडल�अपने���ेप�पथ�पर�आकाशगंगा�क���के�भीतर


828,000 km/h (230 km/s) अथवा�514,000 mph (143 mi/s) क��र�तार�से�ग�त�कर�रहा�है।�इस�र�तार�से�कोई�व�तु�पृ�वी
क��प�र�मा�2 �मनट�54 सेक�ड�म��पूरी�कर�सकती�है।�

एक��काश�वष���री�क��इकाई�है�।�यह�वह��री�है�जो��काश�एक�वष��म��तय�करता�है।��काश�लगभग�300,000 �कलोमीटर���त�सेकंड�के
वेग�से�ग�त�करता�है।�अत: एक�वष��म��यह�लगभग�10 ���लयन��कमी. �री�तय�कर�सकता�है।�इस��कार, एक��काश�वष�
9,500,000,000,000 �कलोमीटर�के�बराबर�होता�है।�

पृ�वी�को�सूय��का�एक�पूरा�च�कर�काटने�म��लगने�वाला�समय�जो�एक�वसंत��वषुव�से�अगले�वसंत��वषुव�तक�मापा�जाता�है�365 �दन, 5 घंटे,


48 �मनट’, 45.51 सेकंड�के�बराबर�होता�है��जसे�सौर�वष��कहते�ह�।�इसे�खगोलीय�वष��अथवा�उ�णक�टब�धीय�वष��भी�कहते�ह��।

पारसेक�लंबाई�क��एक�इकाई�है��जसे�सौर�मंडल�के�बाहर�क��खगोलीय�व�तु��क��बड़ी���रयां�मापने�के��लए��यु���कया�जाता�है।�

IASbaba
Web: http://ilp.iasbaba.com/ Score:
Email: ilp@iasbaba.com 0.00 / 200
Page 98
2019 - Test 4-
Exam Title :
History & Cu...
Email : ilbsnaa4@gmail.com
Contact :
QUESTION 34.
जब�गम��मै�मा�भंजन(fracture), दरार�, तथा��वालामुखी�से�होकर�पृ�वी�क��सतह�पर��वा�हत�होता�है�तो�इसे�कहा�जाता�है?

a) लावा�
b) �पघला�मै�मा�
c) कोर�मै�मा�
d) म�टल�
Correct Answer: A
Your Answer:
Explanation

Solution (a)

मै�मा��पघली�च�ान�का�बना�होता�है�और�भूपप�ट��म��सं��हत�रहता�है�।

लावा�वह�मै�मा�होता�है�जो��वालामुखीय��छ��के�मा�यम�से�पृ�वी�क��सतह�पर�प�ँचता�है।�

QUESTION 35.
�न�न�ल�खत�म��से�कौन-सी�एक�अंतव�धी��वालामुखी��थलाकृ�त�नह��है?

a) बैथो�लथ�
b) डाईक�
c) लैको�लथ�
d) �स�डर�शंकु�
Correct Answer: D
Your Answer:
Explanation

Solution (d)

�वालामुखी�उदगार�के�समय��नमु���होने�वाला�लावा�ठं डा�होने�पर�आ�नेय�च�ान��म��बदल�जाता�है।�ठं डा�होने�क��यह����या�सतह�पर


प�ँचने�पर�हो�सकती�है�अथवा�तब�भी�जब�लावा�भूपप�ट��के�भाग��म��ही�है।�

लावा�के�ठं डे�होने�के��थान�के�अनुसार�आ�नेय�च�ान��को��वालामुखी�च�ान��( जो�सतह�पर�ठं डा�होती�ह��) और��लूटो�नक�च�ान��( जो


पप�ट��म��ठं डा�होती�ह��) म��वग�कृत��कया�जाता�है।�भूपप�ट��के��थान��म��ठं डा�होने�वाला�लावा��व�भ��आकार�लेता�है।�इन�आकार��को�अंतव�
धी��थलाकृ�तयां�कहा�जाता�है।�

मै�मा�साम�ी�क��बड़ी�रा�श�जो�पप�ट��क��गहराई�म��ठं डी�होती�है, वह�बैथो�लथ�कहलाने�वाले�बड़े�गुंबद�के��प�म���वक�सत�होती�है।�

IASbaba
Web: http://ilp.iasbaba.com/ Score:
Email: ilp@iasbaba.com 0.00 / 200
Page 99
2019 - Test 4-
Exam Title :
History & Cu...
Email : ilbsnaa4@gmail.com
Contact :

लैको�लथ�बड़े�आकार�क��गुंबदनुमा�अंतव�धी�संरचना�है��जसका�आधार�समतल�होता�है�और�जो�नीचे�से�एक�पाइप�जैसे��वाहक�से
जुड़ी�होती�है�।�यह�संयु���वालामुखी�क��गुंबदनुमा��वालामुखीय�सतह�के�समान��दखती�है�तथा�यही�अ�धक�गहराई�म��अव��थत�होती�है।�

जब�लावा�भू�म�म��बनी�दरार��के�मा�यम�से�अपना�रा�ता�बनाता�है, यह�धरातल�से�लगभग�लंबवत��प�म��ठोस�होने�लगता�है�।�यह�इसी
अव�था�म��ठं डा�होकर�एक�द�वारनुमा�के��प�म���वक�सत�हो�जाता�है।�ऐसी�संरचना��को�डाईक�कहा�जाता�है�।

QUESTION 36.
�न�न�ल�खत�म��से�कौन�से�भूकंप�के��व�रत��भाव�नह��ह�?

1. मृदा��वीकरण�

2. भू�खलन�

3. अवसंरचना�का��वनाश�

4. सुनामी�

5. बाँध�से�बाढ़�और�तटबंद�का�टू टना�

सही�कूट�का�चयन�क��जए:

a) केवल�1,2,3 और�5
b) केवल�1,2,3 और�4
c) केवल�2,3 4 और�5
d) उपयु���सभी�
Correct Answer: D
Your Answer:

IASbaba
Web: http://ilp.iasbaba.com/ Score:
Email: ilp@iasbaba.com 0.00 / 200
Page 100
2019 - Test 4-
Exam Title :
History & Cu...
Email : ilbsnaa4@gmail.com
Contact :
Explanation

Solution (d)

भूकंप�एक��ाकृ�तक�आपदा�है।�भूकंप�के�त�काल��भाव��न�न�ल�खत�ह�:

1. धरातल�का��हल�जाना�
2. धरातल�का�ऊँचा-नीचा�हो�जाना�
3. भू�और�गाद��खलन�
4. मृदा��वीकरण�
5. धरातल�पर�झटके�लगना�
6. �हम�खलन�
7. भू�म�का��खसक�जाना�
8. बाँध�से�बाढ़�और�तटबंद�टू टना�
9. आग�
10. अवसंरचना��वनाश�
11. व�तु��का��गरना�
12. सुनामी�

पहले�छ: का��भाव�भू�म�के��ा�प�पर�पड़ता�है�जब�क�अ�य��े��के�लोग��क��जान�एवं�संप�त�क����त�का�कारण�बन�सकते�ह�।�

सुनामी�का��भाव�तभी��दखाई�दे गा�जब�भूकंप�का�अ�धके���महासागरीय�जल�के�नीचे�है�और�इसक��ती�ता�उ�च�है।�सुनामी�वे�तरंगे�ह��जो
झटक��से�पैदा�होती�ह��तथा�वे�अपने�आप�म��भूकंप�नह��ह��।�हालां�क�वा�त�वक�भूकंप�ग�त�व�ध�कुछ�सेकंड�के��लए�होती�है, �क�तु�इसक�
ती�ता��र�टर��केल�पर�5 या�अ�धक�होने�पर�यह�बेहद��वनाशकारी�होती�है।�

QUESTION 37.
कानून�और�नै�तकता�के�संदभ��म���न�न�ल�खत�कथन��पर��वचार�क��जए।�

1. क़ानून�मानव�के�बा��काय��से�संबं�धत�ह��न��क�उसके��योजन(motives) से, जब�क�नै�तकता�बा��काय��और��योजन��दोन��से


संबं�धत�है।�

2. क़ानून�����न��है�जब�क�नै�तकता�व�तु�न��है।�

सही�कूट�का�चयन�क��जए:

a) केवल�1
b) केवल�2
c) 1 और�2 दोन��
d) उपरो��म��से�कोई�नह��
Correct Answer: B
Your Answer:
Explanation

Solution (c)

कुछ�लोग��का�मानना�है��क�सं�वधान�म��मा��कानून�होते�ह��और�यह��क�क़ानून�एक�चीज�है�और�मू�य�व�नै�तकता��सरी�चीज।�अत:, हम
सं�वधान�के���त�केवल�एक��व�धक�उपागम�अपना�सकते�ह��न��क�राजनी�तक�दश�न�उपागम।�

यह�सच�है��क�सभी�कानून��म��नै�तक�साम�ी�नह��होती, �क�तु�अनेक�कानून�हमसे�गहराई�से�जुड़े�मू�य��के�साथ��नकटता�से�संबं�धत�होते
ह�।�उदाहरण�के��लए�एक�क़ानून�भाषा�अथवा�धम��के�आधार�पर����य��के�साथ�भेदभाव�का��नषेध�कर�सकता�है।�ऐसा�क़ानून�समानता�के
�वचार�से�जुड़ा�है।�ऐसा�कानून�इस�लए�अ��त�व�म��है��य��क�हम�समानता�को�मह�व�दे ते�ह�।�अत: कानून��और�नै�तक�मू�य��के�बीच�एक�संबंध
होता�है।�

कानून��और�नै�तक�मू�य��के�बीच�कुछ�अंतर-

IASbaba
Web: http://ilp.iasbaba.com/ Score:
Email: ilp@iasbaba.com 0.00 / 200
Page 101
2019 - Test 4-
Exam Title :
History & Cu...
Email : ilbsnaa4@gmail.com
Contact :

1) क़ानून�मानव�के�बा��काय��से�संबं�धत�ह��न��क�उसके�मंत��से, जब�क�नै�तकता�उसके�बा��काय��और�आंत�रक�मंत���दोन��से�संबं�धत
है।�

2) क़ानून�रा�य�का��वषय�है�जब�क�नै�तकता�अंत:करण�का��वषय�है।�

3) कानून�मानव�के�जीवन�के�एक�भाग�से�संबं�धत�है�जब�क�नै�तकता�मानव�के�पूरे�जीवन�से�संबं�धत�है।�

4) क़ानून�का�उ�लंघन�रा�य��ारा�द�डनीय�है�जब�क�नै�तकता�रा�य��ारा�द�डनीय�नह��है।�

5) क़ानून��न��त�और�सट�क�होता�है�जब�क�नै�तकता�अ�प��और�अ�न��त�होती�है।�

6) क़ानून�����न��है�जब�क�नै�तकता�व�तु�न��है।�

7) क़ानून�रा�य�क��सीमा��के�भीतर�काय��करता�है�जब�क�नै�तकता�वै��क�होती�है।�

8) कानूनी��प�से�गलत�बात�नै�तक��प�से�सही�हो�सकती�है�तथा�नै�तक��प�से�गलत�बात�कानूनन�सही�हो�सकती�है।�

QUESTION 38.
�न�न�ल�खत�म��से�कौन-सा�भारतीय�संघवाद�क��सही��ा�या�करता�है?

a) संवैधा�नक��प�से�सम�मत�
b) संवैधा�नक��प�से�असम�मत�
c) आं�शक��प�से�सम�मत�
d) आं�शक��प�से�सम�मत�और�आं�शक��प�से�असम�मत�
Correct Answer: B
Your Answer:
Explanation

Solution (b)

ज�मू�और�क�मीर�(अनु�छे द�370) तथा�पूव��र�(अनु�छे द��371) से�संबं�धत��ावधान�करके�भारतीय�सं�वधान�असम�मत�संघवाद�क�


अ�यंत�मह�वपूण��अवधारणा�क��बात�करता�है।�सं�वधान�ने�बेहद�मजबूत�क���सरकार�क���व�था�क��है।��क�तु�भारतीय�सं�वधान�के�इस
एका�मक�प�पात�के�बावजूद�इसी�संघ�के�भीतर��व�भ��उप-इकाईय��के��व�धक�दज��और��वशेषा�धकार��के�बीच�संवैधा�नक��प�से�अंतः�था�प
त�अंतर�मौजूद�ह�।�

अमे�रक��संघवाद�क��संवैधा�नक�सम�म�त�से�अलग�भारतीय�संघवाद�संवैधा�नक��प�से�असम�मत�है।�कुछ�उप-इकाईय��क���वशेष
ज�रत��को�पूरा�करने�के��लए, इसे�हमेशा�से�ही�मूल�पाठ�का�भाग�रखा�गया�था��क�उन�इकाई��के�साथ�एक��व�श��संबंध�बनाया�जाये�और
उ�ह���वशेष�दजा���दया�जाए।�

उदाहरण�के��लए, भारत�संघ��ारा�ज�मू-क�मीर�का�अ�ध�हण�इस���तब�ता�पर�आधा�रत�था��क�उसक���वाय�ता�क��सुर�ा�सं�वधान�के
अनु�छे द�370 के�अंतग�त�क��जाएगी।�यह�अकेला�रा�य�है�जो�अपने��वयं�के�सं�वधान��ारा�शा�सत�होता�है�।�इसी��कार, अनु�छे द�37
1A के�अंतग�त��वशेष�दज��संबंधी��वशेषा�धकार�पूव��र�के�नगाल�ड�रा�य�को�भी��दान��कए�गए�थे�।

यह�अनु�छे द�न�केवल�नगाल�ड�के�भीतर�पहले�से�चले�आ�रहे�कानून��को�वैधता��दान�करता�है�अ�पतु�आ�जन�पर���तबंध��के�मा�यम�से
�थानीय�पहचान�को�भी�सुर�ा�दे ता�है।�अनेक�अ�य�रा�य�भी�ऐसे��वशेष��ावधान��के�लाभ�ा�तकता��ह�।�इस��कार�भारतीय�सं�वधान�के�अनुसार
इस�अलग��वहार�म��कोई�बुरी�बात�नह��है।�

QUESTION 39.
भारत�का�सं�वधान��वतं�ता�के�प�ात�से�अपने�आधारभूत�दश�न�के�साथ�अ�ु�ण�बना��आ�है।��न�न�ल�खत�म��से�कौन�से�तक��इस�बात�का
समथ�न�करते�ह�?

1. हमारा�सं�वधान�सु�ढ़�है�और�इसका�आधारभूत�ढ़ांचा�दे श�के��लए�अ�त�उपयु��है।�

2. सं�वधान��नमा�ता�काफ���रदश��थे�और�उ�ह�ने�भ�व�य�क��प�र��थ�तय��हेतु�अनेक�समाधान�उपल�ध�करवाए।�

IASbaba
Web: http://ilp.iasbaba.com/ Score:
Email: ilp@iasbaba.com 0.00 / 200
Page 102
2019 - Test 4-
Exam Title :
History & Cu...
Email : ilbsnaa4@gmail.com
Contact :

सही�कूट�का�चयन�क��जए:

a) केवल�1
b) केवल�2
c) 1 और�2 दोन��
d) उपरो��म��से�कोई�नह��
Correct Answer: C
Your Answer:
Explanation

Solution (c)

बदली��ई�प�र��थ�तय��अथवा�समाज�के�भीतर��वचार��के�प�रवत�न�अथवा�राजनी�तक�उतार-चढ़ाव��क��अनु��या�के��प�म��दे श���ारा�अपने
सं�वधान�को�दोबारा��लखना�असामा�य�बात�नह��है।�अपने�74 वष��के�जीवनकाल�म��सो�वयत�संघ�के�चार�सं�वधान�(1918, 1924, 1936
and 1977) �ए।�

1991 म��सो�वयत�संघ�क��क�यु�न�ट�पाट��का�शासन�समा�त��आ�और�शी��ही�सो�वयत�संघ�का��वखंडन�हो�गया।�इस�राजनी�तक�उतार-
चढाव�के�प�ात, नव�न�म�त��सी�संघ�ने�1993 म��एक�नया�सं�वधान�अपनाया।�

भारत�का�सं�वधान�26 नवंबर�1949 को�अपनाया�गया�। इसका�काया��वयन�औपचा�रक��प�से�26 जनवरी�1950 को�आरंभ


�आ�।�इसके�अड़सठ�वष��से�भी�अ�धक�के�प�ात, उसी�सं�वधान�का�ऐसे��ेमवक��के��प�म��काय��करना�जारी�है���जसके�भीतर�हमारे�दे श�क�
सरकार�काय��संचालन�करती�है।�

यह�सच�है��क�हम���वरासत�म��एक�ब�त�बड़ा�सं�वधान��मला�है�।�इस�सं�वधान�का�आधारभूत��ेमवक��हमारे�दे श�के��लए�काफ��उपयु��है

यह�भी�सच�है��क�सं�वधान��नमा�ता�काफ���रदश��थे�और�उ�ह�ने�भ�व�य�क��प�र��थ�तय��हेतु�अनेक�समाधान�उपल�ध�करवाए।��क�तु
�कसी�भी�सं�वधान�म��सभी�आक��मकता��हेतु��ावधान�नह��हो�सकता।�कोई��प��ऐसा�नह��हो�सकता��जसे�प�रवत�न�क��आव�यकता�न�पड़े।

�या�आप�जानते�ह��?

हमारा�सं�वधान�समाज�क��बदली��ई�ज�रत��के��हसाब�से�प�रवत�न�क��आव�यकता�को��वीकार�करता�है।�सं�वधान�क��वा�त�वक�काय��णाली
म���ा�या��संबंधी�पया��त�लचीलापन�मौजूद�है।�राजनी�तक��वहार�और��या�यक��व�थाएं�दोन��ने�सं�वधान�के�काया��वयन�म�
प�रप�वता�और�लोचशीलता��दखाई�है।�इन�कारक��ने�हमारे�सं�वधान�को�एक�बंद�और�ग�तहीन��नयम�पु��तका�क��बजाए�एक�जीवंत
द�तावेज�बनाया�है�।

QUESTION 40.
�न�न�ल�खत�म��से�कौन-सा�कथन�भारतीय�सं�वधान�क���कृ�त�के�संदभ��म��स�य�है?

1. यह�दे श�के�दश�न�क��अ�भ����है�तथा�अप�रवत�नीय�है।�

2. इसे�दे श�के�सामा�य��वधान��से�ऊपर�है।�

3. यह�एक�प�व��द�तावेज�है, अत: इसे�प�रव�त�त�करने�का�कोई�भी��यास�लोक�ंत�के��व���है।�

सही�कूट�का�चयन�क��जए:

a) केवल�1 और�2
b) केवल�2 और�3
c) केवल�2
d) केवल�1 और�3
Correct Answer: C
Your Answer:

IASbaba
Web: http://ilp.iasbaba.com/ Score:
Email: ilp@iasbaba.com 0.00 / 200
Page 103
2019 - Test 4-
Exam Title :
History & Cu...
Email : ilbsnaa4@gmail.com
Contact :
Explanation

Solution (c)

सं�वधान�के��ावधान��वाभा�वक��प�से�सम�या��से��नपटने�के�उन��यास��को���त�ब��बत�करते�ह���जनका�सामना�इसके�बनने�के
समय�हमारा�समाज�कर�रहा�था।�ठ�क�उसी�समय, सं�वधान�को�एक�ऐसा�द�तावेज�भी�होना�था�जो�सरकार�के��लए�भ�व�य�का��ेमवक��भी
�दान�करे�।�अत:, सं�वधान�को�भ�व�य�म��आ�सकने�वाली�चुनौ�तय��के���त�अनु��या�म��स�म�होना�चा�हए।�

इस�अथ��म�, सं�वधान�म��ऐसा�कुछ�अव�य�होगा�जो�ता�का�लक�हो�तथा�कुछ�ऐसा�जो��थायी�मह�व�का�हो�।

सं�वधान�एक��हमा�छा�दत�व�अप�रवत�नशील��प��नह��है�।�यह�एक�ऐसा��प��है��जसे�मानव��ने�बनाया�है�और��जसे�समी�ा, प�रवत�न
और�पुनन��र�ण�क��आव�यकता�पड़�सकती�है।�

यह�सच�है��क�सं�वधान�संबं�धत�समाज�के�सपन��और�आकां�ा��को���त�ब��बत�करता�है�।�यह�भी��यान�म��रखा�जाना�चा�हए��क
सं�वधान�समाज�के�लोकतां��क�शासन�हेतु�एक��ेमवक��है।�

इस�अथ��म�, यह�एक�ऐसा�उपकरण�है��जसका��नमा�ण�समाज�अपने��लए�करते�ह�।�भारतीय�सं�वधान�के��नमा�ता��ने�सं�वधान�को�सामा�य
�वधान�से�ऊपर�रखा�और�उ�मीद�क���क�भावी�पी�ढयां�इस�द�तावेज�का�स�मान�कर�गी।�

�या�आप�जानते�ह��?

भारतीय�सं�वधान�के��नमा�ता��ने�माना��क�भ�व�य�म��इस�द�तावेज�को�संशोधन�क��आव�यकता�पड़�सकती�है।�इसे��लखते�समय�भी�वे�सजग
थे��क�अनेक�मु���पर�मत�भ�ता�है।�जब�कभी�भी�समाज��कसी��वचारधारा��वशेष�क��ओर�आगे�बढे गा, सं�वधान�म��एक�प�रवत�न�क�
आव�यकता�पड़�सकती�है�।

इस��कार, भारतीय�सं�वधान�उपयु���व�ण�त�दोन��उपागम��का��म�ण�है: यह��क�सं�वधान�एक�प�व��द�तावेज�है�तथा�यह��क�यह�एक


उपकरण�है��जसे�समय-समय�पर�प�रव�त�त��कए�जाने�क��आव�यकता�पड़�सकती�है�।

अ�य�श�द��म�, हमारा�सं�वधान�एक���थर�द�तावेज�नह��है�, यह���येक�बात�को�लेकर�अं�तम�श�द�नह��है; यह�अप�रवत�नशील�नह��है।�

QUESTION 41.
�न�न�ल�खत�म��से�कौन-से�1857 के��व�ोह�क���वफलता�के�कारण�थे?

1. �व�भ��समूह��क���भ�-�भ���वचारधाराएं�थी�और�वे�एक�उ�े �य�के��लए�नह��लड़�रहे�थे।�

2. भारतीय��क��तुलना�म��अं�ेज��के�पास�बेहतर�ह�थयार�और�संसाधन�थे।�

3. अं�ेज��के�पास�संचार�क��ती�तम��णाली�थी।�

4. �व�ोह�के�दौरान�अं�ेज�पड़ोसी�दे श��जैसे�चीन, बमा��और�अफगा�न�तान�म��अनेक�यु��लड़�रहे�थे, अत: वे�केवल�भारत�पर��यान�क���त


नह��कर�पाए।�

�न�न�ल�खत�म��से�कूट�का�चयन�क��जए:

a) 1,2 और�3
b) 2,3 और�4
c) 1,3 और�4
d) उपयु���सभी�
Correct Answer: A
Your Answer:
Explanation

Solution (a)

IASbaba
Web: http://ilp.iasbaba.com/ Score:
Email: ilp@iasbaba.com 0.00 / 200
Page 104
2019 - Test 4-
Exam Title :
History & Cu...
Email : ilbsnaa4@gmail.com
Contact :

�व�ोह�क���वफलता�के�कारण�:

• �व�ो�हय��के�मुकाबले�अं�ेज��के�पास�कह��बेहतर�संसाधन�थे।�अं�ेज��का�सौभा�य�था��क����मयाई�और�चीनी�यु��1856 म��ही
समा�त�हो�गए�थे��जससे����टश�सरकार�को�अपनी�सम�त�उजा��भारत�पर�क���त�करने�म��मदद��मली।�
• �व�ुत�टे ली�ाम�ने�अं�ेज��के�संचार�चैनल�म��सुधार��कया�तथा�कोई�भी�सूचना�बेहद�शी�ता�से�प�ंचाई�जा�सकती�थी।�
• भारतीय�लोग�एक�लंबी�अव�ध�तक��वदे शी�सै�नक��का���तरोध�करने�के��लए�रा�वाद�क��भावना�से��े�रत�नह��रह�सके।��न:संदेह
राजा��ने�अपनी�खोई���त�ा�पुन: �ा�त�करने�के��लए, तालुकदार��ने�अपने��वशेषा�धकार�वापस�पाने�के��लए�तथा�का�तकार��ने
आ�थ�क�असंतोष�के�कारण�(हालां�क�एक�सकारा�मक�ढं ग�से) �व�ोह�म��भाग��लया��क�तु�सभी�भारतीय��को�एक�मंच�पर�लाने�हेतु�कोई
महान�आदश���व�मान�नह��था।�
• इस��कार��व�ोह�लंबे�समय�तक�नह��चल�सका।��व�ोही�अं�ेज�सेना�के��व���कोई�संयु��सै�य�मोचा��नह��बना�सके।�वे�अलग-अलग
समूह��म��लड़े।�आव�यकता�के�समय�वे�अपनी�सम�त�श���को�एकब��नह��कर�सके��जसने�उनके�श�ु��के�प��म��काम��कया।�
• दे श�के�भा�य�को��नद� �शत�करने�हेतु�यो�य�और�होनहार�नेता��क��अनुप��थ�त�दे श�ापी��व�ोह�के��लए�नकारा�मक�प�रणाम�लेकर
आई।��व�ोह��वत:�फूत��था, �व�ोही�सं�या�म��अ�धक�थे��क�तु��भा��य�से�उनम��से�कोई�भी�महान�सै�य�जनरल�नह��था।�वे�अपनी�सी�मत
�मता��के�साथ�हताशापूण��ढं ग�से�लड़े।�
• एकाएक��व�ोह�के�फूटने�ने�एक�अ�न��त���थ�त�को�ज�म��दया��जसके��लए�लोग�मान�सक��प�से�तैयार�नह��थे।�वे�उस�आपात��थ�त
म��अपनी�भू�मका�से�अंजान�थे।�

QUESTION 42.
�न�न�ल�खत�म��से��कसने�1857 के��व�ोह�म��भाग�नह���लया�था?

1. �सख�रे�जम�ट�

2. हैदराबाद�का��नजाम�

3. मैसूर�का�राजा�

4. नेपाल�नरेश�

5. प��मी��श�ा��ा�त�म�यवग�य�भारतीय�

�न�न�ल�खत�म��से�कूट�का�चयन�क��जए:

a) 1,2 और�5
b) 1,3,4 और�5
c) 1,2, 3 और�5
d) उपयु���सभी�
Correct Answer: D
Your Answer:
Explanation

Solution (d)

���टश�सरकार�ने��व�ोह�क��आग�को�दे श�के�बड़े��ह�से�म��नह��फैलने�नह���दया।�

पंजाब�और�बॉ�बे��ेसीड�सी��ब�कुल�अछू ती�रही।��सख�रे�जम�ट�ने��व�ोह�को�कुचलने�म��मह�वपूण��भू�मका��नभाई।�

हैदराबाद�के��नजाम, भोपाल�क��बेगम, नेपाल�नरेश�और�मराठा�सरदार��स��धया�ने�अं�ेज��क��मदद�क�।�

आधु�नक��श��त�भारतीयय��ने��व�ोह�को�एक�प�गामी�कदम�माना।�उनक�����टश�सरकार�म��आ�था�थी�और�उनका�मानना�था��क�वे�समाज�म�
प�रवत�न�ला�सकते�ह��और�इसे�आधु�नक�बना�सकते�ह�।�इस�कारण�से�उ�ह�ने��व�ोह�का�समथ�न�नह���कया।�

नए�जम�दार��ने�भी�अं�ेज��क��मदद�क���य��क�उनका�औ�च�य�अंगेजी�राज�पर�आधा�रत�था।�

QUESTION 43.

IASbaba
Web: http://ilp.iasbaba.com/ Score:
Email: ilp@iasbaba.com 0.00 / 200
Page 105
2019 - Test 4-
Exam Title :
History & Cu...
Email : ilbsnaa4@gmail.com
Contact :

�न�न�ल�खत�म��से�कौन-से�नेता�क��1857 के��व�ोह�के�दौरान�मृ�यु�हो�गई�थी?

1. बहा�र�शाह�जफर�II

2. रानी�ल�मीबाई�

3. बेगम�हजरत�महल�

4. ता�या�टोपे�

5. नाना�साहेब�

�न�न�ल�खत�म��से�कूट�का�चयन�क��जए:

a) 1,2 और�3
b) 2,4 और�5
c) 2 और�4
d) उपयु���सभी�
Correct Answer: C
Your Answer:
Explanation

Solution (c)

वृ��मुग़ल�बादशाह�बहा�र�शाह�जफर�II बंद��बनाकर�रंगून�भेज��दया�गया।����टश�कमांडर-इन-चीफ�को�लन�कै�पबेल�ने�बड़ी�क�ठनाई�से
लखनऊ�को�पुन: �ा�त��कया।�अवध�के�आंत�रक�भाग��म��गु�र�ला��क�म�का�यु��जारी�रहा।�बेगम�हजरत�महल�नेपाल�पलायन�कर�गई।�

17 जून�1857 को�महारानी�ल�मी�बाई�लड़ते��ए�मारी�गई, ता�या�टोपे�ने�काफ��लंबे�समय�तक�लड़ाई�जारी�रखी�और�अंत�म��पकड़ा�गया�और


उसे�फांसी�दे �द��गई।�नाना�साहेब�को�हरा��दया�गया�और�वे�मृ�यु�से�बचने�के��लए�नेपाल�के�घने�जंगल��म��पलायन�कर�गए।�कुंवर��स�ह�यु���े�
म��शहीद��ए।�अनेक��अ�य�या�तो�मारे�गए�या�नेपाल�के�घने�जंगल��म��पलायन�कर�गए।�महान��व�ोह�समा�त�हो�गया।�

QUESTION 44.
�न�न�ल�खत�म��से�कौन-से�����ने�1857 के��व�ोह�को�भारतीय��वतं�ता�क���थम�लड़ाई�कहा�है?

a) �बपन�च���
b) आउटरम�
c) वी�डी�सावरकर�
d) महा�मा�गाँधी�
Correct Answer: C
Your Answer:
Explanation

Solution (c)

�वनायक�दामोदर�सावरकर�ने�1857 के��व�ोह�पर�पु�तक��लखते�समय�“भारतीय��वतं�ता�का�यु�” शीष�क�का��योग��कया।�इस�पु�तक�को


अं�ेजी�राज�म����तब��धत�कर��दया�गया�और�यह�सबसे�पहले�नीदरल�ड्स�म���का�शत��ई।�

QUESTION 45.
1773 का�रे�युले�ट�ग�ए�ट����टश�संसद��ारा��न�म�त�एक�कानून�था�जो�भारत�म��ई�ट�इं�डया�क�पनी�के�शासन�के��बंधन�म��प�रवत�न�हेतु�लाया
गया�था।��न�न�ल�खत�म��से�कौन�से�कथन�इस�कानून�के�संदभ��म��सही�ह�?

1. इसने�अ�य��े�सड��सय��पर�बंगाल�क��सव��चता��था�पत�कर�द�।�

IASbaba
Web: http://ilp.iasbaba.com/ Score:
Email: ilp@iasbaba.com 0.00 / 200
Page 106
2019 - Test 4-
Exam Title :
History & Cu...
Email : ilbsnaa4@gmail.com
Contact :

2. इसने�भारत�म��सभी��े�सड��सय��को�यु���ार�भ�करने�के�आदे श�दे ने�अथवा�शां�त�सं�धयां�ह�ता��रत�करने�से���तबं�धत�कर��दया।�

3. इसने�कलक�ा�के�फोट� ��व�लयम�म��सु�ीम�कोट� �क���थापना�क�।�

�न�न�ल�खत�म��से�कूट�का�चयन�क��जए:

a) 1 और�2
b) 2 और�3
c) 1 और�3
d) उपयु���सभी�
Correct Answer: C
Your Answer:
Explanation

Solution (c)

1773 के�अ�ध�नयम�ने�कंपनी�के�राजनी�तक�काय��को�मा�यता�द���य��क�इसने�पहली�बार�यह�बात�कही��क�सरकार�के��व�प�के�बारे�म�
�नद� श�दे ना�संसद�का�अ�धकार�है।भारत�म���शास�नक�मशीनरी�को�क���कृत�करने�का����टश�सरकार�का�यह�पहला��यास�था।�इन�कानून�ने
कंपनी�के��वे�छाचारी�शासन�के��थान�पर�भारत�म�����टश�अ�धकार�वाले��े���हेतु�एक��ल�खत�सं�वधान�क���थापना�क�।�गवन�र-जनरल�को
�वे�छाचारी�बनने�से�रोकने�हेतु�एक��णाली�लाई�गई।�

इस�कानून�ने��प���प�से�अ�य��े�सड��सय��पर�बंगाल�क��सव��चता��था�पत�कर�द�।��वदे श�नी�त�के�मामल��म�, 1773 के�रे�युले�ट�ग�ए�ट�ने


बॉ�बे�और�म�ास�क���े�सड��सय��को�गवन�र-जनरल�और�उसक��प�रषद�के�अधीन�कर��दया।�अब, कोई�अ�य��ेसीड�सी�भारतीय�राजा��के
साथ�लड़ाई�शु��करने, यु��घो�षत�करने�अथवा��कसी�सं�ध�पर�वाता��करने�का�आदे श�नह��दे �सकती�थी।�इसने�कलक�ा�के�फोट� ��व�लयम�म�
सु�ीम�कोट� �क���थापना�क��तथा�भारत�के�आधु�नक�इ�तहास�क��शु�आत��ई।�

QUESTION 46.
�न�न�ल�खत�म��से�कौन-से����टश�कानून�ने�भारतीय��स�वल�सेवा��म��खुली���त�पधा��क��शु�आत�क�?

a) 1813 का�चाट� र�ए�ट�


b) 1833 का�चाट� र�ए�ट�
c) 1853 का�चाट� र�ए�ट�
d) 1858 का�भारत�सरकार�अ�ध�नयम�
Correct Answer: C
Your Answer:
Explanation

Solution (d)

1833 के�चाट� र�ए�ट�ने��व�था�द���क�कोट� �ऑफ़�डायरे�टस��को��र��य��के�चार�गुना���या�शय��क���सफा�रश�करनी�चा�हए��जनम��से�एक


को���तयोगी�परी�ा�हेतु�चय�नत��कया�जाएगा।�1833 के�चाट� र�ए�ट�ने�यह�भी��व�था�द���क�हैलेबरी�कॉलेज�ऑफ़�लंदन�भावी��स�वल
सेवक��को��वेश�दे ने�का�कोटा��न��त�करना�चा�हए।�हालां�क, खुली���त�पधा��क���णाली�को�कभी�भी��भावी�ढं ग�से�लागू�नह���कया�गया।
लॉड��मैकाले�क��अ�य�ता�वाली�एक�स�म�त�ने�इस�संबंध�म���व�नयम�तैयार��कए�थे।��रपोट� �म��कहा�गया��क:

• हैलेबरी�को�ICS के��लए�उ�च��श�ा�कॉलेज�के��प�म��बनाए�रखना�बंद�करना�चा�हए।�
• ICS रंग�ट��के��लए��वशेषीकृत��श�ा�के��थान�पर�एक��ापक�सामा�य��श�ा�होनी�चा�हए।�
• भत��एक�खुली���तयोगी�परी�ा�पर�आधा�रत�हो��जससे�सव��े��उ�मीदवार�आगे�लाए�जा�सक�, न��क�केवल�सतही��ान�वाले�लोग।�
• �नयु���परीवी�ा�अव�ध�के�अधीन�होनी�चा�हए।�

1833 के�चाट� र�ए�ट�ने�कोट� �ऑफ़�डायरे�टस��को�भारतीय��नयु��य��म��उनके�संर�ण�से�वं�चत�कर��दया�तथा�अब�इसे��व�नयम�के�तहत


�कया�जाना�था।�यह��स�वल�सेवा��का�ज�म�था��जसम��1854 से�खुली���त�पधा��लाई�गई।�

QUESTION 47.

IASbaba
Web: http://ilp.iasbaba.com/ Score:
Email: ilp@iasbaba.com 0.00 / 200
Page 107
2019 - Test 4-
Exam Title :
History & Cu...
Email : ilbsnaa4@gmail.com
Contact :

�न�न�ल�खत�म��से�कौन-सी��पगत�के��स�ांत(Doctrine of Lapse Policy) क���वशेषताएं�थी?

1. इस��स�ांत�के�अनुसार��कसी�शासक�के��ारा�गोद��लए�गए�पु��को��स�हासन�का�उ�रा�धकारी�नह��घो�षत�नह���कया�जा�सकता�था।�

2. गोद��लया�पु��उ�रा�धकार�म��अपने�पोषक(foster) �पता�क�����गत�संप�त�और�जायदाद��ा�त�नह��करेगा।�

3. गोद��लया�पु�����टश�सरकार��ारा�उसके��पता�को�द��जा�रही�प�शन�पाने�का�अ�धकारी�नह��होगा।�

�न�न�ल�खत�म��से�सही�कूट�का�चयन�क��जए:

a) 1 और�2
b) 2 और�3
c) 1 और�3
d) उपयु���सभी�
Correct Answer: C
Your Answer:
Explanation

Solution (c)

�पगत�के��स�ांत�क���वशेषताएं�

• इसके�अनुसार, ��य��अथवा�अ��य��(एक�जागीरदार�के��प�म�) �प�से�ई�ट�इं�डया�कंपनी�के�अधीन�कोई�भी�शाही��रयासत��जसके


शासक�का�कोई�वैध�पु�ष�उ�रा�धकारी�नह��है, को�कंपनी��ारा�अपने�म���मला��लया�जाएगा।�
• इसे�लॉड��डलहौज़ी��ारा�नह��लाया�गया�था�हालां�क�उसने�ही�इसे�लेखाब���कया�था, और�अं�ेज��के��लए�अनेक���े���के�अ�ध�हण
हेतु�इसका��योग��कया�था।�
• इसके�अनुसार, भारतीय�शासक�के��कसी�गोद��लए�पु��को�सा�ा�य�का�उ�रा�धकारी�घो�षत�नह���कया�जा�सकता�था।�गोद��लया�पु�
केवल�अपने�पालक��पता�क�����गत�संप�त�और�जायदाद�का�उ�रा�धकारी�होगा।�
• गोद��लया�पु��अपने��पता��ारा��ा�त�क��जा�रही��कसी�प�शन�को�पाने�तथा�उसक��उपा�धय��के��योग�का�भी�अ�धकारी�नह��होगा।�
• इसने�भारतीय�शासक��क��अपनी�मज��का�उ�रा�धकारी��नयु��करने�क��श���को�चुनौती�द�।�

QUESTION 48.
�न�न�ल�खत�म��से�1857 के��व�ोह�के�दौरान�जाग�कता�के��सार�हेतु��वतरण��कया�गया�था?

a) �स��र�और�प��यां�
b) चपाती�और�कमल�का�फूल�
c) लाल�और�हरा�व���
d) टू ट���ई�लकड़ी(Broken stick)
Correct Answer: B
Your Answer:
Explanation

Solution (b)

चपाती�और�कमल�का�फूल�1857 के��व�ोह�का��तीक�बन�गए।�

चपाती�जाग�कता�पैदा�करने�के��लए�रात�रात�गाँव��म��बांट��गई।�लोग��ने�एक��सरे�से�चपाती�बदली।�इसने����टश�अ�धका�रय��म��एक�भय�का
माहौल�बना��दया।�

कमल�के�फूल�छावनी��े���म��डाल��दए�गए�ता�क�अ�य��सपा�हय��का�समथ�न�हा�सल��कया�जा�सके।�साथ�ही�‘सब�लाल�हो�जाएगा’ का�स�दे श
भी�भेजा�गया।�

QUESTION 49.

IASbaba
Web: http://ilp.iasbaba.com/ Score:
Email: ilp@iasbaba.com 0.00 / 200
Page 108
2019 - Test 4-
Exam Title :
History & Cu...
Email : ilbsnaa4@gmail.com
Contact :

�न�न�ल�खत�म��से�कौन-से�उ�े �य�हेतु����टश�सरकार��ारा�पील�आयोग�का�गठन��कया�गया�था?

a) 1857 के��व�ोह�के�प�ात�सेना�के�पुनस�गठन�हेतु।�
b) ई�ट�इं�डया�कंपनी�म���ा�त���ाचार�के�मु�े�के�समाधान�हेतु।�
c) भारत�म���स�वल�सेवा�परी�ा�का�संचालन�करवाने�के�उपाय��क��अनुशंसा�करने�हेतु।�
d) 1857 के��व�ोह�के�प�ात��पगत�क��गई��रयासत��उनके�पूव��शासक��को�लौटाने�हेतु।�
Correct Answer: A
Your Answer:
Explanation

Solution (a)

�सपा�हय��म��एक�और��व�ोह�को�होने�से�रोकने�के��लए����टश�सरकार�ने�सेना�के�पुनस�गठन�के�उपाय�सुझाने�हेतु�पील�आयोग�का�गठन��कया।�

पील�आयोग�म�����टश�और�भारतीय�सेना�के�उ�च�अ�धकारी�शा�मल�थे�और�इसक��अ�य�ता�यु��स�चव�मेजर-जनरल�पील�कर�रहे�थे।�इसका
उ�े �य��व�ोह�प�ात�नी�त��नमा�ण�का�था।�

इसने�अनेक�उपाय�सुझाए, जैसे– भारतीय�सेना�म��यूरोपीय�सै�नक��का�भारतीय�सै�नक��के�सापे��अनुपात�बढ़ाया�जाए, ‘फूट�डालो�और�राज


करो’ क��नी�त�क��शु�आत�करते��ए�जा�त�के�आधार�पर�बटा�लयन��का�गठन�करो, भारतीय��को�उ�च�पद�पर��ो�त�मत�करो�आ�द।�

QUESTION 50.
�न�न�ल�खत�म��से�कौन-से�कथन�वे�लोर��व�ोह�के��लए�उ�रदायी�थे?

1. न�लीय�भेदभाव�क��उप��थ�त�तथा�भारतीय�सै�नक��को�कमतर�माना�जाता�था।�

2. कण�बा�लयाँ(ear rings) व�अ�य�जा�तगत�पहचान�धारण�करना���तबं�धत�था।�

3. �सपा�हय��को�दाढ़��बनाने�और�मूंछे�छोट��करने�को�कहा�गया�था।�

4. ऐसी�लोक��य�अवधारणा��ा�त�थी��क�सभी��सपाही�धीरे-धीरे�ईसा��धम��म��प�रव�त�त�हो�जाएंगे।�

�न�न�ल�खत�म��से�कूट�का�चयन�क��जए:

a) 1 और�4
b) 1,3 और�4
c) 1,2 और�3
d) उपयु���सभी�
Correct Answer: D
Your Answer:
Explanation

Solution (d)

वे�लोर��व�ोह�(1806) के�अनेक�कारण�बताये�जाते�ह�।�कंपनी�क��सेना�म��सेवाएं�दे ने�जाते�समय�भारतीय��सपा�हय��को�अनेक�क�ठनाइय��का


अनुभव�करना�पड़ता�था।��सपा�हय��को�कंपनी�के�अधीन�सेवाएं�दे ने�के��लए��ववश��कया�जाता�था��य��क�उनके�पुराने�सभी��वामी�(दे शी
सरदार) ��य�से�गायब�हो�रहे�थे।�

स�त�अनुशासन, �वहार, नए�ह�थयार, नई��व�धयाँ�और�यू�नफाम��आ�द�सब��सपा�हय��के��लए�नए�थे।�एक�ऐसे�����के��लए�जो�एक�लंबे


समय�से�जीवन�के�पुराने�ढर��पर��टका�है, कोई�भी�नई�चीज�क�ठन�और�गलत�लगती�है।�म�ास�के�गवन�र��व�लयम�ब��ट�क�के�अनुमोदन�से
कमांडर-इन-चीफ�सर�जॉन��ेडॉक�ने�नए��क�म�क��पगड़ी�क��शु�आत�क��जो�यूरोपीय�टोपी�जैसी�लगती�थी।�कण�बा�लयाँ�व�अ�य�जा�तगत
पहचान�धारण�करना�भी���तबं�धत�था।�

IASbaba
Web: http://ilp.iasbaba.com/ Score:
Email: ilp@iasbaba.com 0.00 / 200
Page 109
2019 - Test 4-
Exam Title :
History & Cu...
Email : ilbsnaa4@gmail.com
Contact :

�सपा�हय��को�दाढ़��बनाने�और�मूंछे�छोट��करने�को�कहा�गया�था।��सपा�हय��को�लगा��क�यह�सब�उ�ह�, उनक��धा�म�क�और�सामा�जक
परंपरा��को�अपमा�नत�करने�के��लए�लाया�गया�था।�ऐसी�लोक��य�अवधारणा��ा�त�थी��क�यह�उस����या�क��शु�आत�है��जसके�तहत
सभी��सपाही�धीरे-धीरे�ईसा��धम��म��प�रव�त�त�हो�जाएंगे।�अं�ेज�भारतीय�सै�नक��को�अपने�से�कमतर�मानते�थे।�यह�एक�न�लीय�पूवा��ह�था।�

QUESTION 51.
17व��और�18व��शता�द��के�दौरान�ई�ट�इं�डया�कंपनी�का�पूव��के��ापार�पर�एका�धकार�था।�राजा��को�उनका�समथ�न�दे ने�हेतु��ाय: घूस�द�
जाती�थी।��न�न�ल�खत�म��से�कौन�से�कथन�ई�ट�इं�डया�कंपनी�और�इं�ल�ड�के�स�ाट�के�म�य�संबंध��के�संदभ��म��सही�ह�?

1. 17व��शता�द��म��कंपनी�ने�स�ाट�को�बड़ी�मा�ा�म��ऋण��दान��कए।�

2. चा�स��II ने�कंपनी�को�अनेक�चाट� र��दान��कए��जनम��क�पनी�को�फोट� �का��नमा�ण�करना, सेना�रखने�और�पूव��श��य��के�साथ�सं�ध


अथवा�यु��करने�क��अनुम�त��दान�क��गई�थी।�

3. कंपनी�को�अं�ेज�ब��तय��म��रहने�वाले�सभी�अं�ेज����य��के�संबंध�म���या�यक�काय��करने�क��अनुम�त��दान�क��गई�थी।�

�न�न�ल�खत�म��से�सही�कूट�का�चयन�क��जए:

a) 1 और�2
b) 2 और�3
c) 1 और�3
d) उपयु���सभी�
Correct Answer: D
Your Answer:
Explanation

Solution (d)

�वत: �प��

��ोत: आधु�नक�भारत�क��पुरानी�NCERT पु�तक�

QUESTION 52.
�न�न�ल�खत�म��से�कौन�सा�संगठन�राजा�राम�मोहन�राय�और�हेनरी�डेरो�जयो�आ�द�के�नेतृ�व�म��चल�रहे�सामा�जक�सुधार�आंदोलन��क�
आलोचना�करने�हेतु��था�पत��कया�गया�था?

a) परमहंस�मंडली�
b) रामकृ�ण��मशन�
c) आ�द����समाज�
d) धम��सभा�
Correct Answer: D
Your Answer:
Explanation

Solution (d)

धम��सभा�

धम��सभा�क���थापना�1829 म��कलक�ा�म��राजा�राधाकांत�दे ब��ारा�क��गई�थी।�इस�संगठन�क���थापना�मु�यत: राजा�राम�मोहन�राय�और


हेनरी�डेरो�जयो�आ�द�के�नेतृ�व�म��चल�रहे�सामा�जक�सुधार�आंदोलन��क��काट�करने�हेतु�क��गई�थी।�इस�संगठन�क���थापना�को�औप�नवे�शक
सरकार��ारा�बनाए�गए�उस�कानून�से�ग�त��मली��जसके�तहत�दे श�म��सती��था�पर���तबंध�लगा��दया�गया�था; इस�संगठन�का��मुख��यान�इस
कानून�को��नर�त�करवाने�पर�था��जसे��ह���जा�त�के�कुछ�तबक��ने�सरकार��ारा�उनके�धा�म�क�मामल��म��ह�त�ेप�क��तरह�दे खा।�

IASbaba
Web: http://ilp.iasbaba.com/ Score:
Email: ilp@iasbaba.com 0.00 / 200
Page 110
2019 - Test 4-
Exam Title :
History & Cu...
Email : ilbsnaa4@gmail.com
Contact :
QUESTION 53.
बंगाल�के�नवाब�‘�सराजु�ौला’ और�इं��लश�ई�ट�इं�डया�कंपनी�के�म�य��ई��लासी�क��लड़ाई�ने�कंपनी�को�भारत�म��राजनी�तक�श���का�एक
�मुख�दावेदार�बना��दया।��न�न�ल�खत�कथन��पर��वचार�क��जए:

1. �सराजु�ौला�धोखे�से�हारा�और��लासी�क��लड़ाई�म��शायद�ही�कोई�वा�त�वक�लड़ाई��ई।�

2. अं�ेज��ने��सराजु�ौला�के��थान�पर�मीर�जाफर�को�कठपुतली�नवाब�बनाया।�

3. मीर�जाफर�एक�यो�य��शासक�था��जसने�जन�अनुशासन�बनाए�रखने�और��वयं�को�अं�ेज��के��भाव�से�मु��करने�का��यास��कया।�

उपयु���म��से�कौन�से�कथन�स�य�:

a) 1 और�2
b) 2 और�3
c) 1 और�3
d) उपयु���सभी�
Correct Answer: A
Your Answer:
Explanation

Solution (a)

अं�ेज��ने�मीर�जाफर�को�कठपुतली�नवाब�बनाया।�हालां�क�मीर�जाफर�अपने�नए��वा�मय��के�लालच�को�संतु��करने�म��स�म�नह��था।�कई
अवसर��पर�अं�ेज���ारा�मीर�जाफर�का�अपमान��कया�गया�और�तब�अंतत: उसे��स�हासन�से�हटाकर�मीर�का�सम�को�उस�पर�बैठाया�गया।�

मीर�का�सम�एक�यो�य��शासक�था�और�उसने��वयं�को�अं�ेज��के��भाव�से�मु��करने�का��यास��कया।�इसका�प�रणाम�ब�सर�क��लड़ाई�के
�प�म��सामने�आया।�

QUESTION 54.
चतुथ��आं�ल-मैसूर�यु��अं�ेज��और�मैसूर�के�म�य�एक��नणा�यक�यु��था।�ट�पू�सु�तान�अपने�शहर�क��र�ा�करते��ए�मारा�गया।�चतुथ��आं�ल-
मैसूर�यु��के�संदभ��म���न�न�ल�खत�म��से�कौन-से�कथन�स�य�ह�?

1. मैसूर�पर�अं�ेज�, मराठ��और�हैदराबाद�के��नजाम��ारा�चार��ओर�से�हमला��कया�गया।�

2. श�ु�सेना�और�ट�पू�क��सेना�के�म�य�का�अनुपात�4:1 था।�

3. अं�ेज��ने�लगभग�स�पूण��मैसूर�का��वलय�कर��लया।�मु�य��े��को�ट�पू�के�बड़े�पु��को�दे ��दया�गया��जस�पर�उसके�वंशज��ने�1947 तक
शासन��कया।�

�न�न�ल�खत�म��से�सही�कूट�का�चयन�क��जए:

a) 1 और�2
b) 2 और�3
c) 1 और�3
d) उपयु���सभी�
Correct Answer: A
Your Answer:
Explanation

Solution (a)

चतुथ��आं�ल-मैसूर�यु��(1799) म��ट�पू�सु�तान�क��मृ�यु�हो�गई�और�मैसूर�के��े��को�घटा��दया�गया।��ांसी�सय��के�साथ�मैसूर�के�गठबंधन
को�ई�ट�इं�डया�कंपनी�ने�एक�खतरे�के��प�म��दे खा�और�मैसूर�पर�सभी�चार��दशा��से�हमला�कर��दया�गया।�श�ु�सेना�और�ट�पू�क��सेना�के

IASbaba
Web: http://ilp.iasbaba.com/ Score:
Email: ilp@iasbaba.com 0.00 / 200
Page 111
2019 - Test 4-
Exam Title :
History & Cu...
Email : ilbsnaa4@gmail.com
Contact :
बीच�का�अनुपात�4:1 था।�मैसूर�के�पास�35,000 सै�नक�थे�जब�क�अं�ेज��के�पास�60,000 क��सेना�थी।�मराठ��और�हैदराबाद�के��नजाम
ने�उ�र�से�हमला��कया।��ीरंगप�नम�के�घेरे�(1799) म��अं�ेज��क���नणा�यक�जीत��ई।�ट�पू�सु�तान�अपने�शहर�क��र�ा�करते��ए�मारा�गया।
मैसूर�के�बचे��ए�अ�धकाँश��े��को�अं�ेज�, �नजाम�और�मराठ����ारा��मला��लया�गया।�मैसूर�और��ीरंगप�नम�के�आस-पास�बचे��ए��े��को
वाडयार�वंश�के�भारतीय�राजा�को�स�प��दया�गया��जनके�पूव�ज�हैदर�अली�के�शासक�बनने�से�पूव��वहाँ�के�वा�त�वक�शासक�थे।�वाडयार�वंश�ने
इस�बचे-खुचे�मैसूर�रा�य�पर�1947 तक�शासन��कया�और�त�प�ात�इसका��वलय�भारत�संघ�म��कर��दया।�

QUESTION 55.
�न�न�ल�खत�म��से�कौन-से�क़ानून��ारा�भारत�म��ई�ट�इं�डया�कंपनी�के��ापा�रक��एका�धकार�को�समा�त��कया�गया?

a) 1793 का�चाट� र�ए�ट�


b) 1813 का�चाट� र�ए�ट�
c) 1833 का�चाट� र�ए�ट�
d) 1858 का�भारत�सरकार�अ�ध�नयम�
Correct Answer: B
Your Answer:
Explanation

Solution (b)

1793 के�आरं�भक�चाट� र�ए�ट�ने�ई�ट�इं�डया�कंपनी�को�पूव��के�साथ��ापार�का�बीस�वष�य�एका�धकार�दे ��दया�था।�हालां�क, नेपो�लयन


बोनापाट� �के�उदय�से�इं�ल�ड�के��ापा�रय��के�मु��कल��दन�शु��हो�गए�थे।�

1813 के�चाट� र�ए�ट�ने�ई�ट�इं�डया�कंपनी�के�एका�धकार�को�समा�त��कया, हालां�क�चीन�के�साथ��ापार�और�भारत�के�साथ�चाय�के��ापार


म��कंपनी�का�एका�धकार�कायम�रहा।�इस��कार, चाय�को�छोड़कर�भारत�के�साथ��कया�जाने�वाला�व�तु��ापार�सभी����टश�जन��के��लए
खोल��दया�गया।�यह�1833 तक�चला�जब�अगले�चाट� र�ए�ट�ने�कंपनी�के�इस�एका�धकार�को�भी�समा�त�कर��दया।�

QUESTION 56.
भू�राज�व�के��थायी�बंदोब�त�के�संदभ��म���न�न�ल�खत�कथन��पर��वचार�क��जए:

1. वारेन�हे��टं �स�ने�इसे�बंगाल�और��बहार�म��लागू��कया।�

2. जम�दार��को�भू�वामी�बनाया�गया�तथा�इनके��वा�म�व�को�वंशानुगत�व�ह�तांतरणीय�बनाया�गया।�

3. जोतदार��को�कमजोर�कर�का�तकार�बना��दया�गया।�

4. जम�दार��को�एक��त�लगान�का�50% रा�य�को�दे ना�था�और�शेष�50% अपने�पास�रखना�था।�

उपयु���म��से�कौन�से�कथन�स�य�ह�?

a) 2 और�3
b) 1,2 और�3
c) 2,3 और�4
d) उपयु���सभी�
Correct Answer: A
Your Answer:
Explanation

Solution (a)

बंगाल�का��थायी�बंदोब�त�गवन�र-जनरल�लॉड��कान�वा�लस�के�नेतृ�व�म��ई�ट�इं�डया�कंपनी��ारा�1793 म���कया�गया�था।�यह�असल�म��कंपनी
और�जम�दार��के�बीच�भू�म�राज�व�को��नयत�करने�संबंधी�एक�समझौता�था।�आरंभ�म��बंगाल, �बहार�और�ओ�डशा�म���कया�गया�यह�बंदोब�त
आगे�चलकर�म�ास��ेसीड�सी�और�वाराणसी�के��जल��म��भी��योग��कया�गया।�कान�वा�लस, इं�ल�ड�म���च�लत�भू-राज�व��णाली�से��े�रत
होकर�यह��णाली�लेकर�आया�जहाँ�भू�वामी�अपनी�जमीन��के��थायी�मा�लक�थे�और�वे�का�तकार��से�लगान�एक���कया�करते�थे�और�उनके

IASbaba
Web: http://ilp.iasbaba.com/ Score:
Email: ilp@iasbaba.com 0.00 / 200
Page 112
2019 - Test 4-
Exam Title :
History & Cu...
Email : ilbsnaa4@gmail.com
Contact :
�हत��का��यान�रखते�थे।�उसने�भारत�म��भू�वा�मय��के�एक�वंशानुगत�वग��के��नमा�ण�का��व��दे खा।�इस��णाली�को�जम�दारी��व�था�भी�कहा
गया।�

जम�दार��को�उगाहे�गए�लगान�का�10/11 भाग�रा�य�को�दे ना�था�और�1/11 भाग��वयं�के��लए�रखना�था।�

QUESTION 57.
1857 का��व�ोह����टश�सा�ा�य�के��लए�एक�झटके�क��तरह�था।�चूँ�क�यह�एक�सै�य��व�ोह�क��तरह��ार�भ��आ�था, अत: ऐसी�प�र��थ�त
को�दोबारा�होने�से�रोकने�के��लए�सेना�म��मह�वपूण��प�रवत�न��कए�गए।��न�न�ल�खत�म��से�कौन-से�कथन�भारतीय�सेना�म���कए�गए�प�रवत�न��के
संदभ��म��स�य�ह�?

1. सेना�म��यूरोपीय�सै�नक��का�भारतीय�सै�नक��क��तुलना�म��अनुपात�बढ़ाया�गया।�

2. यूरोपीय�सै�नक��को�मह�वपूण��भौगो�लक�और�सै�य�पद��पर�रखा�गया।�

3. जा�त�और�धम��के�आधार�पर�बटा�लयन��का�गठन�कर�‘फूट�डालो�और�राज�करो’ क��नी�त�का�काया��वयन��कया�गया।�

4. भारतीय��को�अ�धकारी�कॉ�स��से�बाहर�रखने�क��नी�त�का��याग��कया�गया�और�राजभ��भारतीय��को�उ�च�पद��दान��कए�गए।�

�न�न�ल�खत�म��से�सही�कूट�का�चयन�क��जए:

a) 2,3 और�4
b) 1,2 और�3
c) 1,3 और�4
d) उपयु���सभी�
Correct Answer: B
Your Answer:
Explanation

Solution (b)

चौथा��वक�प�गलत�है��य��क�भारतीय��को�अ�धकारी�कॉ�स��से�बाहर�रखने�क��नी�त�का�स�ती�से�पालन��कया�गया�और�भारतीय��को�सूबेदार
से�ऊपर�के�पद�पर�जाने�क��अनुम�त�नह��थी।�

QUESTION 58.
त�वबो�धनी�सभा�के�संदभ��म���न�न�ल�खत�कथन��पर��वचार�क��जए:

1. इसक���थापना�रबी��नाथ�टै गोर��ारा�क��गई�थी।�

2. त�वबो�धनी�सभा�का�उ�े �य�धा�म�क�अ�वेषण�को�बढ़ाना�और�उप�नषद��के�सार�का��सार�करना�था।�

उपयु���म��से�कौन-से�कथन�स�य�है?

a) केवल�1
b) केवल�2
c) 1 और�2 दोन��
d) न�तो�1 न�ही�2
Correct Answer: B
Your Answer:
Explanation

Solution (b)

IASbaba
Web: http://ilp.iasbaba.com/ Score:
Email: ilp@iasbaba.com 0.00 / 200
Page 113
2019 - Test 4-
Exam Title :
History & Cu...
Email : ilbsnaa4@gmail.com
Contact :

इसक���थापना�1838 ,म��दे बे��नाथ�टै गोर��ारा�क��गई�थी।�इस�सभा�क���थापना�राजा�राम�मोहन�राय�के��वचार��के��सार�के��लए�क��गई


थी।�इसे�आरंभ�म��आ�द��हम�समाज�के�नाम�से�जाना�जाता�था।�त�वबो�धनी�सभा�का�उ�े �य�धा�म�क�उ�सुकता�को�बढ़ाना�और�उप�नषद��के
सार�का��सार�करना�था।�

QUESTION 59.
हाल�ही�म��भारत�सरकार�ने�पाइका��व�ोह(Paika rebellion) को��वत��ता�के��थम��व�ोह�के�तौर�पर�मा�यता��दान�क��है।��न�न�ल�खत
म��से�कौन�सा/से�कथन�इस��व�ोह�के�संदभ��म��सही�है/ह�?

1. पाइका�ओ�डशा�के�का�तकार�यो�ा�थे��ज�ह�ने�यु��के�दौरान��स�हासन�के���त�सै�य�सेवाएं��दान�क�।�

2. पाइका�लोग��ने�प�पातपूण��नी�तय��के��व����ाथ�ना��और�या�चका��जैसी�संवैधा�नक��व�धय��के�मा�यम�से�अं�ेजी�शासन�के��व��
शां�तपूण���व�ोह��कया।�

�न�न�ल�खत�म��से�सही�कूट�का�चयन�क��जए:

a) केवल�1
b) केवल�2
c) 1 और�2 दोन��
d) न�तो�1 न�ही�2
Correct Answer: A
Your Answer:
Explanation

Solution (a)

ओ�डशा�का�पाइका��व�ोह�

लोक��य��प�से�भारतीय��वत��ता�का�पहला�सं�ाम�माने�जाने�वाले�1857 के��व�ोह�से�भी�पहले�ओ�डशा�म��घ�टत�1817 के�पाइका��व�ोह


ने�थोड़े�समय�के��लए�भारत�के�पूव��भाग�म��अं�ेजी�राज�क��जड़��को��हला��दया�था।�पाइका�ओ�डशा�के�शासक��के�का�तकार�यो�ा�थे
�ज�ह�ने�यु��के�दौरान��स�हासन�के���त�सै�य�सेवाएं�द��तथा�शां�त�काल�के�समय�जुताई�का�काय���कया।�उ�ह�ने�ब�सी�जगबंधु��ब�ाधर�के
नेतृ�व�म��1817 म��अं�ेजी�जुआ�उतार�फ�कने�हेतु�अं�ेज��के��व����व�ोह�का�झंडा�बुलंद��कया।�

खुदा��के�शासक�पारंप�रक��प�से�जग�ाथ�मं�दर�के�संर�क�थे�और�पृ�वी�पर�भगवान�जग�ाथ�के�नायब�के��प�म��शासन�करते�थे।�वे�ओ�डशा
के�लोग��क��राजनी�तक�और�सां�कृ�तक��वतं�ता�के��तीक�थे।�ओ�डशा�के��मश: उ�र�व�द��ण�म����थत�बंगाल�तथा�म�ास�सूबे�म��अपना
�नयं�ण��था�पत�कर�लेने�के�प�ात�अं�ेज��ने�1803 म��ओ�डशा�पर��नयं�ण�कायम��कया।�उस�समय�ओ�डशा�के�गजप�त�राजा�मुकुंद�दे व-ll
अ�पवय�क�थे�और�उनके�संर�क�जय�राजगु���ारा��कया�गया�आरं�भक���तरोध��नम�मतापूव�क�कुचल��दया�गया�तथा�जय�राजगु��के�जी�वत
रहते�उनके�दो�टु कड़े�कर��दए�गए।�कुछ�वष��प�ात, गजप�त�राजा�क��जनसेना�के�वंशानुगत�सरदार�ब�सी�जगबंधु�के�नेतृ�व�म��पाइका�लोग��ने
�व�ोह�कर��दया�तथा�इसम��जनजातीय�लोग��एवं�समाज�के�अ�य�वग��क��सहायता�ली।��व�ोह�माच��1817 म��शु���आ�और�तेजी�से�फैला।
हालां�क�पाइका�लोग��ने�अं�ेज��के��व���इस��व�ोह�म��एक�मह�वपूण��भू�मका�अदा�क���क�तु�यह��कसी�भी��कार�एक�छोटे �से��वशेष�वग���ारा
�कया�गया��व�ोह�नह��था।�घुमुसर�(वत�मान�के�गंजाम�और�कंधमाल��जले) के�जनजातीय�लोग��और�समाज�के�अ�य�वग��ने�इसम��स��य�भाग
�लया।�असल�म�, पाइका��व�ोह�का�सबसे�भा�यशाली��ण�उस�समय�आया�जब�घुमुसर�के�4OO आ�दवासी�अं�ेजी�राज�का��वरोध�करते��ए
खुदा��म���वेश�कर�गए।�पाइका�लोग��ने�खुदा��(जहाँ�से�अं�ेज�भाग�खड़े��ए�थे) म���वेश�करते�समय����टश�स�ा�के��तीक��पर�हमला��कया,
पु�लस�थान�, �शास�नक�काया�लय��और��े ज़री�को�आग�लगा�द�।�

QUESTION 60.
अं�ेज��के��व���सबसे�आरं�भक��व�ोह�का�नेतृ�व�फक�र��और�स�या�सय���ारा��कया�गया�था��जसे�स�यासी��व�ोह�के�नाम�से�जाना�जाता�है।
यह��व�ोह�कहाँ��आ?

a) बंगाल�
b) म�ास�
c) बॉ�बे�
d) पंजाब�

IASbaba
Web: http://ilp.iasbaba.com/ Score:
Email: ilp@iasbaba.com 0.00 / 200
Page 114
2019 - Test 4-
Exam Title :
History & Cu...
Email : ilbsnaa4@gmail.com
Contact :
Correct Answer: A
Your Answer:
Explanation

Solution (a)

स�यासी��व�ोह�

स�यासी��व�ोह�18 व��शता�द��के�उ�रा�� ��म��बंगाल�म��ई�ट�इं�डया�कंपनी�के��व���स�या�सय��और�फक�र��(�ह���और�मु��लम�दोन�) क�


ग�त�व�धयाँ�थी।�इसे�स�यासी��व�ोह�के�नाम�से�जाना�जाता�है�जो�मु�श�दाबाद�एवं�जलपाईगुड़ी�के�बैकुंठपुर�जंगल��के�आस-पास��आ।
इ�तहासकार�न�केवल�इस�बात�पर�मत�भ�ता�रखते�ह���क��कन�घटना��को��व�ोह�माना�जाए�ब��क�भारतीय�इ�तहास�म��इस��व�ोह�क�
साथ�कता�को�लेकर�भी�उनक��अलग-अलग�राय�है।�जहाँ�कुछ�इसे��वदे शी�शासन�से�मु���पाने�का�आरं�भक�यु��कहते�ह�, �य��क�1764 म�
�ई�ब�सर�क��लड़ाई�के�बाद�कर�सं�ह�करने�का�अ�धकार����टश�ई�ट�इं�डया�कंपनी�को�दे ��दया�गया�था�वह��अ�य�इसे�1770 के�बंगाल�सूबे
के���भ���के�प�ात�डाके�क���ह�सक�घटना��के��प�म���ेणीब��करते�ह�।�

QUESTION 61.
�न�न�ल�खत�म��से�कौन�सा��ह�सवा��धक�गम��है?

a) बुध�
b) शु��
c) बृह�प�त�
d) मंगल�
Correct Answer: B
Your Answer:
Explanation

Solution (b)

बुध�ऐसा��ह�है�जो�सूय��के�सबसे��नकट�है�और�इस��कार�अ�धक���य��उ�मा��ा�त�करता�है, �क�तु�इसके�प�ात�भी�यह�सबसे�गम��नह��है।�शु
��इस��म�म���सरा�है�और�इसका�तापमान�संपूण���ह�पर�462 �ड�ी�से��सयस�रहता�है।�यह�सौरमंडल�का�सबसे�गम���ह�है।�

शु��पर�सवा��धक�तापमान�होने�का�कारण�इस�पर�बड़ी�मा�ा�म��ह�रत�गृह�गैस��क��उप��थ�त�है।�

QUESTION 62.
‘का�ड�लेरा’ के�संदभ��म���न�न�ल�खत�म��से�कौन-सा�कथन�स�य�है?

a) यह�दो�अथवा�अ�धक�समानांतर�पव�त��ृंखला��वाली�एक�वृहत�पव�त��ृंखला�है।�
b) यह��वालामुखी��ेटर�के��वनाश�से��न�म�त��वशाल��वालामुखीय�गत��है।�
c) यह�पव�त�घा�टय��म���न�म�त�झील�है।�
d) यह�च�ानी�म��थल��के�गत��म���न�म�त�झील�है।�
Correct Answer: A
Your Answer:
Explanation

Solution (a)

एक�का�ड�लेरा�दो�अथवा�अ�धक�समानांतर�पव�त��ृंखला��वाली�एक�वृहत�पव�त��ृंखला�होती�है।�उदाहरण, अमे�रका�का�रॉक��पव�त�और
भारत�का��हमालय�पव�त�आ�द।�

सो�चए�

• का�डेरा�

IASbaba
Web: http://ilp.iasbaba.com/ Score:
Email: ilp@iasbaba.com 0.00 / 200
Page 115
2019 - Test 4-
Exam Title :
History & Cu...
Email : ilbsnaa4@gmail.com
Contact :
QUESTION 63.
‘इंसुलर�शे�फ’ का�अथ��है-

a) भूआब��दे श��को�आवं�टत�महा��पीय�शे�फ।�
b) �कसी���प�के�चार��ओर�का�शे�फ।�
c) �न�ेप��के�जमाव�से��न�म�त�शे�फ।�
d) समु����तर�के�कम�होने�से��न�म�त�शे�फ।�
Correct Answer: B
Your Answer:
Explanation

Solution (b)

महा��पीय�शे�फ�पानी�के�नीचे�का�एक�भू�म�े��होता�है�जो�महा��प�का��व�ता�रत�भाग�होता�है।�इसके�कारण�एक�अपे�ाकृत��छछले�जल�के
�े��का��नमा�ण�हो�जाता�है��जसे�शे�फ�ए�रया�कहते�ह�।�अ�धकाँश�शे�फ��हमानी�काल��हम�युग�और�अंतर�हमानी�काल�के�दौरान�अनावृत��ए
थे।�

एक���प�के�चार��ओर�का�शे�फ�इंसुलर�शे�फ�कहलाता�है।�

�या�आप�जानते�ह��?

महा��पीय�शे�फ�का�आ�थ�क�मह�व�

• समु���भोजन�का�अ�धकाँश�भाग�महा��पीय�शे�फ�से�आता�है;
• वे�म��यन�के�समृ�तम��े��ह�;
• ये�ब�धा��वक����थय��और�पे�ो�लयम�के�संभा�वत��थल�ह�।�

सो�चए�!

• अंतरा��ीय�सीबेड�अथॉ�रट��

QUESTION 64.
�र�ट�घा�टय��के�संदभ��म���न�न�ल�खत�म��से�कौन-सा�कथन�स�य�है?

1. इनका��नमा�ण�महा��प-महा��प�अपसरण�के�कारण�होता�है।�

2. ये�समतल�तलहट��वाली��थलाकृ�त�है।�

3. �हमालयी�न�दयाँ�अपनी�वृ�ाव�था(old age) म���र�ट�घा�टय��का��नमा�ण�करती�ह�।�

�न�न�ल�खत�म��से�कूट�सही�का�चयन�क��जए:

a) 1 और�2
b) 2 और�3
c) 1 और�3
d) उपयु���सभी�
Correct Answer: A
Your Answer:
Explanation

Solution (a)

�र�ट�घा�टयाँ�

IASbaba
Web: http://ilp.iasbaba.com/ Score:
Email: ilp@iasbaba.com 0.00 / 200
Page 116
2019 - Test 4-
Exam Title :
History & Cu...
Email : ilbsnaa4@gmail.com
Contact :

एक��र�ट�घाट��अनेक�उ�चभू�मय��अथवा�पव�त��ृंखला��के�बीच�रै�खक�आकार�क���न�नभू�म�है��जसका��नमा�ण�भूग�भ�क��र�ट�अथवा��ंश
से�होता�है।�एक��र�ट�घाट��का��नमा�ण�एक�अपसारी��लेट�सीमा�पर�होता�है, जो�भूपप�ट�य��व�तार�अथवा�सतह�का�अलग�होना�होता�है।�इसे
आगे�चलकर�अपरदन�श��य���ारा�और�भी�गहरा�बना��दया�जाता�है।�जब�तनाव�बल�इतने�श��शाली�होते�ह���क��लेट�को�तोड़�कर�अलग�कर
सक�, तब�एक�स�टर��लॉक�दो��लॉ�स�के��कनार��के�साथ-साथ�इनके�बीच��गरता�है�और�एक��ाबेन�का��नमा�ण�होता�है।�क���के��गरने�से��र�ट
घाट��(जब�यह�नई�है) क��लगभग�समानांतर�खड़ी�द�वार��का��नमा�ण�हो�जाता�है।�यह��वशेषता��र�ट�घाट��क��शु�आत�है, �क�तु�जैसे-जैसे
���या�जारी�रहती�है, घाट��क��चौड़ाई�तब�तक�बढ़ती�है�जब�तक�यह�एक��वशाल��ोणी�न�बन�जाए�और��र�ट�क��द�वार��व�आस-पास�के��े��
से��गरने�वाले�अवसाद��से�न�भर�जाए।�इस����या�का�सव��म��ात�उदाहरण�है�पूव��अ��क���र�ट।�

नोट�: �हमालयी�न�दयाँ��र�ट�घाट��का��नमा�ण�नह��करती।��र�ट�घाट��एक���तीयक�भूआकृ�त�है�तथा�इसका��नमा�ण�आंत�रक�बल���ारा�होता
है�न��क�अपरदन�बल���ारा।�

QUESTION 65.
तांगा�नका, मलावी�और�मृत�सागर�उदाहरण�ह�-

a) मानव��न�म�त�झील�के�
b) �ेटर�झील�के�
c) �र�ट�घाट��झील�के�
d) �वालामुखीय�झील�के�
Correct Answer: C
Your Answer:
Explanation

Solution (c)

�र�ट�घाट��झील��

• एक��र�ट�घाट��तब�बनती�है�जब�पृ�वी�के�दो��लॉक�एक��सरे�से��र�हटते�ह��और�बीच�के��लॉक�को�नीचे�क��तरफ��खसकने�दे ते�ह�।�या
�फर�इसे�दो�समानांतर��ंश��के�बीच�का�धंसा��आ�भूभाग�कह�सकते�ह�।�
• �र�ट�घा�टयाँ�गहरी, संकरी�और�लंबी�होती�ह�।�अत: इसके�साथ�बनने�वाली�झील��भी�गहरी, संकरी�और�ब�त�लंबी�होती�ह�।�
• जल�गत��(�र�ट�म����थत�घाट�) म��एक��होता�है�और�इसक��सतह��सागर�तल�से��ाय: नीची�होती�ह�।�
• इसका�सव��म��ात�उदाहरण�ई�ट�अ��कन��र�ट�घाट��है�जो�ज़ा��बया, मलावी, तंज़ा�नया, के�या�और�इ�थयो�पया�म���व�तृत�है�और
इजराइल�तथा�जॉड�न�के�मृत�सागर�के�साथ-साथ�3,000 मील�क��कुल��री�तक�फैली�है।�
• इसम��तांगा�नका�, मलावी, �डो�फ, एडवड�, अ�बट� �जैसी�झील��और�मृत�सागर�भी�शा�मल�है�जो�औसत�समु��तल�से�1,28
6 फ�ट�नीचे�है�और��व��क��सबसे��न�नतम��ब���पर���थत�झील�है�।

QUESTION 66.
�न�न�ल�खत�यु�म��पर��वचार�क��जए-

पठार� �मुख�ख�नज�

1. �क�बल�� �वण��

2. कटं गा� तांबा�

3. लारे�शयन� लौह�अय�क�

4. मे��सकन� चांद��

उपयु���म��से�कौन�सा/से�यु�म�सही�सुमे�लत�है/ह�?

IASbaba
Web: http://ilp.iasbaba.com/ Score:
Email: ilp@iasbaba.com 0.00 / 200
Page 117
2019 - Test 4-
Exam Title :
History & Cu...
Email : ilbsnaa4@gmail.com
Contact :

a) केवल�1 और�3
b) केवल�1 और�4
c) केवल�2 और�4
d) उपयु���सभी�
Correct Answer: D
Your Answer:
Explanation

Solution (d)

�क�बल��पठार�

• ऑ��े �लया�के�उ�री�भाग�म����थत�है।�
• यह�पठार��वालामुखीय�उदगार��से��न�म�त�है।�
• यहाँ�लौह, �वण�, लेड, ज�ता, चांद��और�हीरे�जैसे�अनेक�ख�नज�पाए�जाते�ह�।�
• यहाँ�हीरे�पाए�जाते�ह�।�

कटं गा�पठार�

• यह�कांगो�म��है।�
• यह�ता�बे�के�उ�पादन�के��लए���स��है।�
• कोबा�ट, यूरे�नयम, ज�ता, चांद�, �वण��और��टन�जैसे�अ�य�ख�नज�भी�यहाँ��मलते�ह�।�

लारे�शयन�पठार�

• यह�कनाडा�के�पूव��भाग�म��है�और�कनाडाई�शी�ड�का��ह�सा�है।�
• यहाँ�उ�म��ेणी�का�लौह�अय�क��मलता�है।�

मे��सकन�पठार�

• इसे�‘ख�नज�भंडार’ कहा�जाता�है।�चांद�, तांबे�जैसे�अनेक�धा��वक�ख�नज�यहाँ�से��ा�त�होते�ह�।�


• �व��क��सबसे�बड़ी�चांद��क��खान��च�आ�आ�इस�पठार�पर���थत�है।�

�या�आप�जानते�ह��?

पैटागो�नयाई�पठार�

• यह�एक��पडमांट�पठार�(शु�क�भूआकृ�त) है�और�अज�ट�ना�के�द��णी�भाग�म��पड़ता�है।�
• यह�एक�वृ���छाया�म��थलीय�पठार�है।�
• यह�भेड़�पालन�हेतु�एक�मह�वपूण���े��है।�

सो�चए�!

• समतल�मैदान��के��कार�

QUESTION 67.
�न�न�ल�खत�म��से�कौन-सी�प�र��थ�तयां�डे�टा�के��नमा�ण�हेतु�आव�यक�ह�?

1. नद��म��अवसाद�क���वशाल�मा�ा�होनी�चा�हए।�

2. नद��के�माग��म���वशाल�झील��क��उप��थ�त।�

3. समीप��छछले�सागर�अथवा�महा��पीय�शे�फ�क��उप��थ�त।�

4. नद�-मुख�के�समकोण�पर��वा�हत�मजबूत�धाराएं।�

IASbaba
Web: http://ilp.iasbaba.com/ Score:
Email: ilp@iasbaba.com 0.00 / 200
Page 118
2019 - Test 4-
Exam Title :
History & Cu...
Email : ilbsnaa4@gmail.com
Contact :

नीचे��दए�गए�कूट�से�सही�उ�र�चु�नए-

a) केवल�1 और�4
b) केवल�1 और�3
c) केवल�1, 2 और�3
d) उपयु���सभी�
Correct Answer: B
Your Answer:
Explanation

Solution (b)

डे�टा�के��नमा�ण�हेतु��न�न�ल�खत�प�र��थ�तयां�आव�यक�ह�।�

• नद��म���वशाल�मा�ा�म��गाद�होनी�चा�हए।�यह�तभी�संभव�होगा�जब�इसके�ऊपरी�और�म�य�चरण��म��स��य�अपरदन�है।�
• म�यवत��चरण�म��वृहत�मा�ा�म���न�ेप�होना�चा�हए।�उदा. बीच�म��झील�क��उप��थ�त�अथवा�उ�च�वा�पीकरण�क��दर�।
• नद���ारा�गाद��न�ेप�क��दर�धारा��और��वारीय���याकलाप���ारा�उ�ह��हटाए�जाने�क��दर�से�अ�धक�होनी�चा�हए�अथा�त�नद��के�मुख
के�समकोण�पर�कोई�मजबूत�धारा�नह��होनी�चा�हए।�
• साथ�लगते��छछले�सागर�अथवा�महा��पीय�शे�फ�क��उप��थ�त।�
• नद��का�वेग�पया��त��प�से�कम�होना�चा�हए�और�नद���ारा�लाई�जाने�वाली�गाद�इसके�मुख�पर��न�े�पत�होनी�चा�हए।�

�या�आप�जानते�ह��?

एक�डे�टा�का��नमा�ण�दो����या��के�संयोग�से�होता�है�:

• सागर�अथवा�म���वेश�के�समय�इसक��ग�त�घट�जाने�के�चलते�नद��क��गाद�वहन�करने�क���मता�का�घट�जाना�तथा�
• नद���ारा�बहा�कर�लाए�गए�मृ��का�कण�लवणीय�जल�क��उप��थ�त�म��जम�जाते�ह��और��न�े�पत�हो�जाते�ह�।�
• सबसे�महीन�कण�सबसे�आगे�तक�ले�जाए�जाते�ह��और�तलहट��पर�एक��होने�लगते�ह�।�

सो�चए�!

• डे�टा��के��कार�

QUESTION 68.
�ाथ�मक�(P) तरंगे�भूकंप�के�दौरान��न�म�त�तरंगे�ह�।�P तरंग��का��कार�ह�?

a) अनु��थ�
b) �व�ुत�चु�बक�य�
c) �व�न�
d) अनुदै�य��
Correct Answer: D
Your Answer:
Explanation

Solution (d)

P तथा�S तरंगे�

एक�तरंग�एक��पंदन�होती�है�जो�पदाथ��(ठोस, �व�अथवा�गैस) को�ह�तांत�रत��कए��बना�एक��थान�से��सरे��थान�पर�उजा��का�ह�ता��ण�करती


है।��काश�और��व�न�दोन��इस��कार�गमन�करते�ह�।�

भूकंप�के�दौरान��नमु���उजा��पृ�वी�के�चार��ओर�तरंग��के��प�म��गमन�करती�है।�दो��कार�क��भूक�पीय�तरंगे�होती�ह�- P- तथा�S-तरंगे।�वे
पृ�वी�से�मा�यम�से�गमन�करने�के�तरीके�म���भ��होती�ह�।�

IASbaba
Web: http://ilp.iasbaba.com/ Score:
Email: ilp@iasbaba.com 0.00 / 200
Page 119
2019 - Test 4-
Exam Title :
History & Cu...
Email : ilbsnaa4@gmail.com
Contact :

P- तरंगे�(P से�आशय��ाथ�मक�से�है) सूचक�पर�सबसे�पहले�प�ँचती�ह�।�ये�अनुदै�य��तरंगे�होती�ह���जसका�अथ��है��पंदन�का�ग�त�क���दशा�म�


होना।�अनुदै�य��तरंग��के�अ�य�उदाहरण��म���व�न�तरंगे�और�एक�ख�चे��ए����ंग�म��मौजूद�तरंगे�आ�द�आती�ह�।�

S- तरंगे�(S से�आशय���तीयक�से�है) �कसी��स�मो�ाफ�के�सूचक�पर��सरे�न�बर�पर�प�ँचती�ह�।�ये�अनु��थ�तरंगे�ह���जसका�अथ��है��पंदन


का�गमन�क���दशा�से�समकोण�पर�होना।�अनु��थ�तरंग��का�अ�य�उदाहरण�है��काश�तरंगे�और�जल�तरंगे।�

QUESTION 69.
‘शी�ड��कार’ के��वालामुखी�के�संदभ��म���न�न�ल�खत�म��से�कौन-सा�कथन�स�य�है?

1. इनका��नमा�ण�अ�यंत�तरल�बेसा��टक�लावा�के�ठोस�होने�से�होता�है।�

2. इन��वालामु�खय��म��भयानक�तरीके�से��व�फोट�होता�है��जससे�लावा�और�अ�य�पदाथ��काफ��ऊँचाई�तक�जाते�ह�।�

3. उनक��ढाल�ती��होती�है।�

�न�न�ल�खत�म��से�सही�कूट�का�चयन�क��जए:

a) केवल�1
b) 2 और�3
c) 1 और�3
d) उपरो��म��से�कोई�नह��
Correct Answer: A
Your Answer:
Explanation

Solution (a)

शी�ड��कार�क���वालामुखीय�भूआकृ�तयाँ�

• हवाई���प���थत��वालामुखी�इसके�सबसे���स��उदाहरण�ह�।�
• ये��वालामुखी�अ�धकांशत: बेसा�ट�के�बने�होते�ह�, जो��क�लावे�का�ऐसा��कार�है�जो�उदगार�के�समय�अ�यंत�तरल�होता�है।�
• ये��वालामुखी�खड़ी�ढलान�वाले�नह��होते।�
• जल�के��कसी��कार��छ��के�भीतर�चले�जाने�पर�ये��व�फोटक�बन�जाते�ह�; अ�यथा�ये�कम��व�फोटक�होते�ह�।�
• उदाहरण�: मोना�लोआ�( हवाई�) ।

QUESTION 70.
�न�न�ल�खत�म��से�कौन-सी�भू-आकृ�तयां�महासागरीय-महासागरीय�अ�भसरण�से�संबं�धत�है?

1. महासागरीय�गत��

2. महासागरीय��रज�

3. �वालामुखीय���प��ृंखला�

4. व�लत�पव�त�

�न�न�ल�खत�म��से�सही�कूट�का�चयन�क��जए:

a) 1 और�2
b) 2, 3 और�4
c) 1 और�3
d) उपयु���सभी�

IASbaba
Web: http://ilp.iasbaba.com/ Score:
Email: ilp@iasbaba.com 0.00 / 200
Page 120
2019 - Test 4-
Exam Title :
History & Cu...
Email : ilbsnaa4@gmail.com
Contact :
Correct Answer: C
Your Answer:
Explanation

Solution (c)

नोट�: �न�न�ल�खत�सारणी�को��यानपूव�क�प�ढ़ए।�य�द�आप�इस�सारणी�को�समझ�और�याद�रख�सकते�ह�, तब�आप�सभी���तीयक


उ�चावच��के��नमा�ण�क���ा�या�करने�म��स�म�ह�गे�।

�लेट�सीमा� �लेट�ग�त�व�ध� सागरतल� दे खी�जाने�वाली उदाहरण�


घटनाएं�

अपसारी��लेट�सीमा� महासागर- अलग� सागरतल�के��सार �सार�के����पर��रज म�य�अटलां�टक


महासागर� से��न�म�त� �ा�प।��लेट�का��े� �रज, पूव�
बढ़ता�है।�अनेक�छोटे �शांतीय�उभार�
�वालामुखी�तथा�भूकंप।

महा��प- महा��प��वखंडन महा��प�एक��सरे�से ई�ट�अ��कन


महा��प� से�नई�महासागरीय �र�हटते�ह�, महासागर �र�ट�
�ोणी�का��नमा�ण बलपूव�क�अ�त�मण
हो�सकता�है।� कर�सकते�ह�, �र�ट
घा�टय��और��लॉक
पव�त��का��नमा�ण।�

अ�भसारी��लेट�सीमा� महासागर- साथ�म�� �न�न�खलन��े� कम�घनी�महा��पीय प��मी�द��ण


महा��प� (सबड�शन�ज़ोन) पप�ट��के�नीचे�घनी अमे�रका�
म���वनाश� महासागरीय
�थलमंडलीय��बक�।
जैसे-जैसे�यह
�ब�लतामंडल�म��उतरती
है, भूकंप�नीचे�जा�रही
�लेट�का�माग��खोजते
ह�’।�एक�गत��का��नमा�ण
होता�है।��न�न�ख�लत
�लेट�आं�शक��प�से
�पघलती�है�तथा
महा��पीय
�वालामु�खय��के
�नमा�ण�हेतु�मै�मा�ऊपर
उठता�है।�

महासागर- �न�न�खलन��े� अ�धक�घन�व�वाली अ�यू�शयन�


महासागर� (सबड�शन�ज़ोन) पप�ट��ह�क��पप�ट��म�
म���वनाश� घुसती�है�और
�न�न�ख�लत�होकर�एक
व��य�गत��और�एक
�वालामुखीय��चाप�का
�नमा�ण�करती�है।�

IASbaba
Web: http://ilp.iasbaba.com/ Score:
Email: ilp@iasbaba.com 0.00 / 200
Page 121
2019 - Test 4-
Exam Title :
History & Cu...
Email : ilbsnaa4@gmail.com
Contact :

महा��प- NA �वशाल�महा��पीय �हमालय, आ��स�


महा��प� �थलमंडल��के��े���के
बीच�ट�कर।�दोन��म��से
कोई�भी��े���न��ख�लत
नह��होता।��लेट�के
�कनारे�संपी�डत�हो�जाते
ह�, व�लत�हो�जाते�ह�
और�ऊपर�उठ�जाते�ह�।�

�पा�तरकारी��लेट�सीमा� एक��सरे�के�पास न�तो��नमा�ण�होता एक��पा�तरकारी��ंश सेन�एं��यास


से�गुजर�जाती�ह�।� है, न�ही��वंस� का��नमा�ण�तब�होता�है �ंश।�
जहाँ��लेट��एक��सरे�के
पास�से�गुजर�जाती�ह�।
�ंश�के�साथ-साथ
ती�ता�वाले�भूकंप।�

QUESTION 71.
�न�न�ल�खत�म��से�कौन-से�प�रवत�न�भारतीय��वतं�ता�अ�ध�नयम�1947 �ारा�सं�वधान�सभा�म���कए�गए�थे?

1. सभा�को�एक�पूण�तया�सं�भु�इकाई�बना��दया�गया�और�इस�अ�ध�नयम�ने�इस�सभा�को�भारत�के�संदभ��म�����टश�संसद��ारा�बनाए�गए
�कसी�भी�कानून�को�समा�त�अथवा�संशो�धत�करने�क��श����दान�क�।�

2. सभा�को�दो�अलग-अलग�काय���न�द����कए�गए�अथा�त��वतं��भारत�हेतु�सं�वधान�का��नमा�ण�और�दे श�के��लए�साधारण��वधान��का
अ�ध�नयमन।�

�न�न�ल�खत�म��से�सही�कूट�का�चयन�क��जए�

a) केवल�1
b) केवल�2
c) 1 और�2 दोन��
d) न�तो�1 न�ही�2
Correct Answer: C
Your Answer:
Explanation

Solution (c)

भारतीय��वतं�ता�अ�ध�नयम�1947 �ारा�सं�वधान�सभा�क����थ�त�म���न�न�ल�खत�तीन�प�रवत�न��कए�गए:

1. सभा�को�एक�पूण��सं�भु�इकाई�बना��दया�गया�जो�अपनी�मज��के�सं�वधान�का��नमा�ण�कर�सकती�थी।�इस�अ�ध�नयम�ने�इस�सभा�को�भारत
के�संदभ��म�����टश�संसद��ारा�बनाए�गए��कसी�भी�कानून�को�समा�त�अथवा�संशो�धत�करने�क��श����दान�क�।�

2. यह�सभा�एक��वधान��नमा��ी�सं�था�भी�बन�गई।�अ�य�श�द��म�, सभा�को�दो�अलग-अलग�काय���दए�गए, अथा�त��वतं��भारत�के��लए


सं�वधान�का��नमा�ण�और�दे श�हेतु�साधारण��वधान��का�अ�ध�नयमन।�ये�दोन��काय��अलग-अलग��दवस��म���कए�जाने�थे।�इस��कार, यह�सभा
�वतं��भारत�(डो�म�नयन�ले�ज�लेचर) क��पहली�संसद�बनी।�जब�भी�यह�सभा�सं�वधान�सभा�के�तौर�पर�समवेत�होती�थी, इसक��अ�य�ता
डॉ. राजे����साद�करते�थे, तथा�जब�यह�एक��वधा�यका�के�तौर�पर�समवेत�होती�थी, इसक��अ�य�ता�जी�वी�मावलंकर�करते�थे।�ये�दोन��काय�
26 नवंबर�1949 को�सं�वधान�के��नमा�ण�का�काय��पूरा�होने�तक�जारी�रहे।�

QUESTION 72.
�न�न�ल�खत�म��से�कौन-सी��मुख�स�म�त�क��अ�य�ता�जवाहरलाल�नेह��ने�नह��क�?

IASbaba
Web: http://ilp.iasbaba.com/ Score:
Email: ilp@iasbaba.com 0.00 / 200
Page 122
2019 - Test 4-
Exam Title :
History & Cu...
Email : ilbsnaa4@gmail.com
Contact :

a) संघ�क��श��य��संबंधी�स�म�त�
b) संघीय�सं�वधान�स�म�त�
c) मौ�लक�अ�धकार��पर�परामश��स�म�त�
d) रा�य�स�म�त�
Correct Answer: C
Your Answer:
Explanation

Solution (c)

�मुख�स�म�तयां�

1. संघ�क��श��य��संबंधी�स�म�त– जवाहरलाल�नेह��

2. संघीय�सं�वधान�स�म�त– जवाहरलाल�नेह��

3. �ा�त��के�गठन�संबंधी�स�म�त– सरदार�पटे ल�

4. �ा�प�स�म�त– डॉ. बी. आर. अंबेडकर�

5. मौ�लक�अ�धकार�, अ�पसं�यको�और�आ�दवा�सय��तथा�छोड़��दए�गए��े���पर�परामश��स�म�त– सरदार�पटे ल।�इस�स�म�त�क�


�न�न�ल�खत�उप-स�म�तयां�थी:

(a) मौ�लक�अ�धकार��संबंधी�उप- स�म�त– जे. बी. कृपलानी�

(b) अ�पसं�यक��हेतु�उप- स�म�त– एच. सी. मुख़ज��

(c) पूव��र��ं�टयर�जनजातीय��े��और�असम�से�बाहर�रखे�गए�व�अंशत: बाहर�रखे�गए��े���संबंधी�उप- स�म�त�

– गोपीनाथ�बारदोलाई�

(d) बाहर�रखे�गए�अथवा�अंशत: बाहर�रखे�गए�(असम�के�अलावा) �े���संबंधी�उप- स�म�त�– ए.वी. ठ�कर�

6. ���या�के��नयम��संबंधी�स�म�त– डॉ. राजे����साद�

7. रा�य��सबंधी�स�म�त�(रा�य��से�वाता��करने�हेतु�स�म�त) –जवाहरलाल�नेह��

8. �ट�य�र�ग�स�म�त– डॉ. राजे����साद�

QUESTION 73.
�न�न�ल�खत�म��से�कौन-से�श�द�44व��सं�वधान�संशोधन��ारा�उ�े �शका�म��जोड़े�गए?

1. समाजवाद��

2. पंथ��नरपे��

3. अखंडता�

�न�न�ल�खत�म��से�सही�कूट�का�चयन�क��जए:

a) 1 और�2
b) 2 और�3
c) 1 और�3
d) उपयु���म��से�कोई�नह��

IASbaba
Web: http://ilp.iasbaba.com/ Score:
Email: ilp@iasbaba.com 0.00 / 200
Page 123
2019 - Test 4-
Exam Title :
History & Cu...
Email : ilbsnaa4@gmail.com
Contact :
Correct Answer: D
Your Answer:
Explanation

Solution (d)

ये�सभी�श�द�44 व��क��बजाए�42 व��सं�वधान�संशोधन��ारा�जोड़े�गए�थे।�

QUESTION 74.
सं�वधान�का�अनु�छे द�3, संसद�को��कसी�रा�य�से��कसी��े��को�पृथ�क�कर�अथवा�दो�या�उससे�अ�धक�रा�य��अथवा�रा�य��के�भाग��को
जोड़कर�अथवा��कसी�रा�य�के��कसी�भाग�म��कोई��े��जोड़कर�नए�रा�य�के��नमा�ण�क��श����दान�करता�है।�इस�संबंध�म���न�न�ल�खत�म��से
कौन�सा�कथन�स�य�है?

1. ऐसे��कसी�प�रवत�न�का���ताव�करने�वाला��वधेयक�रा�प�त�क��पूव��अनुम�त�के�प�ात�ही�संसद�म��पुर:�था�पत��कया�जा�सकता�है।�

2. रा�प�त�को�यह��वधेयक�संबं�धत�रा�य��वधानमंडल�को��न��त�समयाव�ध�म��उसके��वचार���तुत�करने�हेतु�संद�भ�त�करना�होता�है।�

3. रा�य�क��सीमा�प�रवत�न�के�मामले�म��रा�प�त�संबं�धत�रा�य�के��वधानमंडल�क���सफा�रश��मानने�के��लए�बा�य�है।�

�न�न�ल�खत�म��से�सही�कूट�का�चयन�क��जए:

a) 1 और�2
b) 2 और�3
c) 1 और�3
d) उपयु���सभी�
Correct Answer: A
Your Answer:
Explanation

Solution (a)

अनु�छे द�3 संसद�को��न�न�ल�खत�करने�क��श���दे ता�है�:

• �कसी�रा�य�से��कसी��े��को�अलग�कर�अथवा�दो�या�उससे�अ�धक�रा�य��अथवा�रा�य��के�भाग��को�जोड़कर�अथवा��कसी�रा�य�के
�कसी�भाग�म��कोई��े��जोड़कर�नए�रा�य�का��नमा�ण,
• �कसी�रा�य�के��े�फल�को�बढ़ाना,
• �कसी�रा�य�के��े�फल�को�घटाना,
• �कसी�रा�य�क��सीमा��म��प�रवत�न�तथा�
• �कसी�रा�य�के�नाम�म��प�रवत�न�

हालां�क�अनु�छे द�3 इस�संबंध�म��दो�शत��रखता�है: एक, ऐसे��कसी�प�रवत�न�का���ताव�करने�वाला��वधेयक�रा�प�त�क��पूव��अनुशंसा�के


प�ात�ही�संसद�म��पुर:�था�पत��कया�जा�सकता�है।; और��सरी, उस��वधेयक�को�अनुशं�सत�करने�से�पूव��रा�प�त�को�उसे�संबं�धत�रा�य
�वधानमंडल�को�एक��न��त�समयाव�ध�म��उसके��वचार���तुत�करने�हेतु�संद�भ�त�करना�होता�है।�

इसके�अ�त�र�, संसद�क��नए�रा�य��के��नमा�ण�क��श���म���कसी�रा�य�अथवा�संघ�शा�सत��े��के��कसी�भाग�को��कसी�अ�य�रा�य�अथवा
संघ�शा�सत��े��के�साथ��मलाकर�नया�रा�य�अथवा�संघ�शा�सत��े��बनाने�क��श���भी�शा�मल�है।�

रा�प�त�( अथवा�संसद�) रा�य��वधानमंडल��ारा��कट�क��गई�राय�मानने�हेतु�बा�य�नह��है�और�वह�उ�ह���वीकार�अथवा�खा�रज�कर


सकता�है, भले�ही�यह�राय�समय�पर��ा�त��ई�हो।�

QUESTION 75.
�न�न�ल�खत�म��से�कौन-से�अ�धकार�सं�वधान��ारा�केवल�भारतीय�नाग�रक��को��दान��कए�गए�ह�?

1. धम�, मूलवंश, जा�त, �ल�ग�अथवा�ज�म�थान�के�आधार�पर��वभेद�के��व���अ�धकार�(अनु�छे द�15)।�

IASbaba
Web: http://ilp.iasbaba.com/ Score:
Email: ilp@iasbaba.com 0.00 / 200
Page 124
2019 - Test 4-
Exam Title :
History & Cu...
Email : ilbsnaa4@gmail.com
Contact :

2. लोक��नयोजन�म��अवसर�क��समता�का�अ�धकार�(अनु�छे द�16)।�

3. वाक्�व�अ�भ����क���वतं�ता, स�मेलन, संघ�बनाने, संचरण, �नवास�व��ापार�का�अ�धकार�(अनु�छे द�19)।�

4. सां�कृ�तक�एवं�शै��क�अ�धकार�(अनु�छे द�29 एवं�30)।�

�न�न�ल�खत�म��से�सही�कूट�का�चयन�क��जए:

a) 1,2 और�3
b) 2,3 और�4
c) 1,3 और�4
d) उपयु���सभी�
Correct Answer: D
Your Answer:
Explanation

Solution (d)

भारत�का�सं�वधान��न�न�ल�खत�अ�धकार�और��वशेषा�धकार�भारतीय�नाग�रक��को��दान�करता�है�(और��वदे �शय��को�दे ने�से�मना�करता�है):

• धम�, मूलवंश, जा�त, �ल�ग�अथवा�ज�म�थान�के�आधार�पर��वभेद�के��व���अ�धकार�(अनु�छे द�15)।�


• लोक��नयोजन�म��अवसर�क��समता�का�अ�धकार�(अनु�छे द�16)।�
• वाक्�व�अ�भ����क���वतं�ता, स�मेलन, संघ�बनाने, संचरण, �नवास�व��ापार�का�अ�धकार�(अनु�छे द�19)।�
• सां�कृ�तक�एवं�शै��क�अ�धकार�(अनु�छे द�29 एवं�30)।�
• लोक�सभा�और�रा�य��वधानसभा��के�चुनाव�म��वोट�डालने�का�अ�धकार।�
• संसद�और�रा�य��वधानमंडल��के�चुनाव�लड़ने�का�अ�धकार।�
• कुछ�साव�ज�नक�पद��जैसे�भारत�का�रा�प�त, भारत�का�उप-रा�प�त, सव��च��यायालय�तथा�उ�च��यायालय��के�जज, रा�य��के
रा�यपाल, भारत�का�महा�यायवाद��तथा�रा�य��के�महा�धव�ा, को�धारण�करने�क��पा�ता।�

सो�चए!

कौन�से�मौ�लक�अ�धकार��वदे �शय��को��दए�गए�ह�?

QUESTION 76.
‘ई-वे��बल’ �णाली�के�संदभ��म���न�न�ल�खत�कथन��पर��वचार�क��जए।�

1. यह�व�तु�एवं�सेवा�कर�(GST) �व�था�के�अंतग�त�व�तु��के�इं�ा-�टे ट(intra-state) के�साथ-साथ�इं�-�टे ट(inter-state)


आवागमन�को��ै क(track) करने�संबंधी��ेमवक���दान�करता�है।�

2. एक�ही�ई-वे��बल�व�तु��के�दे शभर�म��आवागमन�हेतु�वैध�होगा।�और�यह���येक�रा�य�हेतु�अलग-अलग�पार-प�(transit pass) क�


आव�यकता�को�समा�त�कर�दे गा।�

सही�कथन�का�चयन�क��जए�

a) केवल�1
b) केवल�2
c) 1 और�2 दोन��
d) न�तो�1 न�ही�2
Correct Answer: C
Your Answer:
Explanation

Solution (c)

IASbaba
Web: http://ilp.iasbaba.com/ Score:
Email: ilp@iasbaba.com 0.00 / 200
Page 125
2019 - Test 4-
Exam Title :
History & Cu...
Email : ilbsnaa4@gmail.com
Contact :

ई-वे��बल�

• इले��ो�नक�वे��बल�अथवा�‘ई-वे��बल’ �णाली�नई�व�तु�एवं�सेवा�कर�(GST) �व�था�के�अंतग�त�50,000 �पए�मू�य�से�अ�धक�क�


व�तु��के�10 �कमी. से�अ�धक��री�म���व�य�के�इं�ा��टे ट�के�साथ-साथ�इं���टे ट�आवागमन�का�पता�लगाने�संबंधी��ेमवक���दान
करती�है।�
• ई-वे��बल�का�व�तु��के�लदान�से�पूव��बनाया�जाना�ज�री�है�और�इसम��व�तु, उनके�भेजने�वाले, �ा�त�करने�वाले�व�प�रवहनकता��का
�ववरण�होना�चा�हए।�
• ई-वे��बल�के�अंतग�त���येक�रा�य�हेतु�अलग-अलग�पार-प��क��आव�यकता�नह��होती�तथा�एक�ई-वे��बल�व�तु��को�दे शभर�म��लाने-
ले�जाने�हेतु�वैध�होगा।�
• व�तु��के��ेषक�अथवा��ेता��ारा�बनाया�गया���येक�ई-वे��बल�आपू�त�कता��के�आउटवड��से�स��रटन��(GSTR1) म���वत: अपडेट
होगा।�
• व�तु��को�अपने��वयं�के�अथवा��कराये�के, वायुमाग�, रेल�अथवा�रोड�प�रवहन�म��ले�जाने�पर�ई-वे��बल�बनाना�आव�यक�है।�
• जहाँ�व�तु��को�सड़क�माग��से��ेषण�हेतु��कसी��ासंपोट� र�को��दया�जाता�है�और��ेषक�व��ा�तकता���कसी�ने�भी�ई-वे��बल�नह��बनाया
है, ऐसे�म��इसे�बनाना��ासंपोट� र�क���ज�मेदारी�होगी।�
• जब�भेजने�वाला�अथवा��ासंपोट� र�ई-वे��बल�बनाता�है, सामान�के��ा�तकता��को�इसे�पोट� ल�पर�या�तो��वीकार�करना�या�खा�रज�करना
होगा।�य�द��ा�तकता���ारा�72 घंट��तक�कोई�काय�वाही�नह��क��जाती, इसे��वीकृत�कर��लया�माना�जाएगा।�

Source: http://pib.nic.in/PressReleseDetail.aspx?PRID=1541576

QUESTION 77.
भारत�म��‘वैक��पक��नवेश�कोष�(Alternative Investment Funds)’ के�संदभ��म���न�न�ल�खत�कथन��पर��वचार�क��जए।�

1. यह��नजी�तरीके�से�एक��ऐसे��नवेश�कोष�को�संद�भ�त�करता�है��जनका���ोत�केवल�भारतीय�है।�

2. व�चर�कै�पटल�फ�ड�वैक��पक��नवेश�कोष�का�एक��कार�है।�

3. ये�आयकर�अ�ध�नयम�के�अंतग�त�करयो�य�नह��ह�।�

सही�कथन��का�चुनाव�क��जए�

a) 1 और�3
b) 1 और�2
c) केवल�2
d) 1, 2 और�3
Correct Answer: C
Your Answer:
Explanation

Solution (c)

वैक��पक��नवेश�कोष�

• भारत�म�, वैक��पक��नवेश�कोष��(AIFs) को�भारतीय���तभू�त�एवं��व�नमय�बोड��(वैक��पक��नवेश�कोष) �व�नयम, 2012 के


�व�नयम�2(1)(b) �ारा�प�रभा�षत��कया�जाता�है।�
• यह��नजी�तरीके�से�एक��ऐसे��नवेश�कोष�(चाहे�उनका���ोत�भारतीय�हो�अथवा��वदे शी) को�संद�भ�त�करता�है�जो���ट�अथवा�कंपनी
अथवा��नग�मत��नकाय�अथवा�एक�सी�मत�दा�य�व�वाली�साझेदारी�(LLP) के��प�म��है�और�जो�वत�मान�म��सेबी�के��कसी�कोष��बंधन
�व�नयम��ारा�आ�छा�दत�नह��है�और�न�ही�भारत�के��कसी�अ�य��े�ीय��व�नयामक�(IRDA, PFRDA, RBI आ�द) के���य�
�व�नयमन�के�अंतग�त�आते�ह�।�
• अत: भारत�म��AIFs वे��नजी�कोष�ह��जो�अ�यथा�भारत�क���कसी�अ�य��व�नयामक�य�सं�था�के�अंतग�त�नह��आते।�

IASbaba
Web: http://ilp.iasbaba.com/ Score:
Email: ilp@iasbaba.com 0.00 / 200
Page 126
2019 - Test 4-
Exam Title :
History & Cu...
Email : ilbsnaa4@gmail.com
Contact :

AIFs के��कार�

• �ेणी�I AIF वे�AIFs है��जनका�अथ��व�था�पर�सकारा�मक��भाव�होता�है, और��जनके��लए�सेबी�अथवा�भारत�सरकार��ारा�कुछ


�ो�साहन��अथवा�छू ट��पर��वचार��कया�जा�सकता�है।�
• �ेणी�II AIF वे�AIFs ह���जनके��लए�कोई��वशेष��ो�साहन�अथवा�छू ट�नह��द��जाती।�
• �ेणी�III AIF वे�कोष�ह���जनक��चय�नत�प�र��थ�तय��म��कुछ�नकारा�मक�बा�ताएं�होती�ह��और�जो�एक�बड़ी�सीमा�तक��भाव�का
इ�तेमाल�करते�ह�।�

ये�करयो�य�होते�ह�।�

सो�चए�!

• �ीन��ोथ�इ��वट��फ�ड�(GGEF)
• नेशनल�इ�वे�टम�ट�फ�ड�
• नेशनल�इ�वे�टम�ट�एंड�इ��ा���चर�फ�ड�

Source: http://pib.nic.in/PressReleseDetail.aspx?PRID=1541570

QUESTION 78.
‘�व�ा�ल�मी�पोट� ल’ आव�यकता�क��पू�त��करता�है-

a) मै�सव�ओपन�ऑनलाइन�कोस��(MOOCs) क��
b) �श�ा�ऋण��क��
c) म�हला��श�ा�क��
d) अ�पसं�यक�म�हला��नातक��हेतु�पीएचडी�क��
Correct Answer: B
Your Answer:
Explanation

Solution (b)

�व�ा�ल�मी�पोट� ल�

• यह�सरकार��ारा�यह�सु�न��त�करने�के��लए�शु���कया�गया�था��क�छा��ब�क��क���श�ा�ऋण��हेतु�बनाई�गई�एकल��खड़क���णाली�के
मा�यम�से�आसानी�से�ऋण��ा�त�कर�सक�।�
• इसे��व�ीय�सेवाएं��वभाग�(�व��मं�ालय), उ�च��श�ा��वभाग�(मानव�संसाधन��वकास�मं�ालय) और�इं�डयन�ब��स�एसो�सएशन
(IBA) के��नद� शन�म���वक�सत��कया�गया�है।�

Source: http://www.pib.nic.in/Pressreleaseshare.aspx?PRID=1541568

QUESTION 79.
भारतीय�सं�वधान�के�‘अनु�छे द�335’ के�संदभ��म���न�न�ल�खत�म��से�कौन-सा�सही�है?

a) यह�रा�प�त�को�भी�लोक�सभा�क��अव�ध�एक�बार�म��एक�वष��तक�बढाने�क��अनुम�त��दान�करता�है।�
b) यह�क���सरकार�को�क��-रा�य��व�ीय��व�था��के�भीतर�कोष�आवंटन��क��श����दान�करता�है।�
c) यह�अनुसू�चत�जा�तय��एवं�अनुसू�चत�जनजा�तय��के�सेवा��और�पद��पर�दाव��तथा�रा�य��ारा�‘पद��और�सेवा��पर��नयु���करते
समय’ अनुसू�चत�जा�तय��एवं�अनुसू�चत�जनजा�तय��के�दाव��को�मा�यता�दे ने�क��आव�यकता�से�संबं�धत�है।�
d) यह�रा�प�त�को�दे श�म��आपात��थ�त�क��घोषणा�करने�क��श����दान�करता�है।�
Correct Answer: C
Your Answer:
Explanation

IASbaba
Web: http://ilp.iasbaba.com/ Score:
Email: ilp@iasbaba.com 0.00 / 200
Page 127
2019 - Test 4-
Exam Title :
History & Cu...
Email : ilbsnaa4@gmail.com
Contact :

Solution (c)

सं�वधान�के�अनु�छे द�335 का�मु�य�जोर�रा�य��ारा�‘पद��और�सेवा��पर��नयु���करते�समय’ अनुसू�चत�जा�तय��एवं�अनुसू�चत�जनजा�तय�


के�दाव��को�मा�यता�दे ने�क��आव�यकता�पर�है।�हालां�क�अनु�छे द�335 यह�भी�कहता�है��क�ऐसे�दाव��को�मा�यता�द�ता�संबंधी��च�ता��के
अनु�प�होनी�चा�हए।�

सो�चए�!

• एम. नागराज�बनाम�भारत�संघ�वाद, 2006

Source: https://www.financialexpress.com/opinion/what-backwardness-not-all-sc-st-are-
historically-backward/1273036/

QUESTION 80.
‘प��लक�फाइन��सयल�मैनेजम�ट��स�टम�(PFMS)’ के�संदभ��म���न�न�ल�खत�कथन��पर��वचार�क��जए।�

1. यह�एक�वेब-आधा�रत�ऑनलाइन�सॉ�टवेर�ए�लीकेशन�है�जो�महालेखा��नयं�क�(CGA) के�काया�लय��ारा��वक�सत�और���या��वत�क�
गई�है।�

2. PFMS कवरेज�के�दायरे�म��क���य��े�क�और�क����ायो�जत�योजाना��के�साथ-साथ��व��आयोग�के�अनुदान��स�हत�अ�य��य�भी
शा�मल�ह�।�

3. PFMS का��योग�भारत�सरकार�क��सभी�क���य��े�क�योजना��हेतु�आव�यक�है।�

सही�कथन��का�चुनाव�क��जए�

a) 1 और�2
b) 2 और�3
c) 1 और�3
d) उपयु���सभी�
Correct Answer: D
Your Answer:
Explanation

Solution (d)

प��लक�फाइन��सयल�मैनेजम�ट��स�टम�(PFMS) �जसे�पूव��म��स��ल��लान��क��स�मो�नट�र�ग��स�टम�(CPSMS) के�तौर�पर�जाना�जाता�था,


एक�वेब-आधा�रत�ऑनलाइन�सॉ�टवेर�ए�लीकेशन�है�जो�लेखा�महा�नयं�क�(CGA) के�काया�लय��ारा��वक�सत�और���या��वत�क��गई�है।�

PFMS का�मु�य�उ�े �य�एक�कुशल�कोष��वाह��णाली�के�साथ-साथ�एक�भुगतान�सह�एकाउं�ट�ग�नेटवक��क���थापना�के��ारा�भारत�सरकार


हेतु�उ�चत�प��लक�फाइन��सयल�मैनेजम�ट��स�टम�को�सु�वधाजनक�बनाना�है।�PFMS भारत�सरकार�क���ड�जटल�इं�डया�पहल�के�तहत
�व�भ���हतधारक��को��रयल�टाइम, �व�सनीय�और�अथ�पूण���बंधन�सूचना��णाली�और�एक��भावी��नण�य�समथ�न��णाली��दान�करता�है।�

�व��मं�ालय�ने�भारत�सरकार�क��सभी�क���य��े�क�योजना��हेतु�आव�यक��प�से�प��लक�फाइन��सयल�मैनेजम�ट��स�टम�(PFMS) का
�योग�यह�कहते��ए�आरंभ��कया�है��क�PFMS �व�भ��सरकारी�योजना��का�लाभ�अं�तम�����तक�प�ंचना�सु�न��त�करेगी।�

Read More - http://pib.nic.in/newsite/PrintRelease.aspx?relid=171984

PFMS योजना, �व��मं�ालय�के��य��वभाग�के�कहने�पर�दे श�के�एक�मह�वपूण��साव�ज�नक��व���बंधन�(PFM) के��प�म��लेखा


महा�नयं�क�(CGA) �ारा�आरंभ�क��गई�है।�इस�योजना�का�उ�े �य�पारद�श�ता�को��ो�सा�हत�करना�और�क���सरकार�के�सम���व�ीय��बंधन
एवं�क���सरकार�क���व�भ��योजना��के�दे श�भर�म��काया��वयन�म��एक�मूत��सुधार�लाना�है।�PFMS कवरेज�के�दायरे�म��क���य��े�क�और�क��
�ायो�जत�योजाना��के�साथ-साथ��व��आयोग�के�अनुदान��स�हत�अ�य��य�भी�शा�मल�होते�ह�।�

Source: http://pib.nic.in/PressReleseDetail.aspx?PRID=1541657

IASbaba
Web: http://ilp.iasbaba.com/ Score:
Email: ilp@iasbaba.com 0.00 / 200
Page 128
2019 - Test 4-
Exam Title :
History & Cu...
Email : ilbsnaa4@gmail.com
Contact :
QUESTION 81.
‘जल�माग���वकास�प�रयोजना�(JMVP)’ के�संदभ��म���न�न�ल�खत�कथन��पर��वचार�क��जए।�

1. यह�पूव��र�को�शेष�भारत�से�जोड़ने�के�उ�े �य�से���पु�-बराक�नद��तं��पर���या��वत�क��जा�रही�है।�

2. �व��ब�क�इस�प�रयोजना�के�काया��वयन�हेतु�तकनीक��और��नवेश�सहायता��दान�कर�रहा�है।�

सही�कथन�का�चयन�क��जए�

a) केवल�1
b) केवल�2
c) 1 और�2 दोन��
d) न�तो�1 न�ही�2
Correct Answer: B
Your Answer:
Explanation

Solution (b)

जल�माग���वकास�प�रयोजना�(JMVP)

• रा�ीय�जलमाग�-1 पर�जल�माग���वकास�प�रयोजना�(JMVP) को��व��ब�क�क��तकनीक��और��नवेश�सहायता�से���या��वत��कया�जा


रहा�है।�
• इस�प�रयोजना�के�अंतग�त�वाराणसी�से�ह��दया�(चरण-I) के�बीच�तीन�मीटर�गहराई�वाले�फेयरवे�को��वक�सत��कया�जाना�है�जो
1380 �कमी. क���री�कवर�करेगा।�
• यह�प�रयोजना�उ�र��दे श, �बहार, झारखंड�और�प. बंगाल�म��पड़ती�है।�

लाभ�

• यह�प�रवहन�का�एक�पया�वरण-�हतैषी�और�लागत-�भावी�वैक��पक�माग��उपल�ध�करवाएगा।�
• यह�दे श�म��संभा�रक��लागत��को�कम�करने�म��योगदान�दे गा।�
• यह�म�ट�-मोडल�और�इंटर-मोडल�ट�म�न�स, रोल�ऑन-रोल�ऑफ़�(Ro-Ro) सु�वधाएं, फेरी�सेवाएं�और�नौवहन�मदद�जैसे�अवसंरचना
�वकास�को�बढ़ावा�दे गा।�

Source: http://pib.nic.in/PressReleseDetail.aspx?PRID=1541662

QUESTION 82.
‘राइज��क(RISECREEK)’ हाल�ही�म��सु�ख़�य��म��था।�यह��या�है?

a) नव��वक�सत�माइ�ो�ोसेसर�
b) �वाल��वरंजन�से��नपटने�हेतु�एक�नई��ौ�ो�गक��
c) �ोए�शया�के�हवार�म��नव�न�म�त���प।�
d) चीन��ारा��वक�सत�एक�सुपरसो�नक�हवाई�जहाज�
Correct Answer: A
Your Answer:
Explanation

Solution (a)

समाचार�: कं�यूटर�वै�ा�नक��और�IIT-म�ास�क��एक�छा��ट�म�ने�एक�उ�ोग-मानक�माइ�ो�ोसेसर�(RISECREEK) �वक�सत��कया�है।�

IASbaba
Web: http://ilp.iasbaba.com/ Score:
Email: ilp@iasbaba.com 0.00 / 200
Page 129
2019 - Test 4-
Exam Title :
History & Cu...
Email : ilbsnaa4@gmail.com
Contact :

संबं�धत�त�य�

• इसे�अ�य��के��ारा�भी�अपनाया�जा�सकता�है��य��क�इसका��डज़ाइन�ओपन�सोस��है।�
• यह�NAVIC (इं�डयन�रीजनल�ने�वगेशन�सेटेलाइट) और�इ�टरनेट�ऑफ़��थ��स�(IoT) इले��ॉ�न�स�जैसे�र�ा�और�साम�रक
उपकरण��क��माँग�को�पूरा�कर�सकता�है।�
• 300 �च�स�के�आरं�भक�बैच�को�RISECREEK नाम��दया�गया�है�और�इसे��ोजे�ट�श���के�अंतग�त�उ�पा�दत��कया�गया�है।�
• �ोजे�ट�श���2014 म��IIT-M क��एक�पहल�के��प�म��शु���आ।�

Source: https://www.thehindu.com/sci-tech/technology/iit-madras-powers-up-a-desi-
chip/article24609946.ece

QUESTION 83.
‘भारतीय��टार�कछु ए(Indian star tortoises)’ के�संदभ��म���न�न�ल�खत�कथन��पर��वचार�क��जए।�

1. यह�IUCN क��संकट��त��जा�तय��क��लाल�सूची�म��‘गंभीर��प�से�संकट��त’ के��प�म��सूचीब��है।�

2. यह�केवल�प��मी�घाट�म���था�नक�है।�

3. सीमा�शु�क�अ�ध�नयम, 1962 के�अंतग�त�इसक��ज�ती�क��जा�सकती�है।�

सही�कथन��का�चुनाव�क��जए�

a) 1 और�3
b) 2 और�3
c) केवल�3
d) 2 और�3
Correct Answer: C
Your Answer:
Explanation

Solution (c)

भारतीय��टार�कछु ए�

• यह�IUCN क��संकट��त��जा�तय��क��लाल�सूची�म��‘सुभे�’ के��प�म��सूचीब��है।�


• यह�व�य�जीव�(संर�ण) अ�ध�नयम, 1972 क��सूची�IV म��सूचीब��है�और��वदे शी��ापार�नी�त�के�अंतग�त�इसका��नया�त���तबं�धत
है।�
• सीमा�शु�क�अ�ध�नयम, 1962 के�अंतग�त�इसक��ज�ती�क��जा�सकती�है।�
• इसे�CITES के�अप��ड�स�II म��शा�मल��कया�गया�है।�
• �था�नक: भारत�(आं��दे श, कना�टक, ओ�डशा, त�मलनाडु ); पा�क�तान; �ीलंका�
• यह��ाकृ�तक��प�से���ब�जंगल�, घासभू�मय��और�शु�क�एवं�अ�� शु�क��े���क��कुछ���बभू�मय��म���नवास�करता�है, �क�तु�यह
मानव�के�वच��व�वाले�भू��य��म��भी�सामा�यत: �नवास�करता�है।�

Source: https://www.thehindu.com/news/cities/Visakhapatnam/1125-indian-star-
tortoises-seized-from-three-train-passengers/article24610332.ece

QUESTION 84.
हाल�ही�म��सु�ख़�य��म��रहा�‘�टाररी��काई-2(Starry Sky-2)’ �या�है?

a) नासा��ारा�परी�ण��कया�गया�इले���क��लेन�
b) चीन��ारा��वक�सत�हाइपरसो�नक��लाइट��हीकल�
c) जापान��ारा��डज़ाइन��कया�गया�सौर-उजा��से�चा�लत�दो�सीट��वाला��वमान�

IASbaba
Web: http://ilp.iasbaba.com/ Score:
Email: ilp@iasbaba.com 0.00 / 200
Page 130
2019 - Test 4-
Exam Title :
History & Cu...
Email : ilbsnaa4@gmail.com
Contact :

d) चीन��ारा���े�पत�अंत�र��वेधशाला�
Correct Answer: B
Your Answer:
Explanation

Solution (b)

Xingkong-2/Starry Sky-2

• यह�चीन�का�एक�हाइपरसो�नक��वमान�है�जो�परमाणु�ह�थयार�ले�जाने�म��स�म�है�और�वत�मान�पीढ़��क���कसी�भी�एंट�-�डफ�स
�णा�लय��को�भेद�सकता�है।�

Source: https://www.thehindu.com/todays-paper/tp-life/china-tests-hypersonic-aircraft/
article24618690.ece?utm_source=tp-life&utm_medium=sticky_footer

QUESTION 85.
‘8888 अपराइ�ज�ग’ संबं�धत�है-

a) डेमो�े�टक��रप��लक�ऑफ़�कांगो�से�
b) �यांमार�से�
c) �ूनी�शया�से�
d) सूडान�से�
Correct Answer: B
Your Answer:
Explanation

Solution (b)

8888 अपराइ�ज�ग�

• 8 अग�त�को��यांमार�म��जन��व�ोह�क��30 व��वष�गाँठ�होगी।�
• ‘8888’ अपराइ�ज�ग�(अथवा�अग�त�1988 का�आठवां��दन) लोकतं��समथ�क�आंदोलन�के�स�दभ��म���यांमार�के�सवा��धक
ऐ�तहा�सक��दन��म��से�एक�है।�
• यह�दे श�ापी��वरोध�, माच��और�नाग�रक�अशां�त�क��एक��ृंखला�थी।�
• ‘8888’ एक�जन�आंदोलन�था��जसने�बमा��सोश�ल�ट��ो�ाम�पाट��क��राजनी�तक, आ�थ�क�और�सामा�जक�मामल��पर�पकड़�को
चुनौती�द���जसके�चलते�दे श�भयंकर�गरीबी�म��धंस�गया�था।�
• �वरोध�एक�छा��आंदोलन�के��प�म��आरंभ��ए�और�इनका�आयोजन�रंगून�आट्� स�एंड�साइंसेज�यू�नव�स�ट��तथा�रंगून�इं��ट�ूट�ऑफ़
टे �नोलॉजी�(RIT) के�छा����ारा��कया�गया�था।�

Source: https://www.thehindu.com/opinion/op-ed/thirty-years-after-the-8888-uprising/
article24617719.ece

QUESTION 86.
‘�ल�बन�सं�ध’ के�संदभ��म���न�न�ल�खत�कथन��पर��वचार�क��जए।�

1. इसने�यूरोपीय�आ�थ�क�समुदाय�(EEC) क��शु�आत�क�।�

2. इसने�साझी�मु�ा�यूरो�के��लए�माग���श�त��कया।�

3. इसने�यूरोपीय�संघ�के�कोट� �ऑफ़�ज��टस�क���थापना�क�।�

सही�कथन��का�चुनाव�क��जए�

a) 1 और�2

IASbaba
Web: http://ilp.iasbaba.com/ Score:
Email: ilp@iasbaba.com 0.00 / 200
Page 131
2019 - Test 4-
Exam Title :
History & Cu...
Email : ilbsnaa4@gmail.com
Contact :

b) 1 और�3
c) केवल�3
d) उपरो��म��से�कोई�नह��
Correct Answer: D
Your Answer:
Explanation

Solution (d)

�ल�बन�सं�ध�

• इसने�यूरोपीय�संघ�हेतु��व�नयम��को�अ�तन��कया��जससे�एक�अ�धक�क���कृत�नेतृ�व�व��वदे श�नी�त, संघ�छोड़ने�के�इ�छु क�दे श��हेतु


एक�समु�चत����या�तथा�नई�नी�तय��के��नमा�ण�हेतु�एक�सरल����या��था�पत��ई।�
• इस�पर�यूरोपीय�संघ�के�27 सद�य�दे श���ारा�ह�ता�र��कए�गए�और�यह�आ�धका�रक��प�से��दसंबर�2009 म��इसके�ह�ता��रत�होने
के�दो�वष��बाद��भावी��ई।�
• इसने�दो��व�मान�सं�धय��को�संशो�धत��कया, रोम�क��सं�ध�और�मा���च�सं�ध।�

रोम�क��सं�ध�(1957)

• इसने�यूरोपीय�आ�थ�क�समुदाय�(EEC) क��शु�आत�क�, सद�य�दे श��के�बीच�सीमा�शु�क�घटाए�और�व�तु��के��लए�एक�एकल


बाजार�को�सु�वधाजनक�बनाया�तथा�उनके�प�रवहन�क��नी�तयां�तय�क�।�
• इसे���ट��ऑन�द�फंग�श�न�ग�ऑफ़�द�यूरो�पयन�यू�नयन�(TFEU) के�तौर�पर�भी�जाना�जाता�है।�

मा���च�सं�ध�(1992)

• इसने�यूरोपीय�संघ�के�तीन��त�भ��क���थापना�क��और�साझी�मु�ा�यूरो�के��लए�माग���श�त��कया।�
• इसे���ट��ऑन�यूरो�पयन�यू�नयन�(TEU) के�तौर�पर�भी�जाना�जाता�है।�

कोट� �ऑफ़�ज��टस�ऑफ़�द�यूरो�पयन�यू�नयन�क���थापना�1952 म���ई।�

Source: https://www.thehindu.com/opinion/op-ed/threats-to-eu-values/
article24617328.ece

QUESTION 87.
�न�न�ल�खत�म��से�कौन-से�भारतीय�रा�य�क��सीमा��यांमार�के�‘र�खन��ांत’ के�साथ�लगती�है?

a) नगाल�ड�
b) म�णपुर�
c) �मजोरम�
d) उपरो��म��से�कोई�नह��
Correct Answer: D
Your Answer:
Explanation

Solution (d)

रा�खन�क��सीमा�भारत�के�साथ�नह��लगती।�

IASbaba
Web: http://ilp.iasbaba.com/ Score:
Email: ilp@iasbaba.com 0.00 / 200
Page 132
2019 - Test 4-
Exam Title :
History & Cu...
Email : ilbsnaa4@gmail.com
Contact :

Source: https://www.thehindu.com/opinion/op-ed/thirty-years-after-the-8888-uprising/
article24617719.ece

QUESTION 88.
�न�न�ल�खत�म��से�कौन�से�दे श�‘पै�स�फक��र�ग�ऑफ़�फायर’ �े��म��ह�?

1. कनाडा�

2. �यूज़ीलै�ड�

3. जापान�

4. �चली�

सही�कूट�का�चयन�क��जए�

a) 1, 2 और�3
b) 2, 3 और�4
c) 1, 3 और�4
d) उपयु���सभी�
Correct Answer: D
Your Answer:
Explanation

IASbaba
Web: http://ilp.iasbaba.com/ Score:
Email: ilp@iasbaba.com 0.00 / 200
Page 133
2019 - Test 4-
Exam Title :
History & Cu...
Email : ilbsnaa4@gmail.com
Contact :

Source: https://www.thehindu.com/todays-paper/tp-international/indonesia-
earthquake-toll-rises-to-98/article24618747.ece

QUESTION 89.
‘��ब�टाईफस(Scrub Typhus)’ के�संदभ��म���न�न�ल�खत�कथन��पर��वचार�क��जए।�

1. यह�केवल�उ�णक�टबंधीय��े���म��पाया�जाता�है�

2. यह�घुन(mites) के�मा�यम�से�संचा�रत�होता�है�

3. यह�गंभीर�इ�सेफेलाइ�टस��स��ोम�के�कारण��म��से�एक�है�

सही�कथन��का�चुनाव�क��जए�

a) 1 और�2
b) 2 और�3
c) 1 और�3
d) उपयु���सभी�
Correct Answer: B
Your Answer:
Explanation

Solution (b)

IASbaba
Web: http://ilp.iasbaba.com/ Score:
Email: ilp@iasbaba.com 0.00 / 200
Page 134
2019 - Test 4-
Exam Title :
History & Cu...
Email : ilbsnaa4@gmail.com
Contact :

��ब�टाईफस�

• यह�टाईफस�क��एक��कार�है�जो�अंतराको�शक�य�परजीवी�ओ�रएं�टया�सुटसुगमूशी�के�कारण�होता�है।�
• इसे��बहार�और�अब�उ�र��दे श�म��गंभीर�इ�सेफेलाइ�टस��स��ोम�के�सवा��धक�सामा�य�कारण�के�तौर�पर�दे खा�गया�है।�
• ��ब�टाईफस, ���बकु�लद�घुन�क��कुछ��जा�तय���ारा�संचा�रत�होता�है�जो�घनी�झा�ड़य��वाले�इलाक��म��पाए�जाते�ह�।�
• इसके��लए�कोई�भी�लाइस�सशुदा�ट�का�उपल�ध�नह��है।�

सुटसुगमूशी��ायंगल�

• ��ब�टाईफस, सुटसुगमूशी��ायंगल�(ओ. सुटसुगमूशी�के�नाम�से��लया�गया) नाम�से�जाने�जाने�वाले��व��के�इलाक��म���ा�त�है।�


• यह�उ�र�म��उ�री�जापान�और�सु�र�पूव���स�से�लेकर�द��ण�म��उ�री�ऑ��े �लया�के�पास���थत�सोलोमन�सागर�के�आसपास�के��े��
और�प��म�म��पा�क�तान�और�अफगा�न�तान�तक��व�ता�रत�है।�
• यह�द��ण�अमे�रका�के�भाग��म��भी��था�नक�हो�सकता�है।�

Source: https://www.thehindu.com/news/national/scrub-typhus-is-key-encephalitis-
cause-in-eastern-up-study/article24627299.ece

QUESTION 90.
उमंग�ए�प�संबं�धत�है�

a) अ�भशासन�म����य��नाग�रक�भागीदारी�हेतु�नाग�रक�भागीदारी�मंच�
b) क��, रा�य, �थानीय��नकाय�और�सरकार�क��एज��सय��क��ई-सेवा��तक�अ�खल�भारतीय�प�ँच�
c) आयकर��रट�स��क��ऑनलाइन�फाय�ल�ग�
d) तीथ�या�ा�व�पय�टन�
Correct Answer: B
Your Answer:
Explanation

Solution (b)

उमंग�( यू�नफाइड�मोबाइल�ए�लीकेशन�फॉर��यू-ऐज�गवन�स�)

• इसे�इले��ॉ�नक��और�सूचना��ौ�ो�गक��मं�ालय�क��नेशनल�ई-गवन�स��डवीज़न�(NeGD) �ारा��वक�सत��कया�गया�है।�
• उमंग, क���य�से�लेकर��थानीय�सरकार��नकाय��क��ई-सेवा��के�साथ-साथ�अ�य�नाग�रक�क���त�सेवा��तक�सभी�भारतीय�नाग�रक�
क��प�ँच�का�एक�एकल�मंच��दान�करती�है।�
• यह�एक�एक�कृत�उपागम��दान�करती�है, जहाँ�नाग�रक��व�वध�सरकारी�सेवा��को��ा�त�करने�हेतु�एक�ए�लीकेशन�इन�टॉल�कर
सकते�ह�।�

Source: http://pib.nic.in/PressReleseDetail.aspx?PRID=1541941

QUESTION 91.
‘�धानमं�ी�सुर�ा�बीमा�योजना�(PMSBY)’ के�संदभ��म���न�न�ल�खत�कथन��पर��वचार�क��जए।�

1. यह�18 से�50 वष��आयु�के�उन�लोग��के��लए�उपल�ध�है��जनके�ब�क��म��खाते�ह�।�

2. यह��वशु���प�से�एक�टम��बीमा�पॉ�लसी�है�जो�केवल�मृ�यु�को�कवर�करती�है।�

सही�कथन�का�चयन�क��जए�

a) केवल�1
b) केवल�2
c) 1 और�2 दोन��

IASbaba
Web: http://ilp.iasbaba.com/ Score:
Email: ilp@iasbaba.com 0.00 / 200
Page 135
2019 - Test 4-
Exam Title :
History & Cu...
Email : ilbsnaa4@gmail.com
Contact :

d) न�तो�1 न�ही�2
Correct Answer: D
Your Answer:
Explanation

Solution (d)

�धानमं�ी�सुर�ा�बीमा�योजना�(PMSBY)

• यह�दे श�म��बीमा�कवरेज�का��तर�बढाने�और�आम�आदमी�(�वशेषकर�गरीब) व�समाज�के��वप��वग��को�बीमा�कवर�दे ने�के����कोण�से


शु��क��गई�थी।�
• यह�18 से�70 वष��आयु�के�उन�लोग��के��लए�उपल�ध�है��जनके�ब�क�खाते�ह�।�
• इसका�वा�ष�क��ी�मयम�कर�के�अ�त�र��₹12 है।�
• �धानमं�ी�सुर�ा�बीमा�योजना�को�GST से�छू ट��ा�त�है।�
• �ी�मयम�क��रा�श�खाते�से��वयं�काट�ली�जाएगी।�
• �घ�टना�बीमा�योजना�का�कवर�1 जून�से�लेकर�31 मई�तक�एक�वष��का�होगा�और�ब�क��के�मा�यम�से�इसका���ताव��कया�जाएगा�एवं
साव�जा�नक��े��क��सामा�य�बीमा�कंप�नय��के�मा�यम�से�इसका��शासन��कया�जाएगा।�

जो�खम�कवरेज�:

• मृ�यु�होने�पर�- 2 लाख��पए�
• दोन��आँख��क��पूण��एवं�अ��तल�य���त�अथवा�दोन��हाथ��या�पैर��के��योग�क����त�अथवा�एक�आँख�क��रोशनी�चले�जाना�तथा�हाथ
या�पैर�क����त�- 2 लाख��पए�
• एक�आँख�क��पूण��एवं�अ��तल�य���त�अथवा�एक�हाथ�अथवा�पैर�के��योग�क����त–1 लाख��पए�

Source: http://pib.nic.in/PressReleseDetail.aspx?PRID=1541986

QUESTION 92.
�वदे शी�(��तबं�धत��े�) आदे श, 1963 के�अंतग�त��न�न�ल�खत�म��से�कौन-से�रा�य��क��या�ा�हेतु���तबं�धत��े��पर�मट�(RAP) क�
आव�यकता�होती�है?

1. ज�मू�व�क�मीर�

2. नगाल�ड�

3. म�णपुर�

सही�कथन�का�चयन�क��जए�

a) केवल�1
b) 2 और�3
c) उपयु���सभी�
d) उपरो��म��से�कोई�नह��
Correct Answer: D
Your Answer:
Explanation

Solution (d)

��तबं�धत��े��पर�मट�(RAP)

फॉरन�स��(��तबं�धत��े�) आदे श, 1963 के�अंतग�त��न�न�ल�खत�को�‘��तबं�धत’ �े��घो�षत��कया�गया�है�-

• अंडमान�व��नकोबार���प�समूह�– सम�त�संघ�शा�सत��े��

IASbaba
Web: http://ilp.iasbaba.com/ Score:
Email: ilp@iasbaba.com 0.00 / 200
Page 136
2019 - Test 4-
Exam Title :
History & Cu...
Email : ilbsnaa4@gmail.com
Contact :
• �स��कम�– रा�य�के�कुछ�भाग�

�क�तु�अब�,

• क���य�गृह�मं�ालय�ने�अंडमान�व��नकोबार�से���तबंध�समा�त�करने�का��नण�य��लया��य��क�सरकार�वहाँ�पय�टन�को�बढ़ावा�दे ने�क�
इ�छु क�है।�
• �वदे �शय��को�अब�अंडमान�व��नकोबार�के�29 आवा�सत��े���क��या�ा�हेतु���तबं�धत��े��पर�मट�क��आव�यकता�नह��है।�

सो�चए�!

• संर��त��े��पर�मट�(PAP)
• मु��आवागमन��व�था�(FMR)

Source: https://www.thehindu.com/news/national/access-to-andamans-eased/
article24627284.ece

QUESTION 93.
‘�पोडो�टे रा��ूगेपरडा(Spodoptera frugiperda)’ हाल�ही�म��सु�ख़�य��म��था।�यह��या�है?

a) एक�आ�ामक�कांटेदार�झाड़ी�
b) एक�आ�ामक�कृ�ष�क�ट�
c) स�दय���साधन�म���योग��कया�जाना�एक�पु�प�
d) उपरो��म��से�कोई�नह��
Correct Answer: B
Your Answer:
Explanation

Solution (b)

फॉल�आम�वोम��

समाचार�: फॉल�आम�वोम��(�पोडो�टे रा��ूगेपरडा) नामक�एक�आ�ामक�कृ�ष�क�ट�कना�टक�म��खोजा�गया।�

संबं�धत�त�य�

• उ�री�अमे�रका�का�एक��मुख�म�का�क�ट�फॉल�आम�वोम��2016 म��अ��का�म��प�ँचा।�
• कना�टक�म��इसका��मलना�ए�शया�म��इस�क�ट�के�पाए�जाने�क��पहली�घटना�है।�
• यह�लगभग�100 अलग-अलग�फसल��पर�पलता�है��जनम��स��जयां, चावल�और�ग�ा�शा�मल�है।�
• ये��व�जा�त�भ�क�होते�ह�।�

फॉल�आम�वोम��श�द�अनेक��जा�तय��को�संद�भ�त�कर�सकता�है�जो��ाय: �जा�त�के�लावा���तर�पर��ापक�आ�ामक��वहार�क���ा�या
करता�है।�

Source: https://www.thehindu.com/news/national/karnataka/alarm-as-deadly-maize-
pest-seen-in-karnataka/article24636285.ece

QUESTION 94.
��े टे�जक��े ड�ऑथ�रज़ेशन�(STA) के�संदभ��म���न�न�ल�खत�कथन��पर��वचार�क��जए।�

1. यह�प�रभा�षत�प�र��थ�तय��के�अंतग�त�एक��नयं��त�मद�के��कसी��व�श��लेनदे न�को�लाइस�स�के��बना��नया�त�क��अनुम�त��दान�करता�है।�

2. भारत�अकेला�दे श�है��जसे�अमे�रका��ारा���े टे�जक��े ड�ऑथ�रज़ेशन-1 (STA-1) का�दजा���दान��कया�गया�है।�

सही�कथन�का�चयन�क��जए�

IASbaba
Web: http://ilp.iasbaba.com/ Score:
Email: ilp@iasbaba.com 0.00 / 200
Page 137
2019 - Test 4-
Exam Title :
History & Cu...
Email : ilbsnaa4@gmail.com
Contact :

a) केवल�1
b) केवल�2
c) 1 और�2 दोन��
d) न�तो�1 न�ही�2
Correct Answer: A
Your Answer:
Explanation

Solution (a)

��े टे�जक��े ड�ऑथ�रज़ेशन�

• यह�प�रभा�षत�प�र��थ�तय��के�अंतग�त�एक��नयं��त�मद�के��कसी�लेनदे न-�व�श��लाइस�स�के��बना��नया�त�क��अनुम�त�दे ता�है।��कसी


लाइस�स�अपवाद�हेतु�प�रभा�षत�प�र��थ�तयां�अमे�रक��सरकार�को��नयं�ण��को�लागू�करने�क���मता�सु�न��त�के��लए�एक�ऑ�डट
�े ल�क��अनुम�त�दे ती�ह�।�
• STA, �कसी��व�श��लाइस�स�के��बना�कॉमस��कं�ोल��ल�ट�टू �अलाइज़�म��दज��व�तु��के�एक�प�रभा�षत�समु�चय�के�तथा�चु�न�दा�अ�य
�म��दे श��को��नया�त�क��अनुम�त�दे ता�है।�
• भारत�को���े टे�जक��े ड�ऑथ�रज़ेशन-1 (STA-1) दजा���दया�गया�है�जो�इसे�36 दे श��के�समूह�(अ�धकतर�नाटो�सहयोगी) म���थान
�दलाता�है।�जापान�और�द. को�रया�इस�समूह�म��अकेले�ए�शयाई�दे श�ह�।�
• यह�अ�धक�संवेदनशील�र�ा��ौ�ो�गक�य��तथा�दोहरे��योग�क���ौ�ो�गक�य��का�भारत�को��सार�के�भय�से�र�हत�होकर�ह�तांतरण
�कए�जाने�क��अनुम�त�दे ता�है।�
• STA-1 भारत�को�र�ा�और�अ�य�उ�च�तकनीक�उ�पाद��के�मामले�म���ापक�आपू�त���ृंखला�द�ता��दान�करता�है�जो�अमे�रक�
�णा�लय��के�साथ�इसक��ग�त�व�धय��को�बढ़ावा�दे ने, �णा�लय��क��अ�तरसं��यता�म��मददगार�है�तथा�यह�लाइस�स�के�अनुमोदन�म�
लगने�वाले�समय�और�संसाधन��को�कम�करेगा।�

Source: https://www.thehindu.com/news/international/gokhale-in-us-ahead-of-22-talks/
article24635755.ece

QUESTION 95.
‘�नया�त��म�’ के�बारे�म���न�न�ल�खत�कथन��पर��वचार�क��जए।�

1. यह�अंतरा��ीय��ापार�करने�हेतु�आव�यक��ापक�सूचना��दान�करता�है।�

2. फेडरेशन�ऑफ़�इं�डयन�ए�सपोट� �ऑग�नाइजेशन�(FIEO) �ारा�इसे��वक�सत��कया�गया�है।�

3. यह�भारत�के�सापे��अ�य�दे श��के�ITC HS कोड�के�मान�च�ण�हेतु�आंत�रक��प�से�काय��करता�है।�

सही�कथन��का�चुनाव�क��जए�

a) 1 और�2
b) 2 और�3
c) 1 और�3
d) उपयु���सभी�
Correct Answer: D
Your Answer:
Explanation

Solution (d)

�नया�त��म��

• यह�वा�ण�य�और�उ�ोग�मं�ालय��ारा�शु���कया�गया�है।�
• इसे�फेडरेशन�ऑफ़�इं�डयन�ए�सपोट� �ऑग�नाइजेशन�(FIEO) �ारा��वक�सत��कया�गया�है।�

IASbaba
Web: http://ilp.iasbaba.com/ Score:
Email: ilp@iasbaba.com 0.00 / 200
Page 138
2019 - Test 4-
Exam Title :
History & Cu...
Email : ilbsnaa4@gmail.com
Contact :
• यह�अंतरा��ीय��ापार�करने�के��लए�आव�यक��ापक�सूचना��दान�करता�है��जसम���नया�त�और�आयात�हेतु�बने�नी�तगत��ावधान,
मा�य�GST रेट, उपल�ध��नया�त��ो�साहन, �शु�क, बाजार�प�ँच�ज�रत�- SPS तथा�TBT उपाय�शा�मल�ह�।�
• यह�भारत�के�सापे��अ�य�दे श��के�ITC HS कोड�के�मान�च�ण�हेतु�आंत�रक��प�से�काय��करता�है�तथा�यह��यो�ा��को��कसी�भी
दे श�के�HS कोड�के�बारे�म��परेशानी��दए��बना�वां�छत�डेटा�उपल�ध�करवाता�है।�
• ITC (HS) कोड् स�को�इं�डयन��े ड��ले�र�फकेशन�(ITC) के�नाम�से�जाना�जाता�है�और�ये�को�ड�ग�क��हाम�नाइ�ड��णाली�(HS)
पर�आधा�रत�है।�भारत�म��इसे�आयात-�नया�त�काय�वा�हय��के��लए�अपनाया�गया�था।�भारतीय�सीमाशु�क��वभाग�एक�आठ�अंक�य
ITC (HS) कोड�का��योग�करता�है�जो�रा�ीय��ापार�आव�यकता��के��लए�उपयु��ठहरता�है।�

Source: http://pib.nic.in/PressReleseDetail.aspx?PRID=1542140

QUESTION 96.
�न�न�ल�खत�म��से�कौन-सा�एक�‘यूने�क���व���वरासत��म��त��थल’ है?

a) कंचनजंघा�रा�ीय�पाक��
b) अजंता�गुफाएं�
c) भीमबेटका�के�शैला�य�
d) मानस�व�यजीव�अ�यारण�
Correct Answer: A
Your Answer:
Explanation

Solution (a)

�स��कम�का�कंचनजंघा�रा�ीय�पाक��(KNP) भारत�से�एकमा��‘�म��त’ �वरासत��थल�है।�

एक�‘�म��त��वरासत��थल’ �ाकृ�तक�और�सां�कृ�तक�दोन���कार�क���वशेषताएं�दशा�ता�है।�

भारत�म��अब�27 सां�कृ�तक�संप�तय�, सात��ाकृ�तक��थल��और�एक��म��त��थल�के�साथ�35 �थल�ह���ज�ह���व���वरासत��थल�के��प�म�


अ�धसू�चत��कया�गया�है।�

समाचार�: कंचनजंघा�बायो��फयर��रज़व��भारत�से�11वां�बायो��फयर��रज़व��बन�गया�है��जसे�यूने�को��ारा�ना�मत�व�ड��नेटवक��ऑफ़
बायो��फयर��रज़व��(WNBR) म��शा�मल��कया�गया�है।�

Source: https://timesofindia.indiatimes.com/home/environment/the-good-earth/
kanchenjunga-biosphere-reserve-gets-entry-into-the-unescos-global-list/
articleshow/65327646.cms

QUESTION 97.
‘रेड�कान�र�नो�टस’ �न�न�ल�खत�म��से�कौन-से�कथन�से�संबं�धत�है?

a) उन����य��के�बारे�म��चेतावनी�और�ख़ु�फ़या�सूचना��दान�करना��ज�ह�ने�आपरा�धक�कृ�य��को�अंजाम��दया�है�तथा�उनके��ारा�इन
अपराध��के�अ�य�दे श��म��दोहराए�जाने�क��आशंका�है।�
b) एक�अपराध�के�संबंध�म���कसी�����क��पहचान, �थान�अथवा�ग�त�व�ध�के�बारे�म��अ�त�र��सूचना�जुटाना।�
c) अपरा�धय���ारा�अपराध�करने�के�ढं ग, उन��ारा��यु��व�तु�, उपकरण��और��छपाने�क���व�धय��संबंधी�सूचना�माँगना�अथवा��दान
करना�।�
d) ��यप�ण�अथवा�ऐसी�ही��कसी��व�धक�काय�वाही�हेतु�वां�छत����य��क��अव��थ�त�और��गर�तारी�चाहना।�
Correct Answer: D
Your Answer:
Explanation

Solution (d)

IASbaba
Web: http://ilp.iasbaba.com/ Score:
Email: ilp@iasbaba.com 0.00 / 200
Page 139
2019 - Test 4-
Exam Title :
History & Cu...
Email : ilbsnaa4@gmail.com
Contact :

Source: https://www.thehindu.com/news/national/india-hands-over-extradition-request-
for-nirav-modi-to-uk-authorities/article24601882.ece

QUESTION 98.
�न�न�ल�खत�म��से�जॉइंट�का���हे��सव��लान�ऑफ़�ए�शन�को�संद�भ�त�करता�है-

a) �च�कोट��रपोट� �
b) भारत-ईरान�परमाणु�समझौता�
c) द�हाट� �ऑफ़�ए�शया-इ�तांबुल����या�
d) उपरो��म��से�कोई�नह��
Correct Answer: D
Your Answer:
Explanation

IASbaba
Web: http://ilp.iasbaba.com/ Score:
Email: ilp@iasbaba.com 0.00 / 200
Page 140
2019 - Test 4-
Exam Title :
History & Cu...
Email : ilbsnaa4@gmail.com
Contact :

Solution (d)

जॉइंट�का���हे��सव��लान�ऑफ़�ए�शन�(JCPOA) �जसे�सामा�यत: ईरान�समझौते�के�नाम�से�जाना�जाता�है, ईरान�के�परमाणु�काय��म�पर


एक�अंतरा��ीय�समझौता�है�जो�14 जुलाई�2015 को�ईरान�तथा�P5+1 (सं.रा. सुर�ा�प�रषद�के�पाँच��थायी�सद�य�-चीन, �ांस, �स,
��टे न, सं.रा. अमे�रका�तथा�जम�नी) एवं�यूरोपीय�संघ�के�बीच��वयना�म���आ।�

Source: https://www.thehindu.com/news/national/india-iran-pledge-to-maintain-trade-
levels/article24436444.ece

QUESTION 99.
�न�न�ल�खत�म��से�कौन�सी�अनुसूची�म��दल-बदल��वरोधी��ावधान�ह�?

a) आठव��
b) दसव��
c) छठ��
d) नौव��
Correct Answer: B
Your Answer:
Explanation

Solution (b)

सं�वधान�क��10व��अनुसूची��जसे�लोक��य�तौर�पर�‘दल-बदल��वरोधी�क़ानून' के�तौर�पर�जाना�जाता�है, 1985 म��52व��संशोधन��ारा�जोड़ी


गई�थी।�

QUESTION 100.
‘पो�ु�गीज़�मैन-ऑफ़-वार(Portuguese man-of-war)’ हाल�म��सु�ख़�य��म��था।�यह�संबं�धत�है-

a) �लॉकचैन��ौ�ो�गक��
b) अ�गो�रथम��े �ड�ग�
c) आ�ट�फ��सयल�इंटे�लज�स�
d) उपरो��म��से�कोई�नह��
Correct Answer: D
Your Answer:
Explanation

Solution (d)

पो�ु�गीज़�मैन-ऑफ़-वार�एक�जेली�जैसा��दखने�वाला�समु���जीव�है।�इसे�सामा�य�तौर�पर�‘�लूबोटल’ अथवा�‘�लो�ट�ग�टे रर’ के�नाम�से�जाना


जाता�है।�

जहाँ�अ�धकाँश�जेली�फश�का�डंक�मानव��के��लए�हा�नर�हत�होता�है�और�इससे�ह�क��सी�चुभन�मा��होती�है, वह���लूबोटल�जैसी��जा�तयाँ
जहरीली�होती�ह��और�संपक��म��आने�पर�हा�न�प�ँचा�सकती�ह�।�यहाँ�तक��क�तट�पर�बहकर�आया�एक�मृत��लूबोटल�भी�डंक�मार�सकता�है।�

Source: https://www.thehindu.com/news/national/other-states/portuguese-man-of-war-
spotted-on-goa-beach/article24604555.ece

IASbaba
Web: http://ilp.iasbaba.com/ Score:
Email: ilp@iasbaba.com 0.00 / 200
Page 141

You might also like